Infectious diseases

Ace your homework & exams now with Quizwiz!

ABC for the causes of acute food poisoning

A - Staph. aureus B - Bacilus cereus C - Clostridium perfringens Giardiasis, Amoebiasis (the parasites) have longer incubation period >7 days and prolonged diarrhoea

how long is co-amox taken for animal bite

A 3-day course of co-amoxiclav should be prescribed, to be used immediately

A 34-year-old man was recently diagnosed with HIV. He was started on a drug regime that included ritonavir. He wanted to know how ritonavir can help to prevent progression to AIDS.Which of the following best describes the mode of action of this drug?

'Navir tease a pro' - HIV drugs that end with -navir are protease inhibitors

False positive VDRL/RPR

'SomeTimes Mistakes Happen' (SLE, TB, malaria, HIV)

risk factor for BV or not: having a circumcised partner use of hormonal contraception consistent use of condoms smoking infrequent washing of vulva with water only

- no it is a protective factor - protective factor - reduced risk - yes!! - washing with soaps etc increases risk

NICE recommends either direct referral or seeking specialist advice regarding daily antibiotic prophylaxis for the following groups:

1) Men aged 16 years and over.2) Men and women with recurrent upper UTIs.3) Non-pregnant women with recurrent lower UTIs when the underlying cause is unknown.4) Pregnant women.5) Children and young people under 16 years.6) Men and women with suspected cancer, under 2-week wait rules. For example, have persistent haematuria.The majority of patients with recurrent lower UTIs fall under number 3, like this patient. NICE guidance specifically stipulates seeking specialist advice via advice and guidance rather than directly referring these patients:'...however, due to resource implications and the lower risk of complications for this population, the committee agreed that specialist advice should be sought, rather than a specialist referral.'It is not clear from NICE guidance exactly when advice and guidance should be sought, and what 'no underlying cause' exactly means. There is no clear definition of what constitutes an underlying cause. Numerous medical conditions far-ranging from diabetes, neurological diseases, and benign prostatic hyperplasia, to urinary incontinence, constipation, kidney stones and urological cancers can predispose to recurrent UTIs.The guidance is understood to suggest behavioural measures should be optimised first. The next step is to consider a prescription of either vaginal oestrogen for postmenopausal women or single-dose antibiotic prophylaxis for premenopausal women with an identifiable trigger. Failure of these should lead to seeking advice and guidance from specialist teams regarding the need to refer for investigations, or starting daily antibiotic prophylaxis by the GP.The patient seems to be otherwise healthy and ultrasound results suggest normal bladder function thus referral would not be considered at this stage.

when to do microbiological investigations for diarrhoea

1) You suspect septicaemia 2) there is blood and/or mucus in the stool or 3) the child is immunocompromised

incubation periods

1-6 hrs: Staphylococcus aureus, Bacillus cereus* 12-48 hrs: Salmonella, Escherichia coli 48-72 hrs: Shigella, Campylobacter > 7 days: Giardiasis, Amoebiasis

incubation period of hep b

1-6 months

first line mx for early lyme disease

14-21 day course of oral doxycycline

mrsa mx

1st line - Glycopeptides: vancomycin (but not for VRE or VRSA) or Teicoplanin. Narrow spectrum agents, only available IV and cannot penetrate blood-brain barrier as they are large molecules. Vancomycin is nephrotoxic so levels need to be measured. For VRE - treat with oxazolidinone e.g. linezolid. Narrow spectrum gram-positive agent with excellent penetration into skin and brain. However, can cause bone marrow suppression, peripheral neuropathy and optic neuritis. Other options: Tazocin, daptomycin, tigecycline.

TB mx

2 months of rifampicin, isoniazid, pyrazinamide and ethambutol, then 4 months of rifampicin and isoniazid This is the standard treatment for drug-sensitive tuberculosis, which is split into an initial 2-month intensive phase, followed by a 4-month continuation phase.

incubation period of ebola

2-21 days

Latent tuberculosis treatment options:

3 months of isoniazid (with pyridoxine) and rifampicin, or 6 months of isoniazid (with pyridoxine)

how long is post exposure prophylaxis for HIV given

4 weeks and it is started within 72 hours of exposure and HIV testing is done 45 days (10.5 weeks) after completion of treatment

An 18-year-old man presents worried about blood-borne viruses. He has just returned from backpacking abroad. On the last day, he and his friends had tattoos done to remember their trip, but he is now panicking about whether the equipment was sterile. He has been reading on-line about the risks and requests testing for human immunodeficiency virus (HIV) and hepatitis B and C. You counsel him that tests can be taken straight away if he wishes but that they will need to be repeated as any infection caught from this particular exposure would be too early to show up on tests yet.When should he come back for a blood test for HIV?

4 weeks. Correct answer, HIV tests (for antibodies and p24 antigen) should be taken 4 weeks after possible exposure, by which point most infections can be detected. You might also consider an HIV test at presentation in case of an infection contracted at an earlier date (although at 1 week may be too early for detection), and you should offer to repeat the test at 12 weeks to confirm no infection.

how many doses of tetanus are given over the normal immunisation schedule

5

A 35-year-old man presents with 24 hours of dysuria, urinary frequency and lower abdominal pain. Observations demonstrate; temperature of 37.5ºC heart rate of 70/min blood pressure 120/80 mmHg He is tender in the suprapubic region. His urine dip is positive for nitrites and leucocytes and is negative for blood.What would be the next most appropriate step in this patient's management?

7 day curse of empirical abx for UTI. referral to urology is not routinely required for men who have one uncomplicated lower UTI. NICE advices us that men who present with symptoms suggestive of a lower UTI should be treated empirically ith oral antibiotics such as trimethoprim or nitrofurantoin, depending on local microbiology protocols. The antibiotics should be continued for 7 days. A 3 day course would not be considered sufficient. Antibiotic treatment should not be delayed pending the result of urinary microscopy culture and sensitivity (MC+S), but it is still important to perform this test in order to assess for the presence of resistant bacteria. Waiting for the culture results would be inappropriate in this case since he is symptomatic with a proven infection and therefore requires prompt treatment to prevent ascending infection or sepsis.NICE advises us that men with UTIs should not routinely be referred to urology unless the infection is recurrent. The two-week rule pathway should be used if the man is; Aged 45 years and over with unexplained visible haematuria without urinary tract infection, or visible haematuria that persists or recurs after successful treatment of urinary tract infection. Aged 60 years and over who have unexplained non-visible haematuria and either dysuria or a raised white cell count on a blood test. Intravenous antibiotics are not usually required to treat urinary tract infections unless there is evidence of fever, riggers, chills, vomiting or confusion. Although this patient has a borderline temperature, his other parameters do not suggest that he is septic or unwell enough to require hospital admission. Therefore be reasonable to treat him with oral antibiotics in the first instance.Whilst excluding a sexually transmitted infection may be important, this man has symptoms suggestive of a UTI, and there is nothing noted in the history that suggests he has a sexually transmitted infection. He should therefore be treated with empirical antibiotics in the first instance.

how long do pregnant women need to take abx for for a UTI

7 days

Mantoux test

A Mantoux skin test is another test that determines if a person has been exposed to TB antigens but does not specify if the patient has active or latent TB. This is performed by injecting a small amount of tuberculin into the patient's arm skin (subcutaneously) and the patient returning to have an assessment for the size of the wheal 48 to 72 hours after the test is performed.

A 38-year-old gentleman presents with right upper quadrant pain, fever, and profusely bloody diarrhoea. This came on 3 weeks after returning from India. He was working in rural India for a year and returned to the UK when his contract finished. On examination, he appears dehydrated and he has conjunctival pallor. On abdominal examination, you detect a mass on palpation. You refer him for a CT scan which shows an abscess on the liver. The patient undergoes drainage of the abscess and the surgeon describes the consistency of the contents as 'anchovy sauce'. What organism causes this clinical picture?

A description of the contents for amoebic liver abscesses is described as 'anchovy sauce' - E. histolytica is known to digest hepatic tissue

A 74-year-old man presents to the emergency department with an 8-hour history of headache, vomiting, and blurred vision. Preceding history includes the patient being at a party and drinking brandy. Past medical history includes asthma, type 2 diabetes mellitus and diverticular disease. Observations and blood glucose level are normal.A drug interaction is suspected as the cause.What medication is most likely responsible for this presentation?

A disulfiram-like reaction can occur if metronidazole and alcohol are taken together

A 65-year-old woman, currently on the respiratory ward, is being reviewed during the morning ward round. She seems to be quite confused and unable to recall where exactly she is. All that she is saying is 'it burns...it burns' and points to her lower abdomen. A mid-stream urine shows a growth of 107 organisms/mL. Her past medical history includes hypertension and rheumatoid arthritis for which she takes ramipril and methotrexate respectively.Which of the following medications would be the most appropriate in this scenario?

A pure growth of >10^5 organisms/mL on a mid-stream urine is diagnostic of a urinary tract infection give her nitrofurantoin. not trimethoprim - it is contraindicated due to her being on methotrexate.

You are asked to attend a meeting at a local nursing home. There is currently an increased incidence of MRSA in the patients and a strategy is being drawn up to tackle this. What is the most effective single step to reduce the incidence of MRSA?

hand hygiene

A 27 year old man presents to A+E with large violaceous painless macular patches on his arms and legs as well as generally feeling unwell. His partner also noticed that there was a patch in his mouth as well. He also has a lump in his left groin. On further questioning he regularly injects heroin into his veins and he has confirmed that he shares and reuses needles. What is the most likely diagnosis?

AIDS This is a description of Kaposi's sarcoma, which is an AIDS defining illness. This will have been caused by HIV infection, most likely acquired via injecting drugs intravenously in unclean environments. It is characterised by cutaneous or mucosal lesions. It can also appear on visceral organs but is less common. To confirm the diagnosis, you would need to biopsy the lesions. However you could confirm AIDS by looking at the white blood cell CD4 count 67%

An 8-year-old boy is brought in by his mother with a history of shortness of breath and fever over the last few hours. On examination, he has a toxic appearance, has inspiratory stridor, and is drooling.

The history indicates acute epiglottitis, and this child would obviously need an ambulance to transfer him to the emergency department urgently. The most common cause of acute epiglottitis in children is Haemophilus influenzae type B, and the incidence of acute epiglottitis has fallen since the introduction of the Hib vaccine.

A 29-year-old man presents with a 12 day history of watery diarrhoea that developed one week after returning from India. He had travelled around northern India for two months. On examination he is apyrexial and his abdomen is soft and non-tender. What is the most likely causative organism?

The incubation period and prolonged, non-bloody diarrhoea point towards giardiasis

A 45 year old man presents to A&E with back pain. He describes it in the lower region of his back and says it came on gradually over the past few weeks. He also says he feels feverish and unwell. He has also lost some weight recently, but does not know why. On examination the man looks unkempt and there are needle injection marks on both of his arms. On examination of his back there is point tenderness in the L4 region which is warm to touch. What is the most likely diagnosis?

Discitis This is a classical description of discitis. Symptoms come on gradually with pain on or around the affected area, the patient is feverish and systemically unwell. Intravenous drug users are at high risk for atypical infections and they can often occur in unusual places 76%

A 38-year-old hiker attends the out-of-hours clinic after being bitten by a tick earlier in the day. The tick was safely removed as soon as it was noticed. He is concerned about Lyme disease as he knows it is prevalent in the area. He does not currently have any symptoms but is requesting antibiotics just in case. On examination, there is a small red mark on his left calf in the location of the bite but no rash.What treatment is the most appropriate?

There is no need for prophylactic antibiotics for Lyme disease in asymptomatic patients bitten by a tick

An 84-year-old female with a long standing urinary catheter is admitted with smelly urine. She has no other symptoms or signs. All vital signs are normal. WCC is normal. Three samples taken under sterile conditions from the urinary catheter have all grown Escherichia coli.What is the most accurate definition for this scenario?

Finding bacteria in the urine due to colonisation is is an inevitable consequence of a long term indwelling urinary catheter. This is called asymptomatic bacteruria. Most people who have this condition do not need treatment because the bacteria are not causing any harm. Discuss (3)Improve

mx of cdiff

First episode of C. difficile infection first-line therapy is oral vancomycin for 10 days second-line therapy: oral fidaxomicin third-line therapy: oral vancomycin +/- IV metronidazole Recurrent episode recurrent infection occurs in around 20% of patients, increasing to 50% after their second episode within 12 weeks of symptom resolution: oral fidaxomicin after 12 weeks of symptom resolution: oral vancomycin OR fidaxomicin Life-threatening C. difficile infection oral vancomycin AND IV metronidazole specialist advice - surgery may be considered

what can cause a false negative mantoux test

Immunosuppression can cause a false negative result - including in sarcoidosis, steroid use, AIDS and lymphoma.

A 50 year old man presents with a 3 week history of cough, productive of clear sputum. This affects him more in the evenings but is unrelated to any positional changes.On examination he is comfortable at rest. He has bilateral good air entry on auscultation, with no added breath sounds.Which of the following additional features would prompt a chest x-ray?

Immunosuppressive therapy, such as chemotherapy or biological treatments can cause reactivation of TB.

A 55-year-old woman is admitted to the hospital with suspected pancreatitis. A junior doctor obtains a needlestick injury while attempting to take her blood. The patient has a history of alcohol abuse and intravenous drug misuse.What is the most appropriate step for the doctor to take?

Ask a colleague to complete a risk assessment and take the patient's blood is correct. This is the most appropriate and professional option. Doctors must not manage these alone and it is more appropriate for a colleague to complete these steps with a clear frame of mind and approach to the patient's risk.

A 28 year old male presents to A+E with a 1-week history of fever and constipation. He has no past medical history and returned three days ago from a trip to rural India. On examination, he is hemodynamically stable but febrile, and his pulse is noted to be 35bpm. His abdomen is distended with increased bowel sounds, and there are crops of pigmented macules across his abdomen that blanch on pressure. Given the likely diagnosis, what is the most accurate diagnostic test?

Bone marrow aspirate and culture The patient has presented with signs and symptoms consistent with typhoid. Bone marrow aspirate is the most accurate diagnostic test, though is rarely available in low-resource settings 18%

acute toxoplasmosis

Acute toxoplasmosis in the immunocompetent patient can mimic acute EBV infection (low-grade fever, generalised lymphadenopathy with prominent cervical lymph nodes and malaise) and should be suspected with negative EBV serology. Pregnancy testing and counselling is paramount due to the risk of congenital toxoplasmosis

You receive in your inbox the results of a human immunodeficiency virus (HIV) test. The antibodies and p24 antigen are reported as negative. You check the records and find it was requested by your colleague who is on holiday. The patient is a man who had requested an HIV test after an encounter with a sex worker; your colleague had asked him to attend the sexual health clinic but he had declined, wanting tests done via the GP. The HIV test had been taken 4 weeks after the episode.What should the patient be told about the result?

After an initial negative result when testing for HIV in an asymptomatic patient, offer a repeat test at 12 weeks the combined HIV test for HIV-1 and HIV-2 antibodies and p24 antigen is the preferred test for HIV. HIV infection unlikely but the test needs to be repeated at 12 weeks post-exposure. Correct, although most cases of HIV infection can be detected by 4 weeks, a repeat test at 12 weeks is recommended to confidently exclude the diagnosis.

A 70-year-old woman presents to the GP with a 7-day history of dysuria and urinary urgency. She also needed the toilet 8 times yesterday, which caused great distress.She had a urinary tract infection 12 months ago, for which she needed a catheter during treatment. She has a family history of diabetic nephropathy.The GP suspects another urinary tract infection and a mid-stream urine sample is sent.What aspect of this woman's history necessitates this investigation to be carried out?

Age more than 65 years old is the correct answer. According to NICE guidelines, a mid-stream urine sample should be sent for women with a suspected lower urinary tract infection to look for resistant or atypical organisms with the following indications: Have symptoms that are persistent or do not resolve with antibiotic treatment. Have recurrent UTI (2 episodes in 6 months or 3 in 12 months). Have a urinary catheter in situ or have recently been catheterised. Have risk factors for resistance or complicated UTI such as abnormalities of the genitourinary tract, renal impairment, residence in a long-term care facility, hospitalisation for more than 7 days in the last 6 months, recent travel to a country with increased resistance or previous resistant UTI. Have atypical symptoms. Have visible or non-visible (on urine dipstick) haematuria. Pregnant Age more than 65 years old.

A 29-year-old woman is diagnosed with HIV after being contact traced from a recent sexual partner. She has no symptoms of an infection at this time. Examination of her cardiorespiratory and neurological systems reveals no abnormalities. She is not currently taking any medication. As part of the work-up, her CD4+ cell count is taken which shows the following: CD4+ cell count128/mm3 Hepatitis and sexually-transmitted infection screens are negative.What is the most appropriate management option at this time?

All patients with HIV require highly active anti-retroviral therapy (HAART) at the time of diagnosis. As this patient has a CD4+ cell count <200/mm3, she also requires prophylactic co-trimoxazole, to cover against Pneumocystis jiroveci pneumonia.

A 32-year-old woman attends her GP complaining of general malaise for a week. She complains of headache and mild abdominal pain. She denies any diarrhoea and states that she has been severely constipated. She has just returned from a 3-week holiday in India. On examination her temperature is 38.5ºC, she appears bloated, and there is a sparse macular rash on her chest.What is the likely organism responsible for this presentation?

Although salmonella is a recognised cause of diarrhoea, constipation is more common in typhoid This is a classic presentation of typhoid fever with rose spots. Be aware that constipation is a big clue to typhoid fever

A 29 year old man is seen in clinic with a four-week history of fever, rigors, and abdominal pain. Initial tests for TB, hepatitis viruses and HIV are all negative. He has traveled in the five years to Canada and Australia, and works as an estate agent. He has had several new male sexual partners in the last few months. On examination, he is not jaundiced but is tender in the right hypochondrium. What is the most likely cause of his symptoms?

Amoebic liver abscess This patient has presented with Pyrexia of Unknown Origin (PUO). This is defined as temperatures above 38.3 degrees for greater than three weeks, without a cause identified through initial investigations. An important cause of PUOs are abscesses, and in this patient a hepatic abscess is likely due to his right hypochondrial pain. Those who engage in anal sex are at risk of infection with Entamoeba histolytica infection, which is a cause of sexually acquired amoebic colitis and hepatic amoebic abscesses. These abscesses are known to contain material likened to 'anchovy sauce' once drained. Treatment involves high dose metronidazole/tinidazole and surgical aspiration if high risk of rupture 62%

aspergilloma

An aspergilloma is a mycetoma (mass-like fungus ball) which often colonises an existing lung cavity (e.g. secondary to tuberculosis, lung cancer or cystic fibrosis).Usually asymptomatic but features may include cough haemoptysis (may be severe) Investigations chest x-ray containing a rounded opacity. A crescent sign may be present high titres Aspergillus precipitins

HIV management

Antiretroviral therapy (ART) involves a combination of at least three drugs, typically two nucleoside reverse transcriptase inhibitors (NRTI) and either a protease inhibitor (PI) or a non-nucleoside reverse transcriptase inhibitor (NNRTI). This combination both decreases viral replication but also reduces the risk of viral resistance emergingFollowing the 2015 BHIVA guidelines it is now recommended that patients start ART as soon as they have been diagnosed with HIV, rather than waiting until a particular CD4 count, as was previously advocated. Entry inhibitors - maraviroc (binds to CCR5, preventing an interaction with gp41), enfuvirtide (binds to gp41, also known as a 'fusion inhibitor') - prevent HIV-1 from entering and infecting immune cells Nucleoside analogue reverse transcriptase inhibitors (NRTI) examples: zidovudine (AZT), abacavir, emtricitabine, didanosine, lamivudine, stavudine, zalcitabine, tenofovir general NRTI side-effects: peripheral neuropathy tenofovir: used in BHIVAs two recommended regime NRTI. Adverse effects include renal impairment and ostesoporosis zidovudine: anaemia, myopathy, black nails didanosine: pancreatitis Non-nucleoside reverse transcriptase inhibitors (NNRTI) examples: nevirapine, efavirenz side-effects: P450 enzyme interaction (nevirapine induces), rashes Protease inhibitors (PI) examples: indinavir, nelfinavir, ritonavir, saquinavir side-effects: diabetes, hyperlipidaemia, buffalo hump, central obesity, P450 enzyme inhibition indinavir: renal stones, asymptomatic hyperbilirubinaemia ritonavir: a potent inhibitor of the P450 system Integrase inhibitors block the action of integrase, a viral enzyme that inserts the viral genome into the DNA of the host cell examples: raltegravir, elvitegravir, dolutegravir

A 59-year-old male presents to hepatology clinic following an urgent same day referral from his GP who was concerned about his risk for hepatitis infection. His past medical history includes haemophilia A for which he has required regular blood transfusions since early childhood. An abdominal ultrasound scan shows no evidence of liver cirrhosis. Blood tests have confirmed the suspected diagnosis of acute hepatitis C infection. What would be the most appropriate definitive treatment to commence?

Antiviral combination therapy Given the high transformation rate of acute hepatitis C into the chronic form and the significant risk of liver cirrhosis and hepatocellular carcinoma aggressive initial treatment is of vital importance. This is something which should only be initiated under the care of a specialist doctor. The antiviral agents used, duration of and ultimate effectiveness treatment will depend on the patient's viral load, severity of liver impairment, comorbidities and the viral genotype. Anybody who received blood products prior to 1992 in the UK is at high risk of having contracted hepatitis C infection from a contaminated transfusion. 61%

diagnostic test for tb vs first line ix for tb

As such, sputum culture is required for the optimal finding of active cases of HIV-associated TB.A chest x-ray is a useful first-line investigation for patients with suspected TB however it has poor specificity. Features, such as pleural effusion, lymphadenopathy, and patchy consolidation, are not sufficiently specific to TB and can be seen in other pathologies. This means a chest x-ray is incorrect as it will not be diagnostic.

A 13-year-old boy is admitted to the orthopaedic ward following an operation to internally fixate his fractured femur. It is day 6 post-op and the nurse has called the Foundation Year 1 doctor as the patient has spiked a temperature overnight at 38.9ºC. On assessment the patient's respiratory rate is 30/minute, heart rate 135 bpm, blood pressure is 126/76 mmHg and the patient has a productive cough. What is the most appropriate management option?

As this patient has been in hospital for 6 days, he is showing signs of a late onset hospital-acquired pneumonia and should be treated with an antipseudomonal penicillin (e.g. piperacillin with tazobactam) or antipseudomonal beta-lactam. Discuss (4)Improve

You are working in a GP surgery when you have been asked to review a urine result of a 28-year-old woman who is currently 10 weeks pregnant. The urine sample was collected during her recent appointment with her midwife and the result has returned showing the presence of Escherichia coli. You speak to the patient on the phone to discuss the results and learn that she is well with no history of urinary symptom, abdominal pain or temperature.Which of the following is the most appropriate management option?

Asymptomatic bacteriuria is common. NICE recommends treatment for asymptomatic bacteria in pregnancy as it is a risk factor for pyelonephritis, low birth weight and premature delivery. The current guidance advises treatment for 7 days.

A 15-year-old boy is diagnosed with glandular fever. What is the most appropriate advice to give regarding playing sports?

Avoid contact sports for 4 weeks after having glandular fever.

A 28-year-old woman who is currently 10 weeks pregnant has purulent vaginal discharge. Vulvovaginal swabs have confirmed the presence of Chlamydia trachomatis. She is allergic to penicillin.

Azithromycin, erythromycin or amoxicillin are used in the treatment of Chlamydia trachomatis in pregnancy.As this woman is allergic to penicillin, azithromycin is the most appropriate treatment from the options above.Doxycycline - which is now the first-line treatment in non-pregnant adults - is contraindicated in pregnancy due to the risk of permanent dental discolouration and enamel hypoplasia in the developing fetus.

mx of chlamydia in pregnancy

Azithromycin, erythromycin or amoxicillin may be used to treat Chlamydia in pregnancy

live attenuated vaccines

BCG MMR oral polio yellow fever oral typhoid

A 24-year-old woman presents to your surgery with vaginal discharge. She says it smells quite strongly, but isn't itchy. She has no dysuria or dyspareunia. She has no post-coital bleeding.On examination, there is a watery discharge with an odour. There is no erythema to the labia. Her cervix looks healthy and there is no cervical excitation.What is the most likely diagnosis?

BV

A 31-year-old woman presents as she has noted an offensive, fishy vaginal discharge. She describes a grey, watery discharge. What is the most likely diagnosis?

BV Bacterial vaginosis (BV) describes an overgrowth of predominately anaerobic organisms such as Gardnerella vaginalis. This leads to a consequent fall in lactic acid producing aerobic lactobacilli resulting in a raised vaginal pH.Whilst BV is not a sexually transmitted infection it is seen almost exclusively in sexually active women.Features vaginal discharge: 'fishy', offensive asymptomatic in 50% Amsel's criteria for diagnosis of BV - 3 of the following 4 points should be present thin, white homogenous discharge clue cells on microscopy: stippled vaginal epithelial cells vaginal pH > 4.5 positive whiff test (addition of potassium hydroxide results in fishy odour) Management oral metronidazole for 5-7 days 70-80% initial cure rate relapse rate > 50% within 3 months the BNF suggests topical metronidazole or topical clindamycin as alternatives. Bacterial vaginosis in pregnancy results in an increased risk of preterm labour, low birth weight and chorioamnionitis, late miscarriage it was previously taught that oral metronidazole should be avoided in the first trimester and topical clindamycin used instead. Recent guidelines however recommend that oral metronidazole is used throughout pregnancy. The BNF still advises against the use of high dose metronidazole regimes

clue cells on microscopy what is the dx and tx

BV: oral metronidazole 400mg bd for 5 days.

A man presents with severe vomiting. He reports not being able to keep fluids down for the past 12 hours. You suspect a diagnosis of gastroenteritis and on discussing possible causes he mentions reheating curry with rice the night before. What is the most likely causative organism?

Bacillus cereus characteristically occurs after eating rice that has been reheated

A 27-year-old man attends with a 2-day history of right-sided otalgia and discharge. He is a keen swimmer. On examination, the external auditory canal is swollen and erythematous, with purulent material on the walls.

Bacterial causes of otitis externa are most common, with Pseudomonas aeruginosa being most prevalent. Other causes include Staphylococcus aureus.

A 37 year old presents to the Emergency Department with a two day history of bilateral facial nerve palsy. She is unable to close her eye or mouth without assistance. The rest of her history is unremarkable with the exception of a recent camping holiday in North East America during which she sustained multiple insect bites. What is the likely causative organism?

Borrelia burgdorferi is the causative organism in Lyme disease. Transmission to humans is by bites from the Ixodes tick, common in North America. Symptoms include joint pains, facial nerve palsy, palpitations, headaches and fever.

what vaccines are given at 14

Both the meningitis ACWY and tetanus, diphtheria and polio vaccines are given at 14 years (school year 9) so these options are incorrect.

bacterial vaginosis

Bacterial vaginosis (BV) describes an overgrowth of predominately anaerobic organisms such as Gardnerella vaginalis. This leads to a consequent fall in lactic acid producing aerobic lactobacilli resulting in a raised vaginal pH.Whilst BV is not a sexually transmitted infection it is seen almost exclusively in sexually active women.Features vaginal discharge: 'fishy', offensive asymptomatic in 50% Amsel's criteria for diagnosis of BV - 3 of the following 4 points should be present thin, white homogenous discharge clue cells on microscopy: stippled vaginal epithelial cells vaginal pH > 4.5 positive whiff test (addition of potassium hydroxide results in fishy odour) Management oral metronidazole for 5-7 days 70-80% initial cure rate relapse rate > 50% within 3 months the BNF suggests topical metronidazole or topical clindamycin as alternatives Bacterial vaginosis in pregnancy results in an increased risk of preterm labour, low birth weight and chorioamnionitis, late miscarriage it was previously taught that oral metronidazole should be avoided in the first trimester and topical clindamycin used instead. Recent guidelines however recommend that oral metronidazole is used throughout pregnancy. The BNF still advises against the use of high dose metronidazole regimes

notifiable diseases

Below is a list of notifiable diseases in the UK. The 'Proper Officer' at the Local Health Protection Team needs to be notified. They in turn will notify the Health Protection Agency on a weekly basis. Notable exceptions include: HIV In April 2010 the following diseases were removed from the list: Dysentery Ophthalmia neonatorum Leptospirosis Relapsing fever Therefore, the current notifiable diseases are: Acute encephalitis Acute infectious hepatitis Acute meningitis Acute poliomyelitis Anthrax Botulism Brucellosis Cholera COVID-19 Diphtheria Enteric fever (typhoid or paratyphoid fever) Food poisoning Haemolytic uraemic syndrome (HUS) Infectious bloody diarrhoea Invasive group A streptococcal disease Legionnaires Disease Leprosy Malaria Measles Meningococcal septicaemia Mumps Plague Rabies Rubella Severe Acute Respiratory Syndrome (SARS) Scarlet fever Smallpox Tetanus Tuberculosis Typhus Viral haemorrhagic fever (VHF) Whooping cough Yellow fever

A patient who has recently been on a camping holiday comes to see you as she is concerned that she may have developed Lyme disease. Since returning from holiday she has developed a rash and has felt tired and achy. What is the most appropriate investigation to test for Lyme disease?

Blood test for serology = ELISA is the first-line investigation for suspected Lyme disease in patients with no history of erythema migrans

A 45-year-old man presents to the emergency department with fever and pain in his right hip, which came on gradually over several day. He is concerned as he is now finding it difficult to mobilise because of the pain. His past medical history is relevant for hypertension treated with ramipril and type 2 diabetes treated with metformin and gliclazide. He has no known drug allergies and takes no medications over the counter. On examination, he appears unwell and is pyrexial. There is point tenderness of the right hip, which is warm to the touch, swollen and erythematous. Routine bloods are taken, which show: Haemoglobin (Hb) 132 g/L Mean cell volume (MCV) 87 fL White cell count 12 × 109/L Platelets 231 × 109/L C-reactive protein (CRP) 54 mg/L Hb A1c 50 mmol/mol Blood cultures have been taken and are currently being processed by microbiology. Given the likely diagnosis, which of the following investigations is the gold standard to obtain a definitive diagnosis?

Bone biopsy This man has presented with boney pain and systemic symptoms suggestive of osteomyelitis. Systemic disease such as diabetes mellitus and atherosclerosis are significant risk factors. Bone biopsy with histopathological examination and tissue culture is the gold standard diagnostic investigation for osteomyelitis. Culture and sensitivity results from the sample will also aid in tailoring antimicrobial therapy. 50%

An 18-month-old boy attends the emergency department with his father. He has had a dry, hacking cough and fever over the last 2 days. Dad describes that he coughs continuously and makes a high-pitched gasping noise between coughing fits. During the night, he was coughing so hard that he vomited. Given the likely diagnosis, what long-term complication is this patient at risk of?

Bronchiectasis Whooping cough-mediated damage to the bronchi, particularly in childhood, is associated with the development of bronchiectasis later in life. 64%

A 37 year old female presents to the clinic with a 4-week history of fevers, night sweats, myalgia and joint pain affecting the hips and lower back. She has no past medical history, has no allergies, and works as a research assistant in a veterinary laboratory. She has had no recent travel abroad. On examination, she is hemodynamically stable but looks pale and unwell. There is generalized lymphadenopathy and hepatosplenomegaly, but respiratory, cardiovascular and neurological examinations are otherwise normal. What is the most likely diagnosis?

Brucellosis This patient has presented with a Pyrexia of Unknown Origin (PUO), in addition to non-specific systemic symptoms and hepatosplenomegaly. As a research assistant in a veterinary laboratory, this suggests she has contact with animals, dead meat and/or bodily samples. She is therefore at risk of Brucellosis, and her symptoms correlate with this diagnosis. Blood culture will be needed to confirm this diagnosis. Other classical clues in questions relating to brucellosis include work in abattoirs or consumption of unpasteurized dairy products in high risk countries 63%

EBV associated malignancies

Burkitt's lymphoma Hodgkin's lymphoma nasopharyngeal carcinoma

Sarah is a 21-year-old woman who comes to see you after testing positive for gonorrhoea from a vulvovaginal swab taken last week. Despite information and advice, she is not willing to attend a sexual health clinic and would like you to prescribe her treatment.She has a phobia of needles and refuses intramuscular ceftriaxone.Which of the following options is the most appropriate alternative oral regimen?

Cefixime and azithromycin

renal transplant + infection =

CMV

A previously well 24-year-old female presents with a one-day history of profuse diarrhoea, concerned after having noticed some bright red blood in the stool. For the past 3 days, she had been feeling generally unwell. She denies eating anything unusual although she did attend a barbecue four days ago.Which of the following organisms is most likely responsible for her presentation?

Campylobacter infection is the most common bacterial cause of infectious intestinal disease in the UK. This patient has presented with the hallmark symptoms of prodrome and bloody diarrhoea. The incubation period for Campylobacter is 1-6 days which is in keeping with the presentation for this patient.

Campyloacter jejuni

Campylobacter is the commonest bacterial cause of infectious intestinal disease in the UK. The majority of cases are caused by the Gram-negative bacillus Campylobacter jejuni. It is spread by the faecal-oral route and has an incubation period of 1-6 days.Features prodrome: headache malaise diarrhoea: often bloody abdominal pain: may mimic appendicitis Management usually self-limiting the BNF advises treatment if severe or the patient is immunocompromised. Clinical Knowledge summaries also recommend antibiotics if severe symptoms (high fever, bloody diarrhoea, or more than eight stools per day) or symptoms have last more than one week the first-line antibiotic is clarithromycin ciprofloxacin is an alternative although the BNF states that 'Strains with decreased sensitivity to ciprofloxacin isolated frequently' Complications Guillain-Barre syndrome may follow Campylobacter jejuni infections reactive arthritis septicaemia, endocarditis, arthritis

Mx of pyelonephritis

Cefalexin for 14 days is correct as acute pyelonephritis is treated with a broad-spectrum cephalosporin (such as cefalexin) or quinolone for 14 days.

Cellulitis

Cellulitis is a term used to describe an inflammation of the skin and subcutaneous tissues, typically due to infection by Streptococcus pyogenes or Staphylcoccus aureus.Features commonly occurs on the shins erythema, pain, swelling there may be some associated systemic upset such as fever The diagnosis of cellulitis is clinical. No further investigations are required in primary care. Bloods and blood cultures may be requested if the patient is admitted and septicaemia is suspected. They recommend the following that we admit for intravenous antibiotics the following patients: Has Eron Class III or Class IV cellulitis. Has severe or rapidly deteriorating cellulitis (for example extensive areas of skin). Is very young (under 1 year of age) or frail. Is immunocompromized. Has significant lymphoedema. Has facial cellulitis (unless very mild) or periorbital cellulitis. The following is recommend regarding Eron Class II cellulitis: Admission may not be necessary if the facilities and expertise are available in the community to give intravenous antibiotics and monitor the person - check local guidelines. Other patients can be treated with oral antibiotics. Management The BNF recommends flucloxacillin as first-line treatment for mild/moderate cellulitis. Clarithromycin, erythromycin (in pregnancy) or doxycyline is recommended in patients allergic to penicillin.NICE recommend that patients severe cellulitis should be offered co-amoxiclav, cefuroxime, clindamycin or ceftriaxone.

oedema at bite site, but can develop other features like hepatomegaly, cardiomyopathy

Chagas (trypansomiasis)

chancroid

Chancroid is a cause of sexually-acquired genital ulcers in the tropics. These ulcers are multiple, have ragged edges and a granular base. It can also be associated with painful, unilateral inguinal lymphadenopathy. The causative organism is Haemophilus ducreyi.

ix for TB

Chest X-ray Sputum samples for culture and sensitivity testing (at least three needed - may need to consider lavage or sputum induction if cannot produce) Samples from non-pulmonary sites: may need biopsy and needle aspiration Samples are stained with Ziehl-Neelsen or Auramine staining for direct microscopy For culture, Lowenstein-Jensen (LJ) media is needed. PCR - GeneXpert, rapid results with additional drug sensitivity tested as well. Interferon-Gamma Release Assays (IGRAs) - cannot tell difference between latent or active TB, and not useful in very young or immunosuppressed patients. Results are obtained rapidly, however, and are not affected by prior BCG. Mantoux test - usually offered to contacts of infected patients. Positive in those who have had the BCG vaccine.

A 24-year-old female patient who is currently 22 weeks pregnant calls the GP surgery requesting a telephone consultation. During this, she reveals that she has recently tested positive for Chlamydia trachomatis after finding out her partner had tested positive. She is asymptomatic.She asks you whether she requires any treatment?

Chlamydia trachomatis in pregnant and breastfeeding women should be treated with azithromycin. This course is usually given as a 1g stat dose followed by two days of 500mg OD. A test of cure should be arranged after 3 weeks. If she is symptomatic, then a referral to genitourinary medicine (GUM) should be arranged to exclude other sources of infection.In non-pregnant women, doxycycline would be first-line. This is contraindicated in pregnancy.Ceftriaxone is the first-line treatment for gonorrhoea or suspected pelvic inflammatory disease (PID).

A 34 year old male patient with a history of hepatitis B is seen in clinic with the following blood test results 12 months ago: HBsAg - Positive HBeAg - Negative Anti-HBc(IgG) - Positive Anti-HBc(IgM) - Negative Anti-HBe - Positive Anti-HBs - Negative ALT - 40 AST - 32 Repeat blood tests now show the following: HBsAg- Positive HBeAg- Positive Anti-HBc(IgG) - Positive Anti-HBc(IgM) - Negative Anti-HBe - Positive Anti-HBs - Negative ALT - 150 AST - 102 What is the most likely explanation for these results?

Chronic carrier with low infectivity moved into immune escape phase The blood tests last year showed he was a chronic carrier (as HBsAg positive and anti-HBc IgG positive, but IgM negative). The fact he was HBeAg negative and anti-HBe positive, shows he was of low infectivity, and (unless he was in compensated liver failure) he was not given any medications for the Hepatitis B as a result. The main difference between these blood tests and the new ones is that the HBeAg is now positive, and the LFTs are now raised. This is because the virus has mutated despite anti-HBe being present, and the rise in LFTs shows he is getting liver inflammation as a result. Patients with chronic hepatitis B with low infectivity are screened yearly to check for this progression, as they are a significant source of transmission and are at particular risk of liver disease, cirrhosis, and hepatocellular carcinoma 50%

A 24-year-old female presents to her general practitioner with 3 days of diarrhoea. There is no blood in the stools or history of fever. She is opening her bowels three times a day with watery stool. She has a background of rheumatoid arthritis. She has well-controlled on methotrexate and adalimumab. She does not smoke or drink alcohol.On examination, she has very mild abdominal tenderness. There is no guarding and her abdomen is soft. Bowel sounds are present.Her general practitioner arranges a stool microscopy, which grows Campylobacter jejuni .What is the most appropriate management?

Clarithromycin is the correct answer. She is immunocompromised due to her disease-modifying treatment for rheumatoid arthritis. Although her infection would not be classed as severe, current guidelines advocate treating such patients with antibiotics. The first choice would be clarithromycin.Ciprofloxacin is incorrect. This would be the second line choice if clarithromycin was not tolerated or contraindicated.

A 60 year old male presents to A&E with a painful, immobile left leg. He recalls suffering an injury a week ago when he bumped against the kitchen table. His past medical history includes insulin-dependent type 2 diabetes mellitus. He is allergic to penicillin. He is febrile and tachycardic. On examination, there is marked tenderness, warmth and swelling over the left shin. Blood cultures grew Gram positive cocci in clusters. MRI showed periosteal elevation of the bone. Which is the most appropriate antibiotic treatment?

Clindamycin and fusidic acid This is the treatment of choice for osteomyelitis in penicillin allergic patients. Fusidic acid can be added for the initial 2 weeks, and clindamycin should be continued for 6 weeks otherwise: Flucloxacillin and rifampicin This is the treatment of choice for osteomyelitis, which targets the most common causative organism, Staphylococcus aureus. This is inappropriate here due to his penicillin allergy. Rifampicin can be added for the initial 2 weeks 28%

A 45-year-old man is recovering in hospital following a total hip replacement. He develops a profuse and watery diarrhoea. Several other patients have been suffering from similar symptoms.

Clostridium dificile can spread rapidly on surgical wards. The use of broad spectrum prophylactic antibiotics during arthroplasty surgery can increase the risk.

A 61-year-old man presents to the emergency department with his wife. He has been feeling feverish over the past 2 days, but got particularly anxious this morning as he has been experiencing continuous spasms in his jaw, followed by a stiffening sensation. He has no past medical history of note and does not take any regular medications. He has no known drug allergies. On examination, his facial muscles appear contracted and fixed in a grin-like expression. A bandage is visible on his right hand. When explicitly asked, he reports he cut his finger on a rose bush in his garden 6 days ago. Given the likely diagnosis, what is the most likely causative organism?

Clostridium tetanii This man presents with fever and gradual-onset muscle spasms and trismus of the jaw, consistent with the features of tetanus. Tetanus causes a progressive spastic paralysis and requires prompt treatment with tetanus immunoglobulin to prevent complications including respiratory failure and asphyxia. C. tetani is the causative agent, and is transmitted through exposure to contaminated garden soil, dirt, manure and rusty metals. c botulinism causes flaccid paralysis

A 61-year-old man presents to the emergency department with his wife. He has been feeling feverish over the past 2 days, but got particularly anxious this morning as he has been experiencing continuous spasms in his jaw, followed by a stiffening sensation. He has no past medical history of note and does not take any regular medications. He has no known drug allergies. On examination, his facial muscles appear contracted and fixed in a grin-like expression. A bandage is visible on his right hand. When explicitly asked, he reports he cut his finger on a rose bush in his garden 6 days ago. Given the likely diagnosis, what is the most likely causative organism?

Clostridium tetanii This man presents with fever and gradual-onset muscle spasms and trismus of the jaw, consistent with the features of tetanus. Tetanus causes a progressive spastic paralysis and requires prompt treatment with tetanus immunoglobulin to prevent complications including respiratory failure and asphyxia. C. tetani is the causative agent, and is transmitted through exposure to contaminated garden soil, dirt, manure and rusty metals. 70%

A 34-year-old man presents to the sexual health clinic after unprotected intercourse with a female partner. The encounter happened four weeks ago and today he is complaining of coryzal symptoms accompanied by myalgia. He has a past medical history of asthma, controlled with salbutamol. The doctor counsels him on HIV testing, to which he agrees.What is the most appropriate option regarding his management?

Combination tests (HIV p24 antigen and HIV antibody) are now standard for the diagnosis and screening of HIV HIV p24 antigen tests tend to turn out positive between 1 and 4 weeks post-exposure, whilst HIV antibody tests turn out positive between 4 weeks and 3 months post-exposure. When a patient at risk tests positive, the guidelines suggest repeating the test to confirm the diagnosis before starting any treatment.

A 23 year old female presents to A+E with a 10-day history fever, malaise, dry cough, diarrhoea and right upper quadrant pain. These symptoms began 3 days after she returned from a holiday in South Africa, where she was staying at a lake resort. On examination, she is hemodynamically stable but pyrexic, and hepatosplenomegaly can be felt on palpation of her abdomen. An urticarial rash can be seen across her chest and trunk. What medications should be prescribed?

Corticosteroids and praziquantel Steroids are needed in acute schistosomiasis to suppress the hypersensitivity reaction, while praziquantel is used to treat the adult worms. Repeated treatment with praziquantel will be needed 2-3 months later to ensure all adult worms are destroyed 39%

27-year-old man presents to the emergency department with headache, lethargy and fever. Physical examination reveals pain during neck flexion. A head CT scan is normal, but the evaluation of cerebrospinal fluid obtained by lumbar puncture reveals encapsulated organisms visible by India ink.What is the most likely causative organism?

Cryptococcus neoformans - stains with India ink

A 28-year-old female sex worker attends the emergency department with a severe headache and fever. On examination you elicit some neck stiffness and and mild photophobia and as such a lumbar puncture is performed. The results of the lumbar puncture show a yeast and a capsule in the CSF stained with India ink.Which of the following diagnoses is most likely?

Cryptococcus neoformans is an encapsulated yeast and an obligate aerobe that can live in both plants and animals. Infection with this organism is termed cryptococcosis and usually occurs in the lungs, however in immunocompromised individuals fungal meningitis and encephalitis can occur. A key group of patients affected are HIV positive . Given this patients history as a sex worker it is possible that they have an undiagnosed HIV infection which may have progressed to AIDS.Furthermore, cryptococcus neoformans stains well with india ink allowing it to be detected on a lumbar puncture. Discuss (2)Improve

A 33-year-old known to be HIV positive presents with a 2 day history of diarrhoea. What is the most likely cause of his diarrhoea?

Cryptosporidium is the most common cause of diarrhoea in patients with HIV infection. Histoplasmosis may cause respiratory infection in HIV patients. Mycobacterium avium intracellulare and giardiasis are known causes of diarrhoea in HIV patients but are not as common as Cryptosporidium infection Discuss (4)Improve

serpiginous marks representing larval tracks

Cutaneous larva migrans (CLM)

A 56-year-old man with a longstanding history of chronic kidney disease (CKD) undergoes a renal transplant. The operation is successful, but four weeks later he notices blurring of his vision in his right eye. He is currently taking prednisolone and tacrolimus maintenance therapy. On fundoscopy, a mixture of cotton-wool spots, infiltrates and haemorrhages can be seen in the right eye. What is the most likely diagnosis?

Cytomegalovirus (CMV) infection is important to consider in renal transplant patients. The 'mixture of cotton-wool spots, infiltrates and haemorrhages' is referring to the characteristic 'pizza-pie' appearance on fundoscopy.

Diphtheria

Diphtheria is caused by the Gram positive bacterium Corynebacterium diphtheriaePathophysiology releases an exotoxin encoded by a β-prophage exotoxin inhibits protein synthesis by catalyzing ADP-ribosylation of elongation factor EF-2 Diphtheria toxin commonly causes a 'diphtheric membrane' on tonsils caused by necrotic mucosal cells. Systemic distribution may produce necrosis of myocardial, neural and renal tissuePossible presentations recent visitors to Eastern Europe/Russia/Asia sore throat with a 'diphtheric membrane' - grey, pseudomembrane on the posterior pharyngeal wall bulky cervical lymphadenopathymay result in a 'bull neck' appearanace neuritis e.g. cranial nerves heart block Investigations culture of throat swab: uses tellurite agar or Loeffler's media Management intramuscular penicillin diphtheria antitoxin

CMV

Cytomegalovirus (CMV) is one of the herpes viruses. It is thought that around 50% of people have been exposed to the CMV virus although it only usually causes disease in the immunocompromised, for example people with HIV or those on immunosuppressants following organ transplantation.Pathophysiology infected cells have a 'Owl's eye' appearance due to intranuclear inclusion bodies Patterns of disease: Congenital CMV infection = features include growth retardation, pinpoint petechial 'blueberry muffin' skin lesions, microcephaly, sensorineural deafness, encephalitiis (seizures) and hepatosplenomegaly CMV mononucleosis infectious = mononucelosis-like illness may develop in immunocompetent individuals CMV retinitis = common in HIV patients with a low CD4 count (< 50) presents with visual impairment e.g. 'blurred vision'. Fundoscopy shows retinal haemorrhages and necrosis, often called 'pizza' retina IV ganciclovir is the treatment of choice CMV encephalopathy = seen in patients with HIV who have low CD4 counts CMV pneumonitis CMV colitis

Brucellosis (Undulant Fever)

Definition Brucellosis is caused by the gram-negative, aerobic and intracellular bacillus of Brucella spp. Epidemiology The disease is endemic worldwide, but particularly in areas with high levels of animal farming or animal-to-human contact. Animals affected include cattle, goats, sheep, camels, foxes, and dogs. The Middle East and North Africa are particularly high risk, as well as parts of Europe including Portugal, Spain, Greece and Turkey. Transmission In travellers, this is commonly via consumption of untreated milk/dairy (especially unpasteurized) products, as well as raw meat or liver. Abbatoir workers, meat packers, vets, and hunters can also acquire it through the skin or mucous membrane contact (e.g. conjunctiva from eye splashes, or needlestick injury). The most common mode of transmission in farmers is via inhalation. It is very important to inquire about animal exposure and consumption of unpasteurized dairy products in any returning traveller with possible signs/symptoms consistent with brucellosis. Clinical Features The incubation period is about 5-30 days or longer (can be many months). The presentation is often non-specific, with fever, weight loss, night sweats, lymphadenopathy and joint pain/myalgia or spinal tenderness. It is an important cause of Pyrexia of Unknown Origin (PUO) in a returning traveller, with fevers that are classically remittent (temperature remains normal throughout the day and fluctuates more than 1 degree over 24 hours). Hepatosplenomegaly occurs in about 1/3 of patients, and patients often look pale and unwell on presentation. Other less common signs include: Testicular pain due to orchitis Respiratory signs: consolidation, hilar lymphadenopathy, pleural effusions, lung nodules. These are often associated with a dry cough. Focal brain or cranial nerve lesions Uveitis/conjunctivitis Skin rashes Complications Endocarditis Sacroiliitis or osteomyelitis Epididymo-orchitis ITP CNS involvement: spinal syndromes, peripheral neuropathy, cerebellar ataxia. Abscess formation, usually hepatic Investigations Gold standard: blood cultures with the isolation of Brucella spp using Casteneda's medium Antibody testing if culture not possible (particularly towards O-polysaccharide). Raised serum brucella agglutinins or Rose Bengal Test - need additional confirmatory testing in addition to this. CSF culture or PCR MRI scanning for brucellar spondylodiscitis Liver or bone marrow biopsy in certain cases. Management Treat with Doxycycline, Rifampicin and Gentamicin (or just Doxycycline and Gentamicin) Prevention depends on public health measures and control of infection in animals via vaccination and surveillanc

brucellosis

Definition Brucellosis is caused by the gram-negative, aerobic and intracellular bacillus of Brucella spp. Epidemiology The disease is endemic worldwide, but particularly in areas with high levels of animal farming or animal-to-human contact. Animals affected include cattle, goats, sheep, camels, foxes, and dogs. The Middle East and North Africa are particularly high risk, as well as parts of Europe including Portugal, Spain, Greece and Turkey. Transmission In travellers, this is commonly via consumption of untreated milk/dairy (especially unpasteurized) products, as well as raw meat or liver. Abbatoir workers, meat packers, vets, and hunters can also acquire it through the skin or mucous membrane contact (e.g. conjunctiva from eye splashes, or needlestick injury). The most common mode of transmission in farmers is via inhalation. It is very important to inquire about animal exposure and consumption of unpasteurized dairy products in any returning traveller with possible signs/symptoms consistent with brucellosis. Clinical Features The incubation period is about 5-30 days or longer (can be many months). The presentation is often non-specific, with fever, weight loss, night sweats, lymphadenopathy and joint pain/myalgia or spinal tenderness. It is an important cause of Pyrexia of Unknown Origin (PUO) in a returning traveller, with fevers that are classically remittent (temperature remains normal throughout the day and fluctuates more than 1 degree over 24 hours). Hepatosplenomegaly occurs in about 1/3 of patients, and patients often look pale and unwell on presentation. Other less common signs include: Testicular pain due to orchitis Respiratory signs: consolidation, hilar lymphadenopathy, pleural effusions, lung nodules. These are often associated with a dry cough. Focal brain or cranial nerve lesions Uveitis/conjunctivitis Skin rashes Complications Endocarditis Sacroiliitis or osteomyelitis Epididymo-orchitis ITP CNS involvement: spinal syndromes, peripheral neuropathy, cerebellar ataxia. Abscess formation, usually hepatic Investigations Gold standard: blood cultures with the isolation of Brucella spp using Casteneda's medium Antibody testing if culture not possible (particularly towards O-polysaccharide). Raised serum brucella agglutinins or Rose Bengal Test - need additional confirmatory testing in addition to this. CSF culture or PCR MRI scanning for brucellar spondylodiscitis Liver or bone marrow biopsy in certain cases. Management Treat with Doxycycline, Rifampicin and Gentamicin (or just Doxycycline and Gentamicin) Prevention depends on public health measures and control of infection in animals via vaccination and surveillance.

leprosy

Definition Leprosy is a disease endemic in a large number of developing countries, particularly India, Brazil and Indonesia. It is associated with severe morbidity, reduced psychosocial functioning and stigmatization. Clinical Features Leprosy manifests in a number of different ways due to a range of factors, most important being host immunology and bacterial virulence/initial infectious load. At one end of the spectrum is disseminated lepromatous/ multibacillary leprosy;Where the host immune system cannot contain the bacteria and it becomes widely disseminatedCauses peripheral nerve inflammation and damage, as well as dermatological lesions including nodules, hypoesthetic patches and the development of 'leonine'-like facial appearance.Nerve thickening may be felt on palpation, with the most commonly affected nerves being the ulnar, median, radial cutaneous, greater auricular, common peroneal and posterior tibial nerves. At the other end of the spectrum is tuberculoid/paucibacillary leprosy;Where the immune system is able to control the infection and a milder form of nerve damage and dermatological manifestations occur.The nerve damage in all forms can lead to contractures, ulceration and deformity in the long term. Diagnosis Definitive diagnosis is based on clinical assessment and findings of acid-fast bacilli from biopsies/smears. Treatment Treatment of multibacillary leprosy involves the use of dapsone, rifampicin and clofazimine (an immunosuppressive agent, not needed in paucibacillary disease) for 12-24 months. Thalidomide is another treatment option for non-pregnant individuals. Patients commenced on medications need to be monitored closely throughout the course of treatment for immunological complications known as type I and II (erythema nodosum lepromum) reactions, which require hospital in-patient treatment. Side-effects of dapsone include methaemoglobinaemia, agranulocytosis, Stevens-Johnson Syndrome and the DRESS syndrome, and it can also trigger a haemolytic crisis is G6PD deficiency. Clofozamine can cause abnormal skin pigmentation. Differential Diagnosis The differential diagnosis for other causes of thickened peripheral nerves: Inherited diseases - Charcot Marie Tooth, Refsum's Disease and Neurofibromatosis Endocrinological - Acromegaly Other - AL amyloidosis

A 22 year old man presents to A&E after travelling back from the Amazon rainforest. He has a high fever of 39.1 which is constant, nausea and vomiting, diarrhoea, headache, pain behind the eyes and arthralgia. On examination he has hepatomegaly and a positive tourniquet test. What is the most likely diagnosis?

Dengue Fever This is a classical description of Dengue fever, which presents similarly to malaria, Chikungunya and Zika virus. It is transmitted via mosquitos, particularly the day biting Aedes mosquito. It can be a very severe infection and can lead to haemorrhagic signs. The positive tourniquet test is recommended by the WHO to differentiate the infection from acute gastroenteritis. The test involves inflating a blood pressure cuff to midway between systolic and diastolic for 5 minutes. You then look to see if there are ten or more petechiae per inch squared. If there are, it is a positive test and dengue is suspected 63%

Dengue Fever

Dengue fever is a viral infection that can progress to viral haemorrhagic fever (other examples include yellow fever, Lassa fever, Ebola).Aetiology dengue virus is a RNA virus of the genus Flavivirus transmitted by the Aedes aegypti mosquito incubation period of 7 days. clinical features: Patients with dengue fever can be classified as follows: dengue fever:without warning signswith warning signs severe dengue (dengue haemorrhagic fever). Dengue fever fever headache (often retro-orbital) myalgia, bone pain and arthralgia ('break-bone fever') pleuritic pain facial flushing (dengue) maculopapular rash haemorrhagic manifestations e.g. positive tourniquet test, petechiae, purpura/ecchymosis, epistaxis 'warning signs' include:abdominal painhepatomegalypersistent vomitingclinical fluid accumulation (ascites, pleural effusion) Severe dengue (dengue haemorrhagic fever) this is a form of disseminated intravascular coagulation (DIC) resulting in:thrombocytopeniaspontaneous bleeding around 20-30% of these patients go on to develop dengue shock syndrome (DSS) Investigations typically blood resultsleukopenia, thrombocytopenia, raised aminotransferases diagnostic testsserologynucleic acid amplification tests for viral RNANS1 antigen test Treatment entirely symptomatic e.g. fluid resuscitation, blood transfusion etc no antivirals are currently available

A 32-year-old male patient, with no significant past medical history, presented through the emergency department after recently returning from India, where he had undertaken a prolonged period of farming. He complained of continuous high-grade fevers for 9 days, bilateral subconjunctival haemorrhages with retro-orbital pain and severe myalgia.On examination, there was maculopapular blanching erythema over the body which spared his palms and soles.What is the most likely diagnosis here?

Dengue is a viral disease transmitted by mosquitoes (especially Aedes aegypti) and is widely distributed throughout the tropics and subtropics. Dengue classically presents with high fever, retro-orbital headaches, body aches, exanthem, and generalised lymphadenopathy.

A 60-year-old male patient with known chronic hepatitis B presents with a two week history of right upper quadrant pain, jaundice and weight loss. Observations are unremarkable and he is apyrexial.What is the most likely cause of this man's symptoms?

Deterioration in patient with hepatitis B - ? hepatocellular carcinoma

A 49-year-old man attends the urgent care centre complaining of myalgia. He has just returned from a 6-week hiking trip across Hungary, Austria and the Czech republic, which he had to cut short as he became feverish and felt unable to continue. He has hypertension treated with ramipril and has no known drug allergies. On examination, he appears pyrexial, but comfortable. On his arm, he has a circular rash of 12 cm in diameter, which is red, with a slightly elevated darker centre surrounded by a central ring-like clearing. He reports having first noticed this rash while on his trip, during which he was 'bitten all over', having forgotten to pack insect repellent. The rash first appeared over one of these bites and grew rapidly by 1 cm each day, but was not painful so he largely ignored it. His observations are as follows: HR 72 bpm BP 130/85 mmHg Temperature 38.3 °C Oxygen saturation 99% in air RR 14 breaths per minute Cardiovascular and respiratory examinations are both normal. Given the most likely diagnosis, what is the most appropriate treatment?

Doxycycline This patient with nonspecific fever, myalgia, and erythema migrans (a bullseye-shaped rash) has developed Lyme disease, acquired from a tick bite following travel to an endemic region of Northern Europe. Lyme disease is a bacterial infection caused by Borrelia burgdorferi. First-line treatment is with doxycycline for 2-3 weeks. 75%

A 23 year old Mexican man who emigrated two months ago presents to his GP with back pain and a fever. He keeps goats and other livestock and drinks unpasteurised milk. He also has arthralgia in his knees and on examination his liver edge and spleen are palpable 3 cm below the costal margins. What is the most appropriate treatment for this condition?

Dual antibiotic therapy This is a description of Brucellosis, the most common zoonosis worldwide. It is known as a great mimic as it can have many varied non-specific presentations. The stem points to the condition as the patient works with livestock and drinks unpasteurised milk, both of which are risk factors 34%

atypical lymphocytes + ibiza trip

EBV - IM - glandular disease

A 35-year-old man presents to the emergency department with a 3-month history of fevers and profuse bloody diarrhoea. He has also had associated crampy abdominal pain. On examination, there is right upper quadrant tenderness, his temperature is 38.1ºC, heart rate is 95 beats per minute, blood pressure is 125/83 mmHg, respiratory rate is 12 breaths per minute and his oxygen saturations are 95% on room air.He has no past medical history, nor has he been in contact with anyone else who is currently ill. The symptoms started a month after returning from Mexico.What is the most likely causative agent?

Entamoeba histolytica is correct. This patient has presented with features of dysentery (bloody diarrhoea, fever, and abdominal cramps) following travelling. There has been a long incubation period, along with profuse bloody diarrhoea, fevers, and right upper quadrant pain (suggesting liver involvement and the potential formation of a liver abscess), making the diagnosis likely to be amoebiasis. Amoebiasis is also endemic in South America. The causative organism in amoebiasis of Entamoeba histolytica.

A 30-year-old woman presents to her GP with pain and swelling in her left shin for four days. She is 8 weeks pregnant and has no concerns about her baby. She has suffered from migraines in the past but has no other past medical history. She takes folic acid and has an allergy to penicillin.On examination there is erythema, swelling and tenderness of her left shin. Her temperature is 36.7ºC and other observations are normal. A diagnosis of cellulitis is made.What would be the most suitable treatment for this patient?

Erythromycin is the antibiotic of choice for cellulitis in pregnancy if the patient is penicillin allergic

A 55-year-old woman is currently being treated for tuberculosis and reports blurring of her vision with painful eyes. What is the most likely cause of her symptoms?

Ethambutol Ethambutol can cause ocular toxicity, particularly if there is impairment of renal function. Patients should be warned of this side effect before initiating treatment and need immediate evaluation for possible optic neuritis. 68%

mx of septic arthritis

For management of septic arthritis below the ankle in individuals with diabetes mellitus, see Diabetic foot infections, antibacterial therapy. Seek specialist advice if prostheses present. Flucloxacillin Suggested duration of treatment 4-6 weeks (longer if infection complicated). If penicillin-allergic, clindamycin Suggested duration of treatment 4-6 weeks (longer if infection complicated). If meticillin-resistant Staphylococcus aureus suspected, vancomycin (or teicoplanin) Suggested duration of treatment 4-6 weeks (longer if infection complicated). If gonococcal arthritis or Gram-negative infection suspected, cefotaxime (or ceftriaxone) Suggested duration of treatment 4-6 weeks (longer if infection complicated; treat gonococcal infection for 2 weeks).

Sarah is a 21-year-old woman who comes to see you after testing positive for gonorrhoea from a vulvovaginal swab taken last week. Despite information and advice, she is not willing to attend a sexual health clinic and would like you to prescribe her treatment.She has a phobia of needles and refuses intramuscular ceftriaxone.Which of the following options is the most appropriate alternative oral regimen?

For patients with gonorrhoea, a combination of oral cefixime + oral azithromycin is used if the patient refuses IM ceftriaxone

Patients may be asymptomatic or present with a white/grey, "fishy" offensive-smelling discharge and diagnosis is on confirmation of a higher than normal vaginal pH (above 4.5), the presence of clue cells on microscopy and/or a positive whiff test.

Gardenerella vaginalis

A 27-year-old man who has sex with men (MSM) presents to the sexual health clinic with painless genital ulceration. Which of these organisms is most likely to be the cause? HSV or treponema pallidum

Genital herpes is mostly associated with painful ulceration, while syphilis presents mostly with painless ulceration

genital warts

Genital warts (also known as condylomata accuminata) are a common cause of attendance at genitourinary clinics. They are caused by the many varieties of the human papillomavirus HPV, especially types 6 & 11. It is now well established that HPV (primarily types 16,18 & 33) predisposes to cervical cancer.Features small (2 - 5 mm) fleshy protuberances which are slightly pigmented may bleed or itch Management topical podophyllum or cryotherapy are commonly used as first-line treatments depending on the location and type of lesionmultiple, non-keratinised warts are generally best treated with topical agentssolitary, keratinised warts respond better to cryotherapy imiquimod is a topical cream that is generally used second line genital warts are often resistant to treatment and recurrence is common although the majority of anogenital infections with HPV clear without intervention within 1-2 years

A 78 year old man is brought to the Emergency Department from his care home with a 24-hour history of deteriorating mental state, agitation and fever. His past medical history is notable for Alzheimer's disease, benign prostatic hypertrophy and recurrent episodes of urine retention for which he has a long-term indwelling catheter. On examination, he is hemodynamically stable, but pyrexial and confused. The catheter is draining foul-smelling, turbid yellow urine, and he displays some suprapubic tenderness. Respiratory and cardiovascular examinations are unremarkable. Urine culture reveals evidence of Pseudomonas growth. Given the likely diagnosis, which antibiotic is most appropriate?

Gentamicin Aminoglycosides are the agents of choice for pseudomonal urinary tract infections. This is also the only anti-pseudomonal agent listed. Quinolones are alternatives often used in cases such as this 34%

A 28-year-old woman presents 2 weeks after returning from a holiday kayaking with her family.She reports feeling exhausted as she has been having ongoing fatty diarrhoea with abdominal pain, bloating and flatulence, all of which started while abroad, which appear worse when she takes any dairy.What is the most likely cause of her presentation?

Giardiasis is the correct answer as the patient is presenting with the typical symptoms of abdominal pain, bloating/flatulence, steatorrhoea, lethargy after ongoing diarrhoea and has recently travelled. Foreign travel and river/lake water swimming (or drinking) are risk factors. Lactose intolerance can develop after giardiasis. She would benefit from treatment with metronidazole.

A 15 year old boy presents to the GP after being notified through contact tracing that another child at his school has tested positive for active pulmonary Tuberculosis . He has no cough or fever and feels well within himself. The doctor decides to perform a tuberculin skin test and there is an induration of 1mm size where the tuberculin was injected. What is the next best step in the management of this patient ?

Give BCG vaccination According to NICE guidelines, BCG vaccination should be given to tuberculin skin test negative (mantoux negative) contacts of patients with confirmed pulmonary and laryngeal TB, who have not been previously vaccinated and are under the age of 35 or are over the age of 35 and work in healthcare. This patient had a negative test with only 1mm induration, a positive result would have been 5mm or more 49%

A 35 year old male gardener presents to A+E, having sliced his leg on an old sheet of corrugated metal he had lifted out of the soil. He has no medical conditions or allergies to medications and is unsure of his previous vaccination history. On examination, the patient is systemically well. There is a 3 cm deep cut across the calf that is heavily soiled and bleeds profusely, but there are no obvious signs of infection. The wound is cleaned thoroughly and dressed, with the plan to close it in 48 hours. Prophylactic antibiotics are started, and it is thought that protection against tetanus should be given. Select the most appropriate prophylaxis to offer?

Give a dose of Td/IPV tetanus vaccine in addition to human tetanus immunoglobulin The wound is contaminated with soil so it is high risk. Human tetanus Ig should be given for immediate protection, and in patients who have received the full five-dose course of tetanus vaccine at the correct times (or is up to date with their schedule), no other doses are needed. This patient is unsure of his status and so needs a booster dose - if it is found he has never had the vaccine, then the full course should be arranged subsequently 61%

A phlebotomist gives herself a needlestick injury whilst taking blood from a patient who is known to be hepatitis B positive. The phlebotomist has just started her job and is in the process of being immunised for hepatitis B but has only had one dose to date. What is the most appropriate action to minimise her risk of contracting hepatitis B from the needle?

Give an accelerated course of the hepatitis B vaccine + hepatitis B immune globulin

A 26-year-old woman who recently returned from her medical elective to Bangladesh presents with a non-productive cough and diarrhoea. On examination, her pulse is 58 beats per minute, an erythematous rash is present on her trunk and back. Abdominal examination reveals a palpable mass in the left hypochondriac region. Which of the following is the most likely diagnosis?

Given the woman's recent return from holiday and the nature of her symptoms, it is highly suggestive of typhoid. The erythematous rash on her trunk and back are known as rose spots which are irregular discrete spots measuring between 2-4mm. Patients often develop splenomegaly and a slowed heart rate.

A 15-year-old boy is diagnosed with glandular fever. What is the most appropriate advice to give regarding playing sports?

Glandular fever: avoid contact sports for 4 weeks

what type of bacteria is neisseria meningitidis

Gram negative diplococci

A 24-year-old sexually active woman presents to the GUM clinic complaining of thin, white, fishy-smelling vaginal discharge. What is the best investigation to diagnose this patient?

Gram stain This is a classical description of bacterial vaginosis where the pH of the vagina has been altered by the predominance or absence of lactobacilli, which are endogenous to the vaginal flora. It is not a sexually transmitted infection; however it can be precipitated by sexual contact. On the gram stain, the presence of clue cells also aids diagnosis Urine Nucleic acid amplification test A good investigation for many of the sexually transmitted infections, however in this case it would not be the best investigation. It is more appropriate in testing for Gonorrhoea or Chlamydia infection 43%

removing tick

Grasp the head and body of the tick as close to the skin as possible and remove it only

A 34-year-old woman who is 35 weeks pregnant presents to her general practitioner with painful blisters affecting the vagina and cervix, along with inguinal lymphadenopathy. She has never had these symptoms before. The GP diagnoses primary genital herpes. Which of the following management strategies is most appropriate?

Guidelines issued by the Royal College of Obstetricians and Gynaecologists state that women who present with first-episode genital herpes during their third trimester should be managed with daily suppressive oral aciclovir 400mg until delivery. Delivery should be by caesarean section due to a high risk of neonatal HSV (herpes simplex virus) transmission.

does anti-HCV indicate a hep C infection

HCV = infection. Need HCV RNA to tell whether it is recent or past If the patient is Anti-HCV positive, this does not indicate that they have a current infection. They will need a positive HCV-RNA to indicate active infection, not Anti-HCV.

A 31-year-old man from Russia who is known to be HIV positive presents with purple plaques on his skin. Which of the following viruses is thought to be the cause of Kaposi's sarcoma?

HHV8 = human herpes virus 8.

HIV seroconversion and diagnosis

HIV seroconversion is symptomatic in 60-80% of patients and typically presents as a glandular fever type illness. Increased symptomatic severity is associated with poorer long term prognosis. It typically occurs 3-12 weeks after infection. features: sore throat lymphadenopathy malaise, myalgia, arthralgia diarrhoea maculopapular rash mouth ulcers rarely meningoencephalitis diagnosis: HIV antibodiesmay not be present in early infection, but most people develop antibodies to HIV at 4-6 weeks but 99% do by 3 monthsusually consists of both a screening ELISA (Enzyme Linked Immuno-Sorbent Assay) test and a confirmatory Western Blot Assay p24 antigena viral core protein that appears early in the blood as the viral RNA levels riseusually positive from about 1 week to 3 - 4 weeks after infection with HIV combination tests (HIV p24 antigen and HIV antibody) are now standard for the diagnosis and screening of HIVif the combined test is positive it should be repeated to confirm the diagnosissome centres may also test the viral load (HIV RNA levels) if HIV is suspected at the same time testing for HIV in asymptomatic patients should be done at 4 weeks after possible exposure after an initial negative result when testing for HIV in an asymptomatic patient, offer a repeat test at 12 weeks

HIV seroconversion and diagnosis

HIV seroconversion is symptomatic in 60-80% of patients and typically presents as a glandular fever type illness. Increased symptomatic severity is associated with poorer long term prognosis. It typically occurs 3-12 weeks after infectionFeatures sore throat lymphadenopathy malaise, myalgia, arthralgia diarrhoea maculopapular rash mouth ulcers rarely meningoencephalitis Diagnosis HIV antibodiesmay not be present in early infection, but most people develop antibodies to HIV at 4-6 weeks but 99% do by 3 monthsusually consists of both a screening ELISA (Enzyme Linked Immuno-Sorbent Assay) test and a confirmatory Western Blot Assay p24 antigena viral core protein that appears early in the blood as the viral RNA levels riseusually positive from about 1 week to 3 - 4 weeks after infection with HIV combination tests (HIV p24 antigen and HIV antibody) are now standard for the diagnosis and screening of HIVif the combined test is positive it should be repeated to confirm the diagnosissome centres may also test the viral load (HIV RNA levels) if HIV is suspected at the same time testing for HIV in asymptomatic patients should be done at 4 weeks after possible exposure after an initial negative result when testing for HIV in an asymptomatic patient, offer a repeat test at 12 weeks

what test is offered to all pts diagnosed with TB

HIV testing

A 34-year-old man is seen in the sexual-health clinic with bloody anal discharge associated with perianal and mucosal ulcers; a diagnosis of lymphogranuloma venereum is made.What is the most important investigation to perform?

HIV testing Although caused by Chlamydia trachomatis, the majority of patients in the UK with lymphogranuloma venereum will be HIV positive. The converse is also true, in that any HIV positive patient presenting with proctitis, as in this vignette, should raise concerns of lymphogranuloma venereum.

A 59-year-old patient with poorly controlled HIV presents to the outpatient clinic with a worsening generalised headache and double vision. The patient is known to take his HIV medication sporadically. He lives alone, drinks 20 units of alcohol per week, and does not smoke cigarettes.On examination, cranial nerve testing demonstrates a 6th nerve palsy but is otherwise unremarkable. Investigations are performed, a CT scan demonstrates multiple ring-enhancing lesions. The thallium SPECT scan is negative.What is the most likely diagnosis in this patient?

HIV, neuro symptoms, multiple brain lesions with ring enhancement - toxoplasmosis This is a difficult question and requires an advanced understanding of neurological complications in HIV. The correct answer is toxoplasmosis. Cerebral toxoplasmosis is an opportunistic infection caused by the parasite Toxoplasma gondii, which is found in the faeces of infected cats, and in infected meat. As discussed in the description above, it can cause headaches and double vision. Investigations may demonstrate a single or multiple ring-enhancing lesions on CT scan and a negative thallium SPECT scan.

A 38-year-old woman presents to the emergency department with poorly-defined symptoms of headache and drowsiness. She has a past medical history of asthma and is known to be HIV-positive.An urgent CT head scan is requested, which demonstrates a single brain lesion with homogenous enhancement. A subsequent thallium SPECT scan is conducted and has a positive result.Given the findings, what is the most likely diagnosis?

HIV, neuro symptoms, single brain lesions with homogenous enhancement - CNS lymphoma Toxoplasmosis is an important differential in HIV-positive patients, accounting for 50% of cerebral lesions in that cohort. It presents with similar symptoms to this patient, but CT would show ring-enhancing lesions rather than homogenous enhancement. Treatment is with sulfadiazine and pyrimethamine.

what vaccine is given at 12/13

HPV

severe hepatitis in a pregnant women. Additional factors include thrombocytopenia, which is a recognised extra-hepatic manifestation of this infection, and the history of recent attendance at a barbecue

Hep E remember HELP is associated with the last trimester and with pre-eclampsia

A 24-year-old medical student presents with a 1 week history of intermittent fevers, nausea, vomiting, diarrhoea, fatigue, and malaise. He seems quite anxious as this morning he noticed dark coloured urine and pale stools. On further questioning he reports that he returned from his medical elective in India 4 weeks ago. He denies any smoking or illicit substance use. He drinks 10-20 units of alcohol most weeks whilst at university. He was previously fit and well. He reports 2 sexual partners in the past 3 years. On abdominal examination, there is palpable hepatomegaly extending approximately 2 cm below the right subcostal margin. There is also notable scleral icterus. What is the most likely cause of his symptoms?

Hepatitis A Hepatitis A infection typically presents with the symptoms described above, and a 2-6 week incubation period following exposure is typical. Poor food hygiene in an endemic country such as India is the likely source of infection in this case. The majority of hepatitis A infections will resolve within 2-3 weeks of symptoms onset. 61% Hepatitis B Symptoms are normally more severe in hepatitis B infection, extra hepatic features including urticaria and arthralgia are common. High risk groups for hepatitis B include: intravenous drug users, sex workers and men who have sex with men. 21% Hepatitis C Hepatitis C infection is most often asymptomatic. It is strongly associated with specific risk factors, including: intravenous or nasal drug use, HIV infection, history of organ transplant or blood transfusion. 13% alcoholic hepatitis is a condition due to chronic alcohol use - this pt is too young to develop it

A 26-year-old male presents with nausea, malaise and jaundice. He returned 3 weeks ago from a holiday to India. On examination he has a moderate hepatosplenomegaly and yellowing of the sclera. He also has dark urine and pale stools.What is the most likely diagnosis?

Hepatitis A is an RNA virus that is spread by the faecal-oral route. It is very common and particularly associated with travellers. Treatment is supportive because the condition is usually self-limiting. Hepatitis A infection has a short incubation period of 15 to 50 days which would also fit with this scenario.Hepatitis E is also spread by the faecal-oral route and should be suspected if the disease is more severe in pregnant women, or if hepatitis A has been excluded. In this patient, this is a less likely answer as hepatitis A infection is much more common.Transmission of hepatitis B and C virus results from exposure to infectious blood or body fluids containing blood. Possible forms of transmission include sexual contact, blood transfusions and transfusion with other human blood products, re-use of contaminated needles and syringes, and vertical transmission from mother to child during childbirth. There are no risk factors for hepatitis B or C infection stated in this question.The hepatitis D virus is considered to be a subviral satellite because it can propagate only in the presence of the hepatitis B virus (HBV).

A 30 year old man presents to his GP with jaundice, which developed 3-4 weeks after returning from a trip to Southeast Asia. During his trip he was involved in a road traffic accident requiring the transfusion of 2 units of blood in a rural hospital. He is otherwise well, complaining only of mild abdominal pain and intermittent diarrhoea. On examination he is afebrile, but icteric with mild abdominal tenderness worse on inspiration. A blood count and liver function test are sent by the GP which show a Hb of 130 g/L, lymphocytosis, transaminitis (ALT 1830), and mixed hyperbilirubinaemia. What is the single most likely diagnosis?

Hepatitis B This is the correct answer. Hepatitis B causes an acute hepatitis in 75% of patients, presenting with jaundice, abdominal pain and high transaminases. He likely contracted this infection at the time of transfusion. It carries at 5-10% risk of developing chronic liver disease 77%

hep B

Hepatitis B is a double-stranded DNA hepadnavirus and is spread through exposure to infected blood or body fluids, including vertical transmission from mother to child. The incubation period is 6-20 weeks. The features of hepatitis B include fever, jaundice and elevated liver transaminases.Complications of hepatitis B infection chronic hepatitis (5-10%). 'Ground-glass' hepatocytes may be seen on light microscopy fulminant liver failure (1%) hepatocellular carcinoma glomerulonephritis polyarteritis nodosa cryoglobulinaemia Immunisation against hepatitis B (please see the Greenbook link for more details) children born in the UK are now vaccinated as part of the routine immunisation schedule. This is given at 2, 3 and 4 months of age at risk groups who should be vaccinated include: healthcare workers, intravenous drug users, sex workers, close family contacts of an individual with hepatitis B, individuals receiving blood transfusions regularly, chronic kidney disease patients who may soon require renal replacement therapy, prisoners, chronic liver disease patients contains HBsAg adsorbed onto aluminium hydroxide adjuvant and is prepared from yeast cells using recombinant DNA technology around 10-15% of adults fail to respond or respond poorly to 3 doses of the vaccine. Risk factors include age over 40 years, obesity, smoking, alcohol excess and immunosuppression testing for anti-HBs is only recommended for those at risk of occupational exposure (i.e. Healthcare workers) and patients with chronic kidney disease. In these patients anti-HBs levels should be checked 1-4 months after primary immunisation the table below shows how to interpret anti-HBs levels: AntiHBs level and their responses: > 100 = indicates adequate response, no further testing required. should still receive booster at 5 years. 10-100 = Suboptimal response - one additional vaccine dose should be given. If immunocompetent no further testing is required <10 = Non-responder. Test for current or past infection. Give further vaccine course (i.e. 3 doses again) with testing following. If still fails to respond then HBIG would be required for protection if exposed to the virus Mx of hep B: pegylated interferon-alpha used to be the only treatment available. It reduces viral replication in up to 30% of chronic carriers. A better response is predicted by being female, < 50 years old, low HBV DNA levels, non-Asian, HIV negative, high degree of inflammation on liver biopsy whilst NICE still advocate the use of pegylated interferon firstl-line other antiviral medications are increasingly used with an aim to suppress viral replication (not in a dissimilar way to treating HIV patients) examples include tenofovir, entecavir and telbivudine (a synthetic thymidine nucleoside analogue)

A 48-year-old man with a history of chronic injection drug use, presents with sequential foot-drop and then wrist drop associated with pain and occurring over a 3-week period. Nerve conduction studies show evidence of a multifocal axonal neuropathy. He is also noted to have nail fold infarcts in his hands and feet, purpura and hepatomegaly. What is the most likely underlying diagnosis?

Hepatitis C related cryoglobulinaemia A sub-acute painful multifocal neuropathy (mononeuritis multiplex) occurring in association with features of systemic inflammation (in this case, seen as nail fold infarcts) raises the possibility of vasculitic neuropathy. Hepatitis C infection may be associated with cryoglobulinaemia (proteins that become insoluble at reduced temperatures), which causes a vasculitic syndrome including neuropathy 27%

A 35-year-old male presents to the emergency department with right upper quadrant pain that has come on over the past 24 hours. This has been accompanied by pruritus and fever. He denies any weight loss.He tells you that he sees a specialist for ongoing treatment of Hep B. He reports being compliant with his medications and says he has never had any symptoms since diagnosis. He denies drinking alcohol but says that he does use recreational drugs.On examination, he appears jaundiced and has needle track marks on his arms.Which of these is the most likely diagnosis?

Hepatitis D superinfection is an differential for chronic hepatitis B patients with acute flare up

how is hep E spread

Hepatitis E is associated with faecal-oral spread, commonly affecting shellfish and pork products. Blood results show elevated bilirubin and significant transaminitis.

post splenectomy sepsis

Hyposplenism may complicate certain medical conditions where splenic atrophy occurs or may be the result of medical intervention such as splenic artery embolization and splenectomy for trauma. Diagnosis of hyposplenism is difficult and whilst there may be peripheral markers of the splenectomised state (e.g. Howell-Jolly bodies) these are neither 100% sensitive or specific. The most sensitive test is a radionucleotide labelled red cell scan.Hyposplenism, by whatever mechanism it occurs dramatically increases the risk of post-splenectomy sepsis, particularly with encapsulated organisms. Since these organisms may be opsonised, but this then goes undetected at an immunological level due to loss of the spleen. For this reason, individuals are recommended to be vaccinated and have antibiotic prophylaxis.Key recommendations All those with hyposplenism or may become so (such as prior to an elective splenectomy) should receive pneumococcal, Haemophilus type b and meningococcal type C vaccines. These should be administered 2 weeks prior to splenectomy or two weeks following splenectomy. The vaccine schedule for meningococcal disease essentially consists of a dose of Men C and Hib at 2 weeks and then a dose of the MenACWY vaccine one month later. Those aged under 2 may require a booster at 2 years. A dose of pneumococcal polyvalent polysaccharide vaccine (PPV) is given at two weeks. A conjugated vaccine (PCV) is offered to young children. The PCV is more immunogenic but covers fewer serotypes. Boosting PPV is either guided by serological measurements (where available) or by routine boosting doses at 5 yearly intervals. Annual influenza vaccination is recommended in all cases Antibiotic prophylaxis is offered to all. The risk of post-splenectomy sepsis is greatest immediately following splenectomy and in those aged less than 16 years or greater than 50 years. Individuals with a poor response to pneumococcal vaccination are another high-risk group. High-risk individuals should be counselled to take penicillin or macrolide prophylaxis. Those at low risk may choose to discontinue therapy. All patients should be advised about taking antibiotics early in the case of intercurrent infections. Asplenic individuals travelling to malaria endemic areas are at high risk and should have both pharmacological and mechanical protection. Dosing Penicillin V 500mg BD or amoxicillin 250mg BD

IGRA

IFGA blood testing assesses the level of responsiveness when exposed to TB antigens. These are the Quanteferon GOLD test and the T.SPOT-TB test. If the patient has been exposed to TB previously, these will be positive. However, this will not indicate if the patient has latent or active TB so further testing would be indicated

A 36-year-old woman presents to a sexual health clinic complaining of a lesion on her vulva that has been present for 2 weeks. She reports no other symptoms and has no medical history with no regular medications and no known allergies.Examination reveals a single ulcer on the left labia majora. The patient denies pain and examination is otherwise unremarkable.She engages in regular, unprotected oral and vaginal intercourse with her husband of 4 years.Given the most likely diagnosis, what is the most appropriate management?

IM benzylpenicillin is the correct answer. This patient has presented with likely primary syphilis, often associated with a single painless ulcer, known as a chancre. These ulcers typically self-resolve within 3-6 weeks but must be treated with antibiotics to prevent the infection from developing. It is important not to rule out sexually transmitted infections only on the basis that a patient has a regular partner. A single dose of IM benzylpenicillin is used first-line in the treatment of primary syphilis.

A 32-year-old man presents to the emergency department with copious watery diarrhoea over the past 36 hours. He returned from visiting relatives in rural India 4 days ago, and has subsequently heard of an outbreak of people presenting with similar symptoms in the village where he was staying. He describes severe abdominal cramping and excessive watery diarrhoea and vomiting. He has tried to drink water, but states he is unable to keep any fluids down. On examination, he appears lethargic, with dry mucosal membranes and turgid skin. Observations are taken which show: HR 104 bpm RR 16 breaths per minute O2 saturation 98% in air BP 102/71 mmHg Temperature 38.5 °C Given the likely diagnosis, what is the most important initial management option?

IV Hartmann's solution Aggressive fluid management is the mainstay of cholera treatment and helps to prevent severe dehydration due to the ongoing profuse fluid loss. If the patient can drink, oral rehydration solution may be sufficient. IV fluid rehydration using Hartmann's solution is indicated in cases of severe dehydration. 72%

A 57 year old male presents to A+E with a week-long history of pain, redness, and swelling of his left knee. He is morbidly obese and has chronic osteoarthritis of both knees. He has no allergies to medications. On examination, the patient is hemodynamically stable, though he is mildly pyrexic and tachycardic. The left knee appears grossly swollen and erythematous and feels warm to touch. Movement at the knee joint is greatly reduced. The joint is aspirated and initial gram staining revealed gram positve cocci in strands. What antibiotic should be started empirically before culture and sensitivity results are obtained?

IV Vancomycin This is a narrow spectrum gram-positive agent, which also covers problem gram-positives such as MRSA and coagulase -ve staphylococcus. The gram stain reveals gram +ve cocci in strands, which are likely streptococci. Once full culture and sensitivity results are obtained this can be narrowed further (with advice from micro) e.g. IV penicillin or clindamycin 30%

Mx necrotising fasciitis

IV abx and immediate surgical debridement

A 64 year old male presents to A+E with a 2-day history of swelling and erythema of his left leg. It has been spreading up gradually from his heel, and he has experienced fever and malaise since its onset. He has a background of Type 2 Diabetes Mellitus, hypertension and obesity. He has no known drug allergies. On examination, the patient is hemodynamically stable, alert and orientated. However, he is pyrexic and his blood pressure is lower than his normal base-line. An area of poorly demarcated, the erythematous swelling has spread 2/3rds of the way up the calf, which feels warm to touch. It is noted that the left heel is cracked and dry from Tinea pedis infection. What medication should be prescribed?

IV benzylpenicillin and flucloxacillin This is the treatment of choice for severe cellulitis 31%

A 37-year-old woman presents to the emergency department with a 2-day history of gradually worsening swelling and pain around her right knee. She denies any trauma but adds that she has previously experienced similar pain in her wrists a week ago, which resolved spontaneously. On examination, she is afebrile, with a normal blood pressure and respiratory rate. Her heart rate measures 101 bpm and a mid-diastolic murmur is heard on auscultation, loudest over the apex. Exposure reveals a ring-like blanching erythematous rash over the back. Blood work-up is relevant for: White cell count 12.4 × 109/L Erythrocyte sedimentation rate (ESR) 60mm/h Antistreptolysin O titre (ASOT) 312 U/mL The infectious diseases team suspects a diagnosis of rheumatic fever. What is the most appropriate step in the management of this patient at this stage?

IV benzylpenicillin as a stat dose This patient has presented with features indicative of acute rheumatic fever. She fulfils three major criteria: migratory polyarthritis, carditis manifesting as a new-onset murmur, and erythema marginatum. Evidence of a raised ASOT is also highly suggestive. Treatment should initially be with a stat dose of benzylpenicillin, followed by a course of oral antibiotics to ensure adequate treatment. 55%

A 31-year-old woman presents to the Emergency Department complaining of a headache. She has had 'flu' like symptoms for the past three days with the headache developing gradually yesterday. The headache is described as being 'all over' and is worse on looking at bright light or when bending her neck. On examination her temperature is 38.2º, pulse 96 / min and blood pressure 116/78 mmHg. There is neck stiffness present but no focal neurological signs. On close inspection you notice a number of petechiae on her torso. She has been cannulated and bloods (including cultures) have been taken. What is the most appropriate next step?

IV cefotaxime The patient has meningococcal meningitis. they need appropriate IV abx immediately. With the advent of modern PCR diagnostic techniques there is no justification for delaying potentially lifesaving treatment by performing a lumbar puncture in patients with suspected meningococcal meningitis.

A 23 year old patient presents to A+E with a 2-day history of pain, swelling, and warmth around her right ankle. She has a background of Sickle Cell Disease, for which she is admitted regularly for treatment of crises and pain. She has no allergies to medications. On examination, the patient is stable but in significant pain. Her temperature is 38 degrees, but her observations are otherwise normal. The right ankle is significantly swollen and movement is limited. The joint is aspirated, and gram -ve rods are found. What antibiotic should be given at this stage?

IV ceftriaxone The gram stain shows gram -ve rods, and in a patient with sickle cell anaemia this is more likely to be Salmonella. Irrespective of this, empirical treatment for gram -ve septic arthritis is a third spectrum cephalosporin such as ceftriaxone. This will cover salmonella, as well as any other gram-ve organisms causing septic arthritis (except pseudomonas) 31%

A 51 year old male presents to the clinic with a 3-week history of pain, swelling and discharging pus from his left heel. He suffered a 3 cm-deep penetrating wound to this area a month ago, when he stepped on a large shard of glass. He has no past medical history and has no allergies to medications. On examination, he is hemodynamically stable, alert and orientated. His left heel has significant soft tissue swelling and the overlying skin is erythematous and hot. There is a sinus tract visible, which is discharging pus. Investigations are ordered, and a referral is made to the surgeons. What antibiotic regimen is it most appropriate to begin empirically?

IV flucloxacillin and rifampicin This is the empirical treatment of choice for osteomyelitis, though fusidic acid may be used instead of rifampicin. Given the subacute onset of symptoms, the previous deep penetrating injury, and the discharging sinus tract, it is most likely that this is chronic osteomyelitis rather than cellulitis

A 53-year-old lady comes in to the Emergency Department with a cough productive of green sputum and palpitations. She feels very unwell, feverish and lethargic. On examination she has bronchial breathing at her right base with respiratory rate 25/min, sats 95% on room air. Her heart sounds are normal with an irregularily irregular heartbeat. Her heart rate was 120/min and blood pressure 90/40 mmHg. An ECG shows atrial fibrillation with a fast ventricular rate. She has no history of atrial fibrillation. What is the first treatment that should be given?

IV fluids This patient is clearly septic from a pneumonia. This has tipped the patient into atrial fibrillation (AF) given there is no previous history of atrial fibrillation. It is important to treat the sepsis as the cause of the AF. To do this it is imperative to give IV fluids and IV antibiotics. If the patient's AF does not settle following the resolution of the sepsis then other options would be considered.The patient is haemodynamically unstable and according to WHO surviving sepsis they require fluid resuscitation and broad-spectrum intravenous antibiotics. You would not give a beta-blocker to a hypotensive patient who is septic as this might lead to a further decompensation in their blood pressure.

A 23-year-old male presents to the emergency department due to trauma to his right hand. He accidentally cut his hand with a knife when he was gardening. There was some bleeding which he controlled by compression with a cloth. He has had five doses of tetanus vaccine in the past, with the last dose at 14 years old. The wound is cleaned thoroughly.What is the next step in management?

If a patient has had 5 doses of tetanus vaccine, with the last dose < 10 years ago, they don't require a booster vaccine nor immunoglobulins, regardless of how severe the wound is

Mary, a 34-year-old woman presents to the emergency department after an accident. She was clearing out her allotment when she accidentally stood on an old nail which penetrated through her boots and into the sole of her right foot. She removed the nail herself before seeking help due to excess bleeding.Mary tells you that she has kept up to date with all her vaccinations from childhood, including at least 5 separate tetanus vaccinations, the last of which being 6 years ago. You decide to initially manage Mary by cleaning her wound thoroughly and prescribing some prophylactic antibiotics.In addition to cleaning the wound and prescribing appropriate antibiotics, how should Mary be managed in regard to her tetanus risk?

If a patient has had 5 doses of tetanus vaccine, with the last dose < 10 years ago, they don't require a booster vaccine nor immunoglobulins, regardless of how severe the wound is

SE of TB medication

Immune reconstitution disease occurs typically 3-6 weeks after starting treatment often presents with enlarging lymph nodes Drug adverse effects rifampicinpotent liver enzyme inducerhepatitis, orange secretionsflu-like symptoms isoniazidperipheral neuropathy: prevent with pyridoxine (Vitamin B6)hepatitis, agranulocytosisliver enzyme inhibitor pyrazinamidehyperuricaemia causing gout, arthralgia, myalgia, hepatitis ethambutol optic neuritis: check visual acuity before and during treatment

A 34-year-old intravenous drug user is brought to the emergency department with back pain, bilateral leg weakness and fever. A spinal epidural abscess is suspected.What imaging investigation is required to confirm the diagnosis?

In suspected spinal epidural abscess, a full spine MRI is required to search for skip lesions

A 48 year old farmer attends the emergency department 7 days after cutting his arm from falling on barbed wire in his field. He complaints of fever, headache and painful spasms in his neck and back which last several minutes.You suspect tetanus and he tells you he has completed a course of tetanus vaccination previously.What is the most appropriate treatment?

In this case there is a high risk wound and symptoms so tetanus immunuglobulin would be advised alongside a muscle relaxant such as diazepam, and ventilatory support if needed. A tetanus booster is not recommended in the UK if the patient is already immunized. Tetanus antitoxin may be used developing countries as it is cheaper but it has a higher rate of anaphylaxis and a shorter half life so is not recommended in the UK.High risk wounds Wounds burns needing surgery delayed more than 6 hours Wounds contaminated with soil Compound fractures Wounds containing foreign bodies Wounds/burns in people with systemic sepsis

A 20-year-old woman presents to the emergency department with a sore throat, and shortness of breath. She appears septic with a temperature of 39.5ºC. She was recently treated for tonsillitis with oral antibiotics but did not finish the full course of antibiotics. She is seen to have exudative tonsilitis on oral examination. cardiovascular examination is grossly normal except for slight warmth and swelling along the right sternocleidomastoid muscle. Chest x-ray shows bilateral infiltrates. CT chest showed likely multiple septic emboli and later a CT neck with contrast showed a thrombus in the right internal jugular vein.What is the most likely diagnosis?

Lemierre's syndrome presents with thrombophlebitis of the internal jugular vein following an anaerobic oropharngeal infection. Lemierre's syndrome is typically caused by a primary infection in the oropharyngeal region. Infections, if left untreated can spread to the carotid sheath which contains the internal jugular vein (IJV). The spread of infection to the IJV can cause a thrombus to be formed and then septic emboli occur which typically goes to the lungs, liver, and other areas in the body. Therefore, Lemierre's syndrome best matches the description above and is the correct answer.

infectious mononucleosis

Infectious mononucleosis (glandular fever) is caused by the Epstein-Barr virus (EBV, also known as human herpesvirus 4, HHV-4) in 90% of cases. Less frequent causes include cytomegalovirus and HHV-6. It is most common in adolescents and young adults. The classic triad of sore throat, pyrexia and lymphadenopathy is seen in around 98% of patients: -sore throat -lymphadenopathy: may be present in the anterior and posterior triangles of the neck, in contrast to tonsillitis which typically only results in the upper anterior cervical chain being enlarged. - pyrexia other features include: malaise, anorexia, headache palatal petechiae splenomegaly - occurs in around 50% of patients and may rarely predispose to splenic rupture hepatitis, transient rise in ALT lymphocytosis: presence of 50% lymphocytes with at least 10% atypical lymphocytes haemolytic anaemia secondary to cold agglutins (IgM) a maculopapular, pruritic rash develops in around 99% of patients who take ampicillin/amoxicillin whilst they have infectious mononucleosis sx typically resolve after 2-4 weeks. dx - heterophil antibody test (monospot test) - NICE guidelines suggest FBC and monospot in the 2nd week of the illness to confirm a diagnosis of glandular fever. Mx is supportive and includes: - rest during the early stages, drink plenty of fluid, avoid alcohol. - simple analgesia for any aches or pains - consensus guidance in the UK is to avoid playing contact sports for 4 weeks after having glandular fever to reduce the risk of splenic rupture. There is an interesting correlation between EBV and socioeconomic groups. Lower socioeconomic groups have high rates of EBV seropositivity, having frequently acquired EBV in early childhood when the primary infection is often subclinical. However, higher socioeconomic groups show a higher incidence of infectious mononucleosis, as acquiring EBV in adolescence or early adulthood results in symptomatic disease.

A 24-year-old woman is reviewed in the genitourinary medicine clinic. She presented with vaginal discharge and dysuria. Microscopy of an endocervical swab showed a Gram-negative coccus that was later identified as Neisseria gonorrhoea. This is her third episode of gonorrhoea in the past two years. What is the most likely complication from repeated infection?

Infertility secondary to pelvic inflammatory disease (PID) is the most common complication of gonorrhoea. It is the second most common cause of PID after Chlamydia. Arthropathy may occur but it is far less common.

A 22 year old Pakistani man has been contacted by his GP as his mother has confirmed pulmonary Tuberculosis. He has no symptoms and reports feeling well. He was born in Pakistan and was vaccinated against Tuberculosis as a child, then moved to the UK 15 years ago. What is the next best step in this patients management?

Interferon Gamma Release Assay (IGRA) The Interferon gamma release assay is used to identify patients who may have latent TB infection. It can detect latent TB in patients who have already been vaccinated with a BCG (as opposed to a Tuberculin Skin test which can be less accurate in patients who have previously been vaccinated). In practice, both Interferon gamma release assays and the Tuberculin skin test are used in screening, regardless of immunisation status. 48%

An 82 year old man presents to A&E with a painful rash on his face . He is currently receiving chemotherapy for bowel cancer. On examination the rash is on the left side of the face which is peri-orbital and extends up to the forehead and to the tip of the nose. It has multiple small vesicles and some pustules. What is the most appropriate treatment for this patient?

Intravenous Aciclovir This is a classical description of a herpes zoster virus reactivation otherwise known as shingles. The patient is immunocompromised due to being on chemotherapy and is at the right age when this commonly occurs. The rash has a dermatomal description which is classical and the rash is very painful. This infection is severe as this patient is displaying nose tip vesicles which is known as Hutchinson's sign. This means the virus is three times more likely to cause ocular complications and must be taken very seriously. Intravenous antivirals are warranted Oral Aciclovir A good treatment but due to the ocular involvement and Hutchinson's sign in this patient, intravenous Aciclovir is warranted 35%

risk factors for invasive aspergillosis

Invasive aspergillosis is a systemic Aspergillus infection (Aspergillus fumigatus, Aspergillus flavus, and Aspergillus terreus) that is a leading cause of death in immunocompromised patients. Risk factors include: HIV Leukaemia Following broad-spectrum antibiotics

A 16-year-old, who has recently immigrated from South Asia presents to her GP with pales blotches on her arm.On examination, there are a few large hypopigmented lesions on her left arm. There is loss of sensation to fine touch over the lesion. She is systemically well.Which of the following is compatible with the above findings?

Leprosy leads to skin hypopigmentation

mx of meningitis

Investigations suggested by NICE full blood count CRP coagulation screen blood culture whole-blood PCR blood glucose blood gas Lumbar puncture if no signs of raised intracranial pressure All patients should be transferred to hospital urgently. If patients are in a pre-hospital setting (for example a GP surgery) and meningococcal disease is suspected then intramuscular benzylpenicillin may be given, as long as this doesn't delay transit to hospital. BNF recommendations on antibiotics: initial empirical therapy aged < 3 months = IV cefotaxime + amoxicillin (or ampicillin). Initial empirical therapy aged 3 months - 50 years = IV cefotaxime (or ceftriaxone) Initial empirical therapy aged > 50 years = Intravenous cefotaxime (or ceftriaxone) + amoxicillin (or ampicillin) Meningococcal meningitis = IV benzylpenicillin or cefotaxime (or ceftriaxone) pneumococcal meningitis = IV cefotaxime (or ceftriaxone). meningitis caused by h.influenza = IV cefotaxime (or ceftriaxone). Meningitis caused by listeria = IV amoxicillin (or ampicillin) + gentamicin IV dexamethasone should be given to reduce the risk of neurological sequelae, but the BNF advise to withhold if: - septic shock - meningococcal septicaemia - immunocompromsied - meningitis following surgery If the patient has a history of immediate hypersensitivity reaction to penicillin or to cephalosporins the BNF recommends using chloramphenicol. Management of contacts prophylaxis needs to be offered to household and close contacts of patients affected with meningococcal meningitis. Prophylaxis should also be offered to people who been exposed to respiratory secretion, regardless of the closeness of contact people who have been exposed to a patient with confirmed bacterial meningitis should be given prophylactic antibiotics if they have close contact within the 7 days before onset oral ciprofloxacin or rifampicin or may be used. The Health Protection Agency (HPA) guidelines now state that whilst either may be used ciprofloxacin is the drug of choice as it is widely available and only requires one dose the risk is highest in the first 7 days but persists for at least 4 weeks meningococcal vaccination should be offered to close contacts when serotype results are available, including booster doses to those who had the vaccine in infancy for pneumococcal meningitis, no prophylaxis is generally needed. There are however exceptions to this. If a cluster of cases of pneumococcal meningitis occur the HPA have a protocol for offering close contacts antibiotic prophylaxis. Please see the link for more details

The Bangladeshi parents of a 12-year-old boy come for advice. They have recently emigrated to the UK and have been advised that he should have the BCG vaccine for tuberculosis by immigration officials. He is well and asymptomatic. What is the most appropriate next step?

It is important that a tuberculin skin test is performed (to exclude past exposure to tuberculosis) prior to giving then the bcg BCG is a live vaccine and could potentially reactivate latent TB.

44-year-old farmer presents with headache, fever and muscle aches. He initially thought he had a bad cold but his symptoms have got progressively worse over the past week. During the review of systems he reports nausea and a decreased urine output. On examination his temperature is 38.2ºC, pulse 102 / min and his chest is clear. Subconjunctival haemorrhages are noted but there is no evidence of jaundice. What is the most likely diagnosis?

Leptospirosis The main clue in the question is the patients occupation. Mycoplasma and Legionella are less likely due to the absence of chest symptoms and signs. Liver failure is seen in only 10% of patients with leptospirosis..

A seven-year-old girl, who recently emigrated from sub-Saharan Africa with her parents, has presented to the Paediatric Assessment Unit after a referral from her GP. She is known to be HIV positive.She originally presented with a persistent cough. On examination, she has cervical and axillary lymphadenopathy and you note tonsillar enlargement when looking in the mouth. There were no other salient findings.What would be the most likely malignant cause for her presentation?

Kaposi's sarcoma is one of the most common paediatric cancers in sub-Saharan Africa. Unlike adults, it's much rarer to find the characteristic skin lesions in a paediatric population, with the most common symptom being lymphadenopathy.

A 23 year old man presents to A&E with diarrhoea, a cough and wheeze. On further questioning he returned from Sub-Saharan Africa around seven weeks ago where he frequently swam in fresh water. On examination he has widespread urticaria and hepatosplenomegaly. He has no past medical history or relevant family history. He has some blood tests which show he has an eosinophilia. What is the most likely diagnosis?

Katayama syndrome This is the acute illness associated with Schistosomiasis infection. It occurs two to eight weeks after the initial exposure. It is caused by the migration of the Schistosomules throughout the body and can lead to this constellation of symptoms 45%

flu-like symptoms and a dry cough, relative bradycardia and confusion. Blood tests may show hyponatraemia and recent visit to spain

Legionella pneumonia often occurs in outbreaks centred around a communal water supply. This is alluded to in this question by the recent conference in Spain. It also frequently leads to bi-basal pneumonia as demonstrated in the question.

legionella

Legionnaire's disease is caused by the intracellular bacterium Legionella pneumophilia. It typically colonizes water tanks and hence questions may hint at air-conditioning systems or foreign holidays. Person-to-person transmission is not seenFeatures flu-like symptoms including fever (present in > 95% of patients) dry cough relative bradycardia confusion lymphopaenia hyponatraemia deranged liver function tests pleural effusion: seen in around 30% of patients Diagnosis urinary antigen Management treat with erythromycin/clarithromycin

A 25-year-old man returns from a gap-year in Central and South America and presents with a 2 month history of an ulcerating lesion on his lower lip. Examination of his nasal and oral mucosae reveals widespread involvement. What is the likely diagnosis?

Leishmaniasis (very rare): solitary or multiple reddish macules/papules around sandfly bites; mucosal version can develop a period of time after cutaneous

measles ix

Measles RNA detection by PCR This may be used to confirm measles infection in swabs taken between 24 and 72 hours following the onset of the rash; however, this child's rash started 4 days ago, therefore serology is more appropriate here. Measles-specific IgM and IgG serology This is the most sensitive laboratory test in suspected measles when the onset of the rash occurred over 3 days ago.

You are a junior doctor working in emergency medicine. You review a 20-year-old medical student who has been brought in by his concerned flatmates. He recently returned from southeast Asia where he spent several weeks volunteering following a major flood. He received all vaccinations recommended to him prior to travel.He has a one-week history of fever and flu-like symptoms. On examination, there is hepatomegaly, red conjunctiva and he is jaundiced. Bloods reveal an acute kidney injury and deranged liver function tests.What is the most likely diagnosis?

Leptospirosis is commonly preceded by flu-like symptoms, associated with subconjunctival suffusion and is a cause of hepatorenal failure. Leptospirosis is alluded to by the history of time in a flood-affected area.

You are called to see a 57 year old female patient in the oncology ward, with an acute onset fever. Her background is notable for having advanced breast cancer, for which she is currently receiving cycles of chemotherapy. She has no known drug allergies. On examination, she is hemodynamically stable, though she is warm peripherally and looks unwell. The entry site of her PICC line is erythematous and oozing green material. Blood cultures are taken and she is started on empirical treatment with Tazosin. The blood culture results later reveal an Enterococcus faecalis bacteremia, with resistance to Vancomycin, teicoplanin and Amoxicillin. Given this result, which narrower-spectrum antibiotic should Tazocin be switched to?

Linezolid This is the only agent listed that is used for treatment of Vancomycin-Resistant Enterococci (VRE). Teicoplanin could be used first-line in VRE isolates that are sensitive to it, but here there is Teicoplanin resistance 36%

A 73-year-old woman is admitted to hospital. After being treated with a broad-spectrum antibiotic for sepsis secondary to a lower respiratory tract infection. After a period of improvement, the patient deteriorates and is discovered to have an MRSA bacteraemia. The patient is documented as having previously had an allergic reaction to vancomycin. With which antibiotic would it be appropriate to treat the patient?

Linezolid is the second line antibiotic to treat MRSA

A 73-year-old woman is admitted to hospital. After being treated with a broad-spectrum antibiotic for sepsis secondary to a lower respiratory tract infection. After a period of improvement, the patient deteriorates and is discovered to have an MRSA bacteraemia. The patient is documented as having previously had an allergic reaction to vancomycin. With which antibiotic would it be appropriate to treat the patient?

Linezolid is the second line choice antibiotic for MRSA, after vancomycin or teicoplanin

A 30-year-old HIV positive man attends your travel clinic asking for your advice on holiday vaccinations. His is taking anti-retroviral therapy and his most recent CD4 count is 200 cells/mm³. He is otherwise well and has no other medical conditions.Which of the following vaccines are contraindicated in this man?

Live attenuated vaccines such as BCG are contraindicated in all HIV positive patients. Other live attenuated vaccines which should not be given in immunocompromised patients are: Yellow fever Oral polio Intranasal influenza Varicella Measles, mumps and rubella (MMR)

A 22-year-old male presents to the GUM clinic as he has received a notification that his ex-partner has had a positive test for gonorrhoea. On questioning, he reports discharge from the tip of his penis and some discomfort on urinating. He is concerned about the implications of this diagnosis.What is a potential local complication of gonorrhoea to inform him of in this case?

Local complications that may develop include urethral strictures, epididymitis and salpingitis (hence may lead to infertility). Disseminated infection may occur: Disseminated gonococcal infection (DGI) and gonococcal arthritis may also occur, with gonococcal infection being the most common cause of septic arthritis in young adults. The pathophysiology of DGI is not fully understood but is thought to be due to haematogenous spread from mucosal infection (e.g. Asymptomatic genital infection). Initially there may be a classic triad of symptoms: tenosynovitis, migratory polyarthritis and dermatitis. Later complications include septic arthritis, endocarditis and perihepatitis (Fitz-Hugh-Curtis syndrome)Key features of disseminated gonococcal infection tenosynovitis migratory polyarthritis dermatitis (lesions can be maculopapular or vesicular)

A 23-year-old female presents to her general practitioner complaining of a two-day history of rash at the back of her knee. She has a past medical history of eczema and asthma and is on regular Symbicort and emollients. She is a non-smoker and non-drinker and has recently returned from a camping holiday where she had been doing some hiking through a forested area.On examination, there is a bulls-eye lesion just above her right popliteal fossa. She is otherwise systemically well and apyrexial.What is the most appropriate initial management?

Lyme disease can be diagnosed clinically if erythema migrans is present → give antibiotics - oral doxycycline for 21 days. second line options include amoxicillin and azithromycin.

lyme disease

Lyme disease is caused by the spirochaete Borrelia burgdorferi and is spread by ticks.Early features (within 30 days) erythema migrans'bulls-eye' rash is typically at the site of the tick bitetypically develops 1-4 weeks after the initial bite but may present soonerusually painless, more than 5 cm in diameter and slowlly increases in sizepresent in around 80% of patients. systemic featuresheadachelethargyfeverarthralgia Later features (after 30 days) cardiovascularheart blockperi/myocarditis neurologicalfacial nerve palsyradicular painmeningitis Investigation NICE recommend that Lyme disease can be diagnosed clinically if erythema migrans is presenterythema migrans is therefore an indication to start antibiotics enzyme-linked immunosorbent assay (ELISA) antibodies to Borrelia burgdorferi are the first-line testif negative and Lyme disease is still suspected in people tested within 4 weeks from symptom onset, repeat the ELISA 4-6 weeks after the first ELISA test. If still suspected in people who have had symptoms for 12 weeks or more then an immunoblot test should be doneif positive or equivocal then an immunoblot test for Lyme disease should be done Management of asymptomatic tick bites tick bites can be a relatively common presentation to GP practices, and can cause significant anxiety if the tick is still present, the best way to remove it is using fine-tipped tweezers, grasping the tick as close to the skin as possible and pulling upwards firmly. The area should be washed following. NICE guidance does not recommend routine antibiotic treatment to patients who've suffered a tick bite Management of suspected/confirmed Lyme disease doxycycline if early disease. Amoxicillin is an alternative if doxycycline is contraindicated (e.g. pregnancy)people with erythema migrans should be commenced on antibiotic without the need for further tests ceftriaxone if disseminated disease Jarisch-Herxheimer reaction is sometimes seen after initiating therapy: fever, rash, tachycardia after first dose of antibiotic (more commonly seen in syphilis, another spirochaetal disease)

A 45 year old man with type 2 diabetes undergoes spinal surgery for a herniated disc. He is discharged after two days. Two weeks later, the patient presents to his GP with pain in his back that is worsened by activity and often worse at night. On examination, there is tenderness over the L4 vertebra and the symptoms are reproduced by percussion at this level. There is limited spinal flexion due to pain. Neurological examination is normal and his observations are stable. Which is the next best investigation to confirm the diagnosis?

MRI scan This is the correct answer. For this patient, the correct diagnosis is acute osteomyelitis. There are two risk factors which predispose him, which are his diabetes and the prior spinal surgery. The presentation is a typical one, with pain worsened by activity and at night with no neurological symptoms. Local tenderness to gentle spinal percussion is a useful clinical sign for osteomyelitis. The best imaging tool for diagnosing osteomyelitis is MRI, as it can detect early signs of infection which might not be seen on X-ray or CT 73%

A 32-year-old woman presents to the GP complaining of a 4-day history of fevers, nausea, dysuria, increased urinary frequency, and abdominal pain. She denies haematuria or discharge.On examination, the patient has tenderness in the right flank. She explains that the pain started in the suprapubic region but has now spread and has worsened over the past 4 days. The pain is present all the time and ibuprofen has not helped. She has a temperature of 37.9ºC but observations are otherwise normal. There is no significant past medical or drug history.What is the most appropriate next step in this patient's management?

MSU then oral abx - not an uncomplicated case so does not IV abx

mx of legionella

Macrolides such as clarithromycin are used to treat Legionella

A 33-year-old woman returns from a holiday. On her last day abroad she ate lunch at an unlicensed street food vendor. Eight days after her arrival home she experiences bloating, abdominal pain, and non-bloody diarrhoea and she has noticed her stools are floating in the toilet bowl. The patient's symptoms continue for 9 weeks.What is the most likely causative pathogen given the patient's symptoms

Malabsorption can be caused by chronic Giardia infection

A 39-year-old man returns from a two week business trip to Kenya. Four weeks after his return he presents to his GP complaining of malaise, headaches and night sweats. On examination there is a symmetrical erythematous macular rash over his trunk and limbs associated with cervical and inguinal lymphadenopathy. What is the most likely diagnosis?

Man returns from trip abroad with maculopapular rash and flu-like illness - think HIV seroconversion HIV life cycleIncubation = around 3 weeks silentAcute phase = seroconversion = 3-6 weeks post infection. mono-like illness, night sweats and lymphadenopathy very charitristic!Asymptomatic carrier phase - some are fast responder some are slow responder but in general mean of 8-10 years of silenceearly symptomatic phase = usually cd4 of 500-200 generelized painful lymphadenopathyAid = under200. PCP mos common aids defining infection

A 39-year-old man returns from a two week business trip to Kenya. Four weeks after his return he presents to his GP complaining of malaise, headaches and night sweats. On examination there is a symmetrical erythematous macular rash over his trunk and limbs associated with cervical and inguinal lymphadenopathy. What is the most likely diagnosis?

Man returns from trip abroad with maculopapular rash and flu-like illness - think HIV seroconversion Stereotypes are alive and well in medical exams. For questions involving businessmen always consider sexually transmitted infections. The HIV prevalence rate in Kenya is currently around 8%. Discuss (7)Improve

A 68-year-old woman was undergoing inpatient treatment for community-acquired pneumonia when she began to deteriorate. Blood cultures were taken and sent for microscopy, culture and sensitivities. The laboratory report returns and describes a positive sample culture, with growth of an extended spectrum beta-lactamase (ESBL)-producing Escherichia coli strain 536. Of the following options, which would be the most appropriate treatment for this woman's bacteraemia?

Meropenem The sensitivity report shows the presence of an ESBL-producing E. coli. A carbapenem antibiotic, such as meropenem, is warranted in treating this woman's bacteraemia as both penicillin and cephalosporin antibiotics will be ineffective. 51%

MRSA

Methicillin-resistant Staphylococcus aureus (MRSA) was one of the first organisms which highlighted the dangers of hospital-acquired infections.Who should be screened for MRSA? all patients awaiting elective admissions (exceptions include day patients having terminations of pregnancy and ophthalmic surgery. Patients admitted to mental health trusts are also excluded) from 2011 all emergency admissions will be screened How should a patient be screened for MRSA? nasal swab and skin lesions or wounds the swab should be wiped around the inside rim of a patient's nose for 5 seconds the microbiology form must be labelled 'MRSA screen' Suppression of MRSA from a carrier once identified nose: mupirocin 2% in white soft paraffin, tds for 5 days skin: chlorhexidine gluconate, od for 5 days. Apply all over but particularly to the axilla, groin and perineum The following antibiotics are commonly used in the treatment of MRSA infections: vancomycin teicoplanin linezolid Some strains may be sensitive to the antibiotics listed below but they should not generally be used alone because resistance may develop: rifampicin macrolides tetracyclines aminoglycosides clindamycin Relatively new antibiotics such as linezolid, quinupristin/dalfopristin combinations and tigecycline have activity against MRSA but should be reserved for resistant cases

A 26-year-old man is being investigated for a 5-day history of dysuria and urinary frequency at the GP. He states that he also started to notice urethral discharge yesterday.He has no significant past medical history and is not currently sexually active. He has a family history of renal cell carcinoma.On examination, he is abdomen is soft and non-tender, and there is no evidence of organomegaly.A urine dipstick shows ++nitrites, and ++ leucocytes and is negative for erythrocytes. His urine is cloudy in appearance.What is the most appropriate management for this patient?

Mid-stream urine culture is the correct answer. This patient has features of a urinary tract infection (dysuria and urinary frequency). Although a sexually transmitted infection is a reasonable differential, this man has stated that he is not sexually active, making this diagnosis less likely than a urinary tract infection. All men should have a mid-stream urine culture sent before starting antibiotics because they have a longer urinary tract at risk of atypical and resistant bacterial colonisation.

A 42-year-old patient is referred to an infective disease unit due to an erythematous lesion on her ankle and some periorbital oedema which she noticed weeks after returning from a trip to South America. She is diagnosed with Chagas' disease. She is incredibly worried about her prognosis and wants to begin treatment immediately.Which severe complication is it most important to warn her about?

Most of the complications in Chagas' (American trypanosomiasis) are related to muscle changes caused by the parasites in the heart and the GI tract. Consequently, heart failure, megaoesophagus and megacolon are serious complications. Cardiomyopathy is the most common and worrying complication. It is therefore important to get an echocardiogram as soon as possible.

A 41-year-old female presents with 3 day history of a dry cough and shortness of breath. This was preceded by flu-like symptoms. On examination there is a symmetrical, erythematous rash with 'target' lesions over the whole body. What is the likely organism causing the symptoms?

Mycoplasma is associated with erythema multiforme

A 67 year old female presents to the Emergency Department with a 2 week history of fever, malaise and progressive shortness of breath. Observations show a temperature of 38.3 degrees Celsius, respiratory rate of 30, oxygen saturations of 89% on room air, heart rate of 75 beats per minute, and blood pressure of 105/70. On examination, there is dullness to percussion and coarse crepitations heard over the left lower lobe. She has a painful purplish discolouration of her fingers and toes, which she says are exacerbated in the cold weather. Which is the most likely causative organism?

Mycoplasma pneumoniae This has an insidious onset and an incubation period of 2-3 weeks after exposure. The painful purplish discolouration of the extremities are suggestive of cold autoimmune haemolytic anaemia (AIHA). Intravascular haemolysis occurs in the cold, leading to acrocyanosis or Raynaud's phenomenon. This is classically associated with Mycoplasma infection 48%

Mycoplasma pneumoniae

Mycoplasma pneumoniae is a cause of atypical pneumonia which often affects younger patients. It is associated with a number of characteristic complications such as erythema multiforme and cold autoimmune haemolytic anaemia. Epidemics of Mycoplasma pneumoniae classically occur every 4 years. It is important to recognise atypical pneumonia as it may not respond to penicillins or cephalosporins due to it lacking a peptidoglycan cell wall. Features the disease typically has a prolonged and gradual onset flu-like symptoms classically precede a dry cough bilateral consolidation on x-ray complications may occur as below Complications cold agglutins (IgM): may cause an haemolytic anaemia, thrombocytopenia erythema multiforme, erythema nodosum meningoencephalitis, Guillain-Barre syndrome and other immune-mediated neurological diseases bullous myringitis: painful vesicles on the tympanic membrane pericarditis/myocarditis gastrointestinal: hepatitis, pancreatitis renal: acute glomerulonephritis ix = diagnosis is generally by Mycoplasma serology positive cold agglutination test Mx = doxycycline or a macrolide (e.g. erythromycin/clarthromycin)

Mycoplasma pneumoniae

Mycoplasma pneumoniae is a cause of atypical pneumonia which often affects younger patients. It is associated with a number of characteristic complications such as erythema multiforme and cold autoimmune haemolytic anaemia. Epidemics of Mycoplasma pneumoniae classically occur every 4 years. It is important to recognise atypical pneumonia as it may not respond to penicillins or cephalosporins due to it lacking a peptidoglycan cell wall.Features the disease typically has a prolonged and gradual onset flu-like symptoms classically precede a dry cough bilateral consolidation on x-ray complications may occur as below Complications cold agglutins (IgM): may cause an haemolytic anaemia, thrombocytopenia erythema multiforme, erythema nodosum meningoencephalitis, Guillain-Barre syndrome and other immune-mediated neurological diseases bullous myringitis: painful vesicles on the tympanic membrane pericarditis/myocarditis gastrointestinal: hepatitis, pancreatitis renal: acute glomerulonephritis Investigations diagnosis is generally by Mycoplasma serology positive cold agglutination test Management doxycycline or a macrolide (e.g. erythromycin/clarithromycin)

A 19-year-old male university student presents to Accident and Emergency with shortness of breath and significant fatigue. He reports a three-week history of fevers and worsening dry cough. On examination, there are inspiratory crepitations in the right lung. The patient also mentions that they have pain in their toes. A blood test and chest X-ray are requested. Bloods: WCC: 11.4x10^9/L (4-11) CRP: 102mg/L (<5) Hb: 93g/L (140-180) Chest radiograph: Patchy consolidation in the right lower lobe. What is the most likely causative organism?

Mycoplasma pnuemoniae This is the correct answer. This patient has clinical features consistent with mycoplasma pneumonia, including a worsening dry cough and patchy lower lobe consolidation. Mycoplasma infections tend to occur in epidemics and are seen in settings such as hospitals and universities. The pain in the toes and the low haemoglobin a are secondary to a cold autoimmune haemolytic anaemia (a recognised complication of mycoplasma infection) 49%

ix for genital herpes

NAAT

A patient who is awaiting an inguinal hernia repair is found to be positive for MRSA after screening at the pre-admission clinic. What treatment should he be offered, if any?

Nasal mupirocin + chlorhexidine for the skin

A 60 year old female is admitted to the day surgery unit for an elective cholecystectomy. Routine skin swabs were done pre-operatively and came back positive for methicillin-resistant Staphylococcus aureus (MRSA). She is otherwise well and physical examination is normal. Which treatment should be initiated?

Nasal mupirocin and chlorhexidine wash This is the standard decolonisation therapy for MRSA. MRSA colonisation on the skin does not indicate infection and thus antibiotics are not necessary Vancomycin This is first line treatment for MRSA infections, as it is effective against problem Gram positive organisms including MRSA and coagulase negative Staphylococcus. However, this patient is not likely to have an infection and does not require antibiotics. Inappropriate use of antibiotics should be avoided due to emergence of resistant strains

A 45-year-old man presents to the Emergency Department due to severe pain in the perineal area over the past 6 hours. On examination the skin is cellulitic, extremely tender and haemorrhagic bullae are seen. What is the most appropriate management?

Necrotising fasciitis is a medical emergency that is difficult to recognise in the early stages.It can be classified according to the causative organism: type 1 is caused by mixed anaerobes and aerobes (often occurs post-surgery in diabetics). This is the most common type type 2 is caused by Streptococcus pyogenes Risk factors skin factors: recent trauma, burns or soft tissue infections diabetes mellitusthe most common preexisting medical conditionparticularly if the patient is treated with SGLT-2 inhibitors intravenous drug use immunosuppression The most commonly affected site is the perineum (Fournier's gangrene).Features acute onset pain, swelling, erythema at the affected siteoften presents as rapidly worsening cellulitis with pain out of keeping with physical featuresextremely tender over infected tissue with hypoaesthesia to light touchskin necrosis and crepitus/gas gangrene are late signs fever and tachycardia may be absent or occur late in the presentation Management urgent surgical referral debridement intravenous antibiotics Prognosis average mortality of 20%

A 22 year old man presented to the emergency department of his local hospital complaining of progressively worsening bilateral headache over 2 days. He tells you that he does not normally suffer from headaches and is otherwise fit and well. On examination, he is febrile at 38.4, but there is no associated focal neurological features, neck stiffness or rash, and he denies photophobia. A CT head is performed which shows some subtle sulcal effacement, and a lumbar puncture is performed. Frankly purulent fluid is exuded with a raised opening pressure, and the sample is sent to the laboratory for further tests. What is the most likely underlying cause of this patient's symptoms?

Neisseria meningitidis This is the correct answer. Despite our best efforts with vaccination of at risk populations, Neisseria meningitidis remains the most common cause of acute bacterial meningitis in young adults the UK. Subtle sulcal effacement may be seen on CT as the subarachnoid space fills with frank pus, and all patients with headache, fever and a frankly purulent CSF must be assumed to have bacterial meningitis until otherwise proven. Classical features, including photophobia and neck stiffness are variably present 50%

A 29-year-old man presents to the emergency department with acute confusion and is violent towards staff. While being restrained by security, a junior doctor is bitten by the patient on the right arm. The man is known to have HIV and uses intravenous drugs.On examination of the right arm, there are multiple small puncture wounds with some bleeding.What is the most appropriate next step in regard to post-exposure prophylaxis (PEP)?

No post-exposure prophylaxis for HIV is required is correct. Human bites are generally regarded as 'low-risk' incidents in regard to HIV transmission, therefore, PEP for HIV is not immediately required. Since this patient has a history of HIV and intravenous drug use, a risk assessment may be done, which may override this decision, however, at this point in time, PEP is not needed.

A 56-year-old man presents to the ED complaining of a 3-day history of persistent fever and a productive cough. He has been experiencing the symptoms for three days and they do not seem to improve. He was previously fit and well.On examination, he looks distressed and he has a productive cough with yellow sputum. His heart rate is 92/min, respiratory rate 22/min, blood pressure 130/95 mmHg and temperature 39.2 ºC. On auscultation, bibasal crackles are audible and a crusty lesion can be observed on his upper lip.He has a past medical history of asthma.What is the most likely causative agent of his presentation?

Pneumonia caused by Streptococcus pneumoniae is associated with cold sores

A 30-year-old man comes to surgery. He has been handed a slip from an ex-girlfriend stating she has tested positive for Chlamydia. He last slept with her 2 months ago. He has no symptoms of note, in particular no dysuria or discharge. What is the most appropriate management?

Offer Chlamydia testing and antibiotic treatment immediately without waiting for the results

A 24-year-old man presents to his GP with a 10-day history of foul-smelling diarrhoea, abdominal pain and feeling exhausted.He has asthma and irritable bowel syndrome and takes beclometasone regularly and salbutamol, mebeverine and hyoscine butylbromide as required. He is allergic to penicillin. He is a non-smoker, drinks a few pints of beer a week and works as a journalist in Oman, but returned 3 weeks ago for a family event.On examination, his abdomen is distended but there is no rigidity or guarding. Bowel sounds are present.What is the most likely cause of this patient's presentation?

Ongoing diarrhoea, lethargy, bloating, flatulence, steatorrhoea, weight loss +/- recent travel → ?giardiasis The history of recent travel combined with the symptoms of ongoing foul-smelling or greasy diarrhoea (due to malabsorption of fats), lethargy and bloating points to giardiasis. This is a protozoan parasite transmitted via the faecal-oral route. It has an incubation period of 3-25 days and symptoms often persist for weeks. The diagnosis can be confirmed via stool microscopy, direct fluorescence antibody test or polymerase chain reaction. Metronidazole is the first-line treatment.

post strep reactive arthritis

Post-streptococcal reactive arthritis This man has presented with a complex 8-week history of intermittent multifocal pain presenting variably in both lower limbs. Investigations reveal nonspecific inflammation with no obvious bone or soft-tissue abnormality. The history of recurrent tonsillitis, along with a recent similar infection with a raised antistreptolysin O titre, is most consistent with post-streptococcal reactive arthritis. This is a difficult diagnosis to establish in practice and, in most cases, it is self-limiting. Management is conservative, although follow-up for specialist treatment should be considered if symptoms do not resolve in 6 months. 77%

A 78 year old man presents to A&E in January. He complains of a 24 hour history of a fever, dry cough, sore throat, coryza, headache and myalgia. He has a past medical history of heart failure secondary to a myocardial infarction two years ago. He has a chest x ray which is clear and his inflammatory markers are mildly raised. You are concerned that he might have influenza. What is the best treatment initially to treat this emerging infection?

Oral Oseltamivir This is a description of an influenza infection. The key points to consider here are that the patient is presenting in January, which is within flu season in the UK, he has many of the cardinal features of a flu infection and his chest x ray does not demonstrate a bacterial pneumonia. It has been shown that patients with severe co-morbidities such as heart failure should receive antivirals to reduce the effects of a severe infection and further complications. The main types used are neuraminidase inhibitors 36%

A 31-year-old woman is diagnosed with a primary herpes infection at 35+2 weeks gestation. She does not have any other medical problems. She has had a normal pregnancy up to this point. A Caesarean section is planned for delivery at 39 weeks. What treatment should be initiated?

Oral aciclovir 400 mg TDS (three times daily) until delivery is recommended in the RCOG guidelines for women who present with a primary herpes infection in their third trimester of pregnancy, especially if the woman is expected to deliver within 6 weeks.

A 30 year old female presents to her GP with a 3-week history of malaise, fevers and joint pain. She has no medical history and has no allergies to medications. On examination, she is hemodynamically stable and cardiovascular, respiratory and abdominal examinations are unremarkable. A circular, well-defined erythematous rash is noted on her back, which she had not noticed previously. Given these findings, what treatment should be prescribed?

Oral doxycycline for 3 weeks The presence of the target rash (erythema chronicum migrans), negates the need for serological tests to be ordered and is an indication for starting treatment for Lyme disease. A proportion of patients do not remember a tick bite, as with this patient. Doxycycline is first-line treatment for uncomplicated Lyme disease 44%

mx of tinea capitis

Oral itraconazole and topical ketoconazole This child has presented with tinea capitis or scalp ringworm, most likely from exposure from shedding of cat hair contaminated with fungal spores. Oral antifungals are required for treatment of tinea capitis owing to poor penetrance of topical agents through the root of the hair follicle. Adjunct treatment with topical agents can help to reduce spore transmission to family members. 36%

A 28 year old man presents to A+E after mild vomiting and diarrhoea for 24 hours. Whilst he cannot tolerate food, he is able to tolerate small amounts of fluid. On further questioning he says he had a barbecue yesterday with his family which consisted of chicken and beef. His observations are as follows RR- 18 02 sats-97% HR 80 BP 130/78 Temp 37.4. Clinically, he appears euvolaemic. What is the next best step in the management of this patient?

Oral rehydration salts This is likely to be a simple episode of gastroenteritis, most probably caused by Campylobacter jejuni. This is commonly acquired due to undercooked poultry. This patients observations are stable and he is not showing any signs of shock or severe infection. Therefore he would not require antibiotics or intravenous fluid resuscitation at present, as this will likely resolve on its own 62%

A 24-year-old recent immigrant from Albania presents to the emergency department with fever, headache and malaise. Over the past 24 hours he has also developed bilateral pain and swelling at the angle of the jaw, which is made worse by talking or chewing. On examination his pulse is 90/min, temperature 38.4ºC and bilateral palpable, tender parotid glands are noted.Given the likely diagnosis, which one of the following complications is he most likely to develop?

Orchitis is the most common complication of mumps in post-pubertal males. There is a link between mumps and pancreatitis (the 'M' in GET SMASHED) but this is much less common than orchitis.

A 22-year-old man is admitted to hospital with a lower respiratory chest infection. He had a splenectomy after being involved in a car accident. What is the most likely infective organism?

Organisms causing post splenectomy sepsis: Streptococcus pneumoniae Haemophilus influenzae Meningococci Encapsulated organisms carry the greatest pathogenic risk following splenectomy. The effects of sepsis following splenectomy are variable. This may be the result of small isolated fragments of splenic tissue that retain some function following splenectomy. These may implant spontaneously following splenic rupture (in trauma) or be surgically implanted at the time of splenectomy.

A 28-year-old man who has sex with men (MSM) presents to the genitourinary medicine (GUM) clinic complaining of pain during receptive intercourse for the last week. During this time he has also noticed fresh rectal bleeding from the rectum and a white-yellow discharge. He also recalls having a painless penile ulcer about one month ago, but this seems to have resolved itself now.The patient is known to be HIV positive and is receiving treatment for this: his last appointment was 3 months ago and showed an undetectable viral load. He has several regular sexual partners and does not use condoms.Observations are all within the normal range. On examination, you notice left-sided inguinal lymphadenopathy which is tender to touch. Rectal examination is limited by pain but peri-anal ulceration and a blood-stained mucopurulent discharge can be seen.What is the most likely diagnosis?

Painless genital pustule → ulcer → painful inguinal lymphadenopathy → proctocolitis - lymphogranuloma venereum This patient has lymphogranuloma venereum (LGV) which is caused by three serovars of Chlamydia trachomatis. Primary LGV infection causes a single painless pustule which develops into an ulcer - this is on the coronal sulcus of this penis in men, and the posterior vaginal wall or vulva in women. This pustule/ulcer lasts a few days-weeks and may go unnoticed. The secondary stage occurs weeks after the primary infection and causes painful inguinal lymphadenopathy, which is usually unilateral but can be bilateral. The classic 'groove sign' (inguinal ligament separating inguinal and femoral lymph nodes) is pathognomonic but rarely seen in practice. Proctocolitis may then develop, important risk factors being MSM and HIV infection. This causes rectal bleeding and discharge, and ulceration around the anus.

invubation periods

Parasites (often written as the name of the disease rather than the organism as above so not in italics) have a longer incubation period

A 32-year-old primigravida woman attends her 20-week antenatal scan screening appointment. There have been no concerns up to this point of her pregnancy.On examination, the midwife notes the woman's uterus to be 25cm. A subsequent ultrasound scan demonstrates polyhydramnios, ascites and fetal skin oedema.What is the most common and likely underlying cause for this presentation?

Parvovirus B19 is correct. It is the most common non-immune cause of foetal hydrops in pregnancy (the only immune cause is Rh disease). The virus causes foetal anaemia which results in fluid building up in the serous cavities of the foetal body. This leads to high-output cardiac failure, as the foetal body responds to the increased demands from the anaemia. This can be seen in the question by the polyhydramnios (increased amniotic fluid), ascites and foetal skin oedema. Untreated, this can lead to miscarriage or stillbirth. A treatment in-utero is foetal transfusion, whereby, red blood cells are injected into the fetus whilst in the womb.

A 40-year-old patient who is on methotrexate for rheumatoid arthritis presents because her 5-year-old daughter has been suffering from chickenpox and she is concerned about developing it. She has not previously had had chickenpox herself and is currently well, with no symptoms or rash.What would be the most appropriate advice to give this patient?

Patients who are immunosuppressed secondary to long-term steroids or methotrexate should receive VZIG if they are exposed to chickenpox and have no antibodies to varicella

A 76-year-old woman presents to her GP after looking after her grandchild yesterday, who was ill with chickenpox at the time. She doesn't think she has had chickenpox before. She is worried since she knows she has a weakened immune system due to long term methotrexate treatment for rheumatoid arthritis. On examination, there is no rash or systemic upset and she feels well in herself. Her varicella antibodies are checked and are negative.What is the most appropriate management?

Patients who are immunosuppressed secondary to long-term steroids or methotrexate should receive VZIG if they are exposed to chickenpox and have no antibodies to varicella This woman has presented with long-term immunosuppression and chickenpox exposure. Her varicella antibodies confirm that she has never been exposed to chickenpox before, meaning if she was to become ill, it would be a primary infection. NICE recommends that at-risk groups who may develop a primary infection should be given varicella zoster immunoglobulin as prophylaxis, and therefore, this is the correct answer.

A 50-year-old patient established on methotrexate for rheumatoid arthritis presents to his GP as he is concerned about possibly being exposed to chickenpox. He received a call today from his family informing him that his grandson has just been diagnosed with chickenpox.The patient looked after his grandson 2 days ago, who did have an itchy rash at the time, but the family had thought this was just a mild reaction to a new washing powered.What is the most appropriate management option?

Patients who are immunosuppressed secondary to long-term steroids or methotrexate should receive VZIG if they are exposed to chickenpox and have no antibodies to varicella therefore send a blood test to see if their are varicella antibodies first

which HIV pts should have co-trimoxazole

Patients with HIV and a CD4+ T lymphocyte cell count <200/mm³ indicates immunodeficiency. This suggests that the patient is highly susceptible to opportunistic infection such as pneumocystis jiroveci pneumonia (PCP). Antimicrobial prophylaxis is indicated in patients with CD4+ count <200/mm³, as it significantly reduces the risk of developing PCP. The antimicrobial prophylaxis regimen of choice is co-trimoxazole (trimethoprim with sulfamethoxazole).

A 38-year-old man who has recently emigrated from eastern Europe presents to the Emergency Department one hour after sustaining a 4 cm deep laceration to the dorsum of his left hand. He works as a builder and sustained the laceration after cutting into a cardboard box using a Stanley knife.On examining the wound there is no sign of a foreign body or neurovascular deficit. He is referred to Plastics for apposition of the wound.You ask him about his tetanus vaccination status. He has 'no idea' but can remember getting some vaccinations as a child.What is the most appropriate action with respect to tetanus?

Patients with an uncertain tetanus vaccination history should be given a booster vaccine + immunoglobulin, unless the wound is very minor and < 6 hours old

You are working in the urgent care centre, where Sarah, a 46-year-old woman comes to see you with a laceration to her left lower leg. She explains that 3 hours ago she was using a sharp gardening tool to remove weeds in her garden when it slipped and hit deep into her foot.On examination, there is a 5.5cm laceration on the medial aspect of Sarah's left lower leg which is 3cm deep. There is a lot of soil visible inside the wound.Sarah is unsure about her tetanus immunisation history and there is no further information about this in her records.With regards to tetanus prophylaxis, which of the following option is most appropriate to manage Sarah's wound?

Patients with an uncertain tetanus vaccination history should be given a booster vaccine + immunoglobulin, unless the wound is very minor and < 6 hours old

important organism causing LRTI in cystic fibrosis patients

Patients with cystic fibrosis develop bronchiectasis early on during their life resulting in repeated hospital admissions with lower respiratory tract infections. The pathological process behind bronchiectasis results in sputum pooling within the larger airways with poor removal. Subsequently colonisation occurs with bacteria and occasionally fungi. The most common bacteria is the gram negative rod Pseudomonas aeruginosa and should always be taken into account if providing empirical treatment. If the patient is systemically well then antibiotic sensitivities should be sought from a culture sample before starting treatment. However, an anti-pseudomonal agent such as piperacillin with tazobactam or ciprofloxacin should be used as part of empirical treatment for sepsis in cystic fibrosis patients.

A 27-year-old man attends the general practitioner (GP) complaining of 'spots' around the head of his penis. He reports that they have always been there and that they have not changed at all. The patient reports that he is not sexually active and has not had any sexual partners in the past.On examination there are multiple flesh coloured papules on the corona of the penis. The GP explains that these are pearly penile papules.What is the single best advice the GP can give the patient?

Pearly penile papules are a normal variant of the glans. They are approximately 1-2mm in size and are distributed around the corona of the penis. They are asymptomatic and are often a cause of concern for patients, who should be reassured that no intervention is required.They are not caused by any form of virus and therefore sexual health screening should only be conducted on the grounds of genuine concern. Most initiatives for investigating common sexually transmitted infections cover an age range of 18 to 25-year-olds.

ou review a 14-year-old boy who has recently emigrated from Russia. He was involved in a car accident two years ago and underwent an emergency splenectomy.Following the accident, he takes penicillin V on a daily basis. He is unsure of his vaccination history.Which organism is he particularly susceptible to, despite taking prophylactic antibiotics?

Penicillin V would protect him against Streptococcus pneumoniae but not Haemophilus influenzae due to the production of beta-lactamases by the organism. Discuss (1)Improve

A 31-year-old man of no fixed abode presents to the emergency department complaining of progressively worsening lumbar back pain over the last 10 days. He now finds the pain debilitating on movement. He reports no history of trauma. He feels generally well in himself and has no respiratory or cardiovascular symptoms. He denies fevers, but feels he might have lost some weight and recently experienced night sweats; however, he attributes this to a history of intravenous drug use and recent withdrawal. He currently moves between living in homeless shelters and tent-cities. He smokes 10 cigarettes a day and drinks 20 units of alcohol a day. His observations are: Temperature 37.9°C RR 18 breaths per minute HR 85 bpm BP 132/89 mmHg Oxygen saturation 97% in air Physical findings reveal localised tenderness to the thoracolumbar junction and restricted spinal motion due to kyphotic deformity. Neurological examination of the lower limbs revealed pain on both active and passive movement, with power 4/5 throughout and slightly impaired sensation bilaterally. He denies any saddle paraesthesia, erectile dysfunction, urinary or bowel incontinence. A plain X-ray of the spine demonstrates a reduction in the L1 vertebral height and an irregularly defined anterior vertebral margin. Biopsy was subsequently performed, which reported a chronic granulomatous inflammatory process with presence of necrosis. What is the most likely diagnosis?

Pott's disease of the spine This patient has presented with Pott's disease, or tubercular spondylitis, one of the commonest extrapulmonary manifestations of tuberculosis. Risk factors for infection with tuberculosis include prolonged exposure to poor living conditions and being immunocompromised. Clinical features include localised back pain, neurological deficits and radiographic findings indicative of vertebral body involvement. A biopsy demonstrating granuloma formation is highly suggestive of a tubercular process. 60%

A 27-year-old man is recovering on the ward following an uncomplicated open reconstruction of the lateral collateral ligament of his left knee. He initially recovers well and is eating and drinking after 24 hours. However, within 36 hours of the procedure, his temperature spikes to 38.6 °C, with a heart rate of 104 bpm and blood pressure of 106/75 mmHg. Cardiovascular and respiratory examinations are unremarkable, but inspection of the surgical site reveals severe swelling and discolouration of the knee, with blisters protruding from the surgical incision. Subcutaneous emphysema can be seen extending distally. He complains of severe pain and weakness in his lower leg that is not adequately controlled with his postoperative analgesia. Given the likely complication that has occurred, what is the most appropriate management option for this patient at this time?

Penicillin and clindamycin with surgical debridement This patient has developed gas gangrene postoperatively, most likely due to Clostridium perfringens. Management requires prompt antimicrobial treatment with good Gram positive, negative and anaerobic coverage. Penetrance into necrotic tissue is however often suboptimal and surgical debridement is required to prevent further spread of infection. 73%

A 30-year-old lady from Eswatini presents with progressive shortness of breath and lethargy. She reports recurrent chest infections over the last few years but this episode is different. She does not have a cough. She is taking no medications. On examination her chest is clear. Her saturations are 98% on air, at rest and 94% after she walks around the consultation room. Chest X-ray is normal.What is the most likely diagnosis?

Pneumocystis jiroveci commonly presents desaturation on exertion and often Chest x-ray appears normal. It almost exclusively happens in immunosuppressed patients. Eswatini has one of the highest prevalences of HIV in the world (source WHO), making it relevant in this particular scenario. The patient has had recurrent chest infections which in such a young patient should be suggestive of immunosuppression.

A 39-year-old newly diagnosed HIV positive gentleman presents with a dry cough, fever and pleuritic chest pain. Based on findings in the history, examination and investigations, he is treated empirically for presumed Pneumocystis jirovecii pneumonia. Which of the following findings would you expect to find on further investigation?

Pneumocystis jirovecii pneumonia usually occurs at CD4 counts under 200 cells/mm³

A 31-year-old female with no past medical history of note is admitted to hospital with dyspnoea and fever. She has recently returned from holiday in Turkey. A clinical diagnosis of pneumonia is made. On examination she is noted to have an ulcerated lesion on her upper lip consistent with reactivation of herpes simplex. Which organism is most associated with this examination finding?

Pneumonia caused by Streptococcus pneumoniae is associated with cold sores

This patient has been treated acutely for a type of non-falciparum malaria with an appropriate acute treatment (chloroquine) which seems to have worked. Since Plasmodium vivax and Plasmodium ovale species both have hypnozoite stages in the liver, these need to be eradicated with an additional therapy to prevent relapse of malarial illness.

Primaquine is the appropriate treatment for this.

HIV, neuro symptoms, widespread demyelination

Progressive multifocal leukoencephalopathy (PML)

A 33-year-old man with HIV presents to his general practice surgery. Over the last few months, he has been getting more clumsy and drops things more often. People have commented that his speech has become slurred and he has been more irritable than usual. His CD4 count is 110 cells/mm³. He admits he has not been compliant with his medication. An MRI is ordered which shows features of progressive multifocal leukoencephalopathy. What type of viral DNA will be found in his CSF?

Progressive multifocal leukoencephalopathy is caused by the JC virus and BK virus in some occasions. The virus stays dormant and controlled by the immune system.

A 73-year-old man attended the emergency department with a 6-week history of flu-like symptoms, weight loss and shortness of breath. At presentation, he was determined to be septic and unresponsive to resuscitation, so was started on vasopressor support and intubated. He was initially treated empirically with IV co-amoxiclav. A subsequent echocardiogram demonstrated a vegetation of the aortic valve, and blood cultures showed growth of Staphylococcus aureus. His antibiotic therapy was adjusted to Flucloxacillin, and his condition initially improved with repeat blood cultures 6 days later demonstrating no visible growth. On day nine of intubation, he developed a fever of 39.4 °C, with increased oxygen demand and excessive production of rusty-coloured sputum. A sample was sent for culture, and a chest X-ray performed in the meantime revealed widespread peribronchial infiltration and bilateral upper zone consolidation. Microscopy, culture and sensitivity of the sputum sample demonstrated growth of Gram-negative biofilm-forming bacilli with intrinsic resistance to ampicillin, sulphonamides, tetracyclines, chloramphenicol and first- and second-generation cephalosporins. What is the most likely causative agent behind this patient's deterioration?

Pseudomonas aeruginosa P. aeruginosa is a clinically relevant Gram-negative bacterium that commonly causes infections in immunocompromised or severely ill patients, as well as those who require lines, catheters or other foreign body adjuncts. This patient has developed ventilator-associated pneumonia. P. aeruginosa is associated with high levels of intrinsic resistance to multiple classes of antibiotics. 48%

A 61-year-old man is seen in the diabetic foot clinic because of a chronic ulcer on his left leg. The ulcer has been present for the past 5 months and is well demarcated with no sensation at the base of the ulcer. He has grown the same bacteria from swabs taken from the ulcer multiple times. On examination his heart rate is 81 bpm, his blood pressure is 132/83 mmHg and his temperature is 37.1ºC. The base is wet with a pale green slough over it and smells damp and offensive.Which of the following organisms is most likely to be responsible for this ulcer based on this clinical picture?

Pseudomonas commonly causes opportunistic infections in patients with any degree of immunosuppression and is an important pathogen in diabetic patients, especially in neuropathic ulcers. The chronicity of the ulcer is the biggest clue to the pathogen as all the other options are bacteria associated with invasive or necrotic infections which would be associated with a systemic response and a much shorter history. Pseudomonas is the only pathogen which could cause a chronic sloughy ulcer with a duration of months without any systemic symptoms. The green colour is also characteristic of Pseudomonas infections as the bacteria produce a blue-green pigment called pyocyanin as well as the classical offensive smell.

A 26-year-old man has noticed live lice in his pubic hair. It is associated with intense itching. He has had multiple casual sexual partners in the last few months. He does not wish to attend the sexual health clinic and is asking for treatment in primary care.

Pubic lice infestation is caused by the parasite Phthirus pubis. It is treated with insecticides - either malathion lotion or permethrin cream. Both should be applied to the whole body and washed off after 12 hours. The patient should be advised to reapply the treatment 7 days after initial application to ensure that lice emerging from surviving eggs are eradicated.

hep E

RNA hepevirus spread by the faecal-oral route incubation period: 3-8 weeks common in Central and South-East Asia, North and West Africa, and in Mexico causes a similar disease to hepatitis A, but carries a significant mortality (about 20%) during pregnancy does not cause chronic disease or an increased risk of hepatocellular cancer a vaccine is currently in development, but is not yet in widespread use

Hepatitis E

RNA hepevirus spread by the faecal-oral route. incubation period: 3-8 weeks. common in Central and South-East Asia, North and West Africa, and in Mexico causes a similar disease to hepatitis A, but carries a significant mortality (about 20%) during pregnancy does not cause chronic disease or an increased risk of hepatocellular cancer a vaccine is currently in development, but is not yet in widespread use

An 18-year-old man is bitten by a frantic dog whilst taking a gap year in Ecuador. He is worried about rabies and phones for advice. He was not immunised against prior to travelling to Ecuador. What is the most appropriate advice after thorough cleansing of the wound?

Rabies - following possible exposure give immunglobulin + vaccination

ou are phoned for advice. The parents of a 19-year-old man have just been messaged by their son who is currently backpacking in Thailand. Earlier in the day he was bitten by a dog whilst staying in a rural community. Prior to travelling, he received vaccination against rabies as he was going to be visiting many rural areas. What is the most appropriate advice?

Rabies is nearly always fatal if untreated. Whilst you are not expected to remember all the countries where there is a high risk of rabies it is clear that being bitten by a dog in a rural area represents a risk. He needs to urgently see a local doctor as booster vaccination is indicated to minimise his risk of developing rabies. Flying home simply delays the most important intervention. Discuss (6)Improve

The phlebotomist in a GP surgery sustains a needlestick injury whilst taking blood from a patient who is known to be HIV positive. Following thorough washing of the wound what is the most appropriate management?

Refer to Emergency Department + oral antiretroviral therapy for 4 weeks

A 15-week pregnant woman contacts her GP for advice because her 6-year-old son has recently been diagnosed with erythema infectiosum. She is clinically well and is immune to rubella. The GP arranges serology for parvovirus B19, which reveals parvovirus IgM positive and IgG negative.What would be the most appropriate management?

Refer to fetal medicine for further tests, bloods show a recent infection with parvovirus Parvovirus B19 is the cause of erythema infectiosum (also known as slapped cheek). It is a common childhood infection, with many adults being previously infected (but the infection being asymptomatic).Infection with parvovirus B19 is uncommon in pregnancy and the majority of pregnant women with parvovirus infection will deliver healthy babies. However, potential complications can include hydrops fetalis and fetal death. The risk of these complications is similar in symptomatic and asymptomatic women with parvovirus.Parvovirus B19 is infectious from up to 3 weeks before the rash develops. It is no longer infectious once the rash appears. The risk of transmission increases with gestational age.Immediate serological testing for parvovirus B19 is required for all pregnant women in contact with someone with a rash consistent with parvovirus. Significant contact is defined as face-to-face or in the same room for more than 15 minutes in the 3 weeks before the index case developed a rash. It is also important to ensure the pregnant woman is immune to rubella.Blood should be tested for parvovirus B19 specific IgM and IgG.IgG positive & IgM negative - shows immunity to parvovirus. Reassure, no further action.IgG negative & IgM positive - non-immune. Recent parvovirus infection in last 4 weeks. Refer immediately for further tests/fetal medicine.IgG negative & IgM negative - repeat test in 4 weeks. If both tests still negative, this confirms susceptibility, but no recent infection. Reassure, further action required only if subsequent exposure occurs.

A 19 year old Thai man presents to the respiratory clinic with a cough he has had for six weeks. On further questioning he has been coughing up blood, has had some night sweats and has lost 5 kg. He has no significant past medical history and takes no medication. What is the next best step in the management of this patient?

Start initial phase quadruple antibiotic therapy immediately It is important to start therapy without delay when signs and symptoms point to a diagnosis of acute pulmonary TB. According to both British Thoracic Society and NICE guidelines, you can start treatment without waiting for the results of the sputum culture (which you would ideally ask the patient to produce prior to starting therapy). The classic quadruple therapy regimen includes Isoniazid, Rifampicin, Ethambutol and Pyrazinamide 52%

A 25 year old female presents to A&E with a 3 day history of fever and bilateral painless parotid swelling. She also complains of right sided hearing loss. She has no other past medical history. There are no other abnormalities on physical examination. Which is the next best investigation to confirm the diagnosis?

Salivary IgM testing This is the diagnostic test for mumps, which detects mumps IgM antibodies. Mumps causes parotid swelling and occasionally sensorineural hearing loss. Reverse transcriptase PCR on an oral swab may also be performed 48%

A 43-year-old woman is brought to the emergency department by her husband after she collapsed earlier at home. She is pyrexial and is lying very still. She has no underlying medical conditions and takes no medications. She reports feeling unwell for the last 2 weeks, but did not seek medical attention as she was busy with a backlog of work following her return from a holiday in Vietnam 1 month ago. Her symptoms started with a low-grade fever and diarrhoea, then progressed to nonspecific abdominal pain and headache 1 week ago. She denies noticing any blood in her stool. She reports that, since collapsing earlier, the abdominal pain has become unbearable with 10/10 severity. On examination, she has a tender, rigid abdomen that contracts involuntarily on palpation. Periumbilical rose-coloured spots and hepatosplenomegaly are noted. A CT scan demonstrates free gas in the peritoneal cavity, ileocecal thickening and a small volume of free fluid within the abdomen. What is the most likely underlying organism responsible for this patient's presentation?

Salmonella enterica serovar Typhi This patient has developed intestinal perforation secondary to typhoid fever, a systemic and progressive disease that occurs following exposure to S. enterica Typhi. The initial symptoms include gastrointestinal upset, headache and fever, which may then progress to abdominal pain and neurological symptoms including delirium and coma. Hepatosplenomegaly and rose-coloured spots may be seen in affected individuals. Gastrointestinal ulceration with bleeding and perforation is a major complication of typhoid fever and requires prompt operative treatment and antimicrobial therapy. 63%

A 7 year old boy presents to the Emergency Department with pain in his left hip. He recently moved to UK from Zimbabwe and has had several episodes of severe pain in his hip over the past five years. His mother reports that he has a problem with his blood but is not sure what. On examination, the left hip joint is tender to touch and he is unable to bear weight on the left side. The range of motion is also restricted due to pain. What is the most likely pathogen to cause this presentation?

Salmonella species This is the correct answer. Sickle cell disease is more common in African countries and his episodes of severe hip pain are most likely acute vaso-occlusive crises affecting the femoral head. One of the orthopaedic manifestations of sickle cell disease is osteomyelitis of the affected joint, which is most commonly the femoral or the humeral head. The most likely pathogen is the Salmonella species 37%

A 25-year-old man has had 5 days of diarrhoea which he describes as 'yellowy green' in colour and non-bloody. He also reports a persistent fever and diffuse abdominal pain for the past 2 weeks, which started a few days after returning from a trip to Indonesia.On examination, there is mild hepatomegaly and numerous small pink spots over the abdomen, mostly around the umbilicus. His temperature is 39.5 ºC, pulse rate 70 bpm, BP 130/90 mmHg, and respiratory rate 20 breaths per minute.Given the likely diagnosis, what is the causative organism?

Salmonella typhi causes typhoid fever, which characteristically manifests with non-bloody, yellow-green ('pea-soup') diarrhoea, persistent fever, and abdominal pain. (Note: paratyphoid fever is caused by Salmonella paratyphi and has a very similar clinical manifestation to typhoid fever, so they are collectively referred to as enteric fever). The incubation period of enteric fever is 5-30 days, and has 3 characteristic phases: - Week 1: systemic illness (headache, rising fever), constipation, relative bradycardia. - Week 2: persistent fever, 'rose-coloured' spots (most commonly around the umbilicus) and non-bloody yellow/green diarrhoea. - Week 3: features of week 2 with possible complications (e.g. hepato- and/or splenomegaly, GI bleed/perforation etc).

A 19-year-old woman presents to the emergency department with a 2-day history of right upper quadrant pain and diarrhoea. She returned from a holiday to Uganda 1 month ago. She reports she slept under netting, used insect repellent daily, and took regular atovaquone and proguanil. She received all recommended travel immunisations at her local travel clinic before her trip. On further questioning, she reports feeling generally run down over the last week, with a fever, mild headache, general myalgia and a dry cough. When asked, she recalls developing an itchy tingly red rash after swimming in Lake Victoria during her holiday, which has since resolved. She is usually fit and well, with no significant past medical history. She is sexually active and uses only condoms for contraception; she reports having sex with two casual partners during her time in Uganda. Blood tests are performed which show evidence of eosinophilia. What is the most likely diagnosis?

Schistosomiasis This patient most likely has Katayama fever, an acute presentation of schistosomiasis infection. Katayama fever tends to present with a broad spectrum of symptoms around a month after initial exposure. A characteristic 'swimmer's itch' may be seen following exposure to fresh water in an endemic area. Eosinophilia is a common laboratory finding. 81%

A 62-year-old woman attends the GP with a 7-day history of dysuria and urinary urgency.She experienced a similar episode 12 months ago. She was also admitted 6 months ago for renal colic that resolved conservatively.A urine dipstick is performed. It shows ++nitrites, ++leucocytes and ++erythrocytes. Her urine is cloudy in appearance. A mid-stream urine culture is sent.What part of this patient's history warrants this investigation to be performed?

Send an MSU for all women with a suspected UTI if associated with visible or non-visible haematuria. Urine dipstick results is the correct answer. The patient in the vignette has a urinary tract infection confirmed by her urine dipstick results. There is also the presence of non-visible haematuria (++ erythrocytes on urine dipstick), which warrants a midstream urine culture. The presence of visible or non-visible haematuria warrants MSU for culture as it suggests atypical organisms colonising the urinary tract.According to NICE guidelines, other indications for a mid-stream urine culture include: Have symptoms that are persistent or do not resolve with antibiotic treatment. Have recurrent UTI (2 episodes in 6 months or 3 in 12 months). Have a urinary catheter in situ or have recently been catheterised. Have risk factors for resistance or complicated UTI such as abnormalities of the genitourinary tract, renal impairment, residence in a long-term care facility, hospitalisation for more than 7 days in the last 6 months, recent travel to a country with increased resistance or previous resistant UTI. Have atypical symptoms. Pregnant Age more than 65 years old.

sepsis

Sepsis is defined as life-threatening organ dysfunction caused by a dysregulated host response to an infection. Sepsis is increasingly recognised as an important cause of mortality in the UK and there has been increasing efforts recently to improve the care of patients who present with sepsis. How sepsis is classified has changed in recent years - the Surviving Sepsis Guidelines were updated in 2017.The new guidelines recognise the following terms: sepsis: life-threatening organ dysfunction caused by a dysregulated host response to infection septic shock: a more severe form sepsis, technically defined as 'in which circulatory, cellular, and metabolic abnormalities are associated with a greater risk of mortality than with sepsis alone'* The old category of severe sepsis is no longer used.The term 'systemic inflammatory response syndrome (SIRS)' has also fallen out of favour. Adult patients outside of ICU with suspected infection are identified as being at heightened risk of mortality if they have quickSOFA (qSOFA) score meeting >= 2 of the following criteria: respiratory rate of 22/min or greater, altered mentation, or systolic blood pressure of 100mmHg or less qSOFA score: RR > 22/min. altered mentation. systolic BP < 100mmHg. within an ICU setting a full SOFA** score is often used. Mx: NICE released their own guidelines in 2016. These focused on the risk stratification and mx of patients with suspected. For risk stratification NICE recommend using the following criteria: Red flag criteria; Responds only to voice or pain/ unresponsive Acute confusional state Systolic B.P <= 90 mmHg (or drop >40 from normal) Heart rate > 130 per minute Respiratory rate >= 25 per minute Needs oxygen to keep SpO2 >=92% Non-blanching rash, mottled/ ashen/ cyanotic Not passed urine in last 18 h/ UO < 0.5 ml/kg/hr Lactate >=2 mmol/l Recent chemotherapy amber flag: Relatives concerned about mental status Acute deterioration in functional ability Immunosuppressed Trauma/ surgery/ procedure in last 6 weeks Respiratory rate 21-24 Systolic B.P 91-100 mmHg Heart rate 91-130 OR new dysrhythmia Not passed urine in last 12-18 hours Temperature < 36ºC Clinical signs of wound, device or skin infection Clearly the underlying cause of the patients sepsis needs to be identified and treated and the patient supported regardless of the cause or severity. If however any of the red flags are present the 'sepsis six' should be started straight away: 1. Administer oxygen: Aim to keep saturations > 94% (88-92% if at risk of CO2 retention e.g. COPD) 2. Take blood cultures 3. Give broad spectrum antibiotics 4. Give intravenous fluid challenges: NICE recommend a bolus of 500ml crystalloid over less than 15 minutes 5. Measure serum lactate 6. Measure accurate hourly urine output *these patients can be clinically identified by a vasopressor requirement to maintain a MAP ≥ 65mmHg and serum lactate >2mmol/L in the absence of hypovolemia**To help identify and categorise patients the Sequential (Sepsis-Related) Organ Failure Assessment Score (SOFA) is increasingly used. The score grades abnormality by organ system and accounts for clinical interventions. However, laboratory variables, namely, PaO2, platelet count, creatinine level, and bilirubin level, are needed for full computation.

Input from the infectious diseases team is requested for a 40-year-old sewerage worker who presented to the emergency department complaining of fever and right upper quadrant abdominal pain. He mentions that in the preceding week he experienced a flu-like illness and he noticed his eyes had gone red. On examination, there is tender hepatomegaly and the patient appears jaundiced.What test is most commonly used to confirm this patient's underlying condition?

Serology is the most commonly used test for the diagnosis of leptospirosis, however, antibodies may not be present until after 7 days

sewage workers are at risk of what condition

Sewage workers are at risk of leptospirosis which is transmitted through rat urine. It typically presents as above and can progress to renal failure.

TPHA remains positive, VDRL becomes negative

She has been successfully treated for syphilis

A 30-year-old woman with rheumatoid arthritis and on long-term methotrexate presents to clinic today as she is very worried that 3 days ago whilst attending her niece's birthday party, she came into contact with another child who was found to have chickenpox. She cannot remember if she had chickenpox as a child and is wondering whether she requires any treatment.What is the next most appropriate course of action?

She should receive VZIG if antibody tests are negative

A 45-year-old woman presents with a 3 day history of fever, myalgia and fatigue 1 week after returning from Kenya where she was visiting relatives.On examination she has mild jaundice and splenomegaly of 4cm.What is the most likely diagnosis?

Splenomegaly and jaundice are commonly seen in malaria and people travelling to malaria areas to visit friends and relative are often the most at risk as if they were born in a country with high levels of malaria they may believe they still have immunity and so don't take antimalarials. hep A would have a longer incubation period and present with more jaundice and less splenomegaly

A 4 year old girl is brought into Paediatric Emergency with a 2 day history of high fever and irritability. She has developed red, peeling skin over the past day. Her mother reports that she had a sore throat prior to this, which has since resolved. On examination, she has widespread erythema with several blisters. Her skin is extremely tender to touch and peels easily when rubbed. What is the most likely diagnosis?

Staphylococcal scalded skin syndrome This is caused by Staphylococcus aureus. It presents with red tender blistering skin and a positive Nikolsky sign, in which the layers of skin separate on gentle pressure. A prodrome of sore throat or conjunctivitis may occur 70%

staphylococci

Staphylococci are a common type of bacteria that are often found normal commensal organisms but may also cause invasive disease. Some basic facts include: Gram-positive cocci facultative anaerobes produce catalase the two main types of staphylococci you need to know to know about are staph aureus and staph epidermis. staph aureus: -coagulase-positive -causes skin infections (e.g. cellulitis), abscesses, osteomyelitis, TSS. staph epidermis: - coagulase-negative -cause of central line infections and infective endocarditis.

A 19 year old male is seen in General Practice with a six-hour history of vomiting, abdominal pain and non-bloody diarrhoea. He has no other past medical history, has no allergies and does not take any medications. He has not been abroad in the last five years. His housemate has had similar symptoms, and they attribute this to some cheese and ham sandwiches they ate at a cafe this morning. What is the most likely cause of their gastroenteritis?

Staphylococcus aureus Staph aureus gastroenteritis is associated with eating contaminated milk products and hand-made foods such as sandwiches, puddings and pastries. It has a very fast onset (within 1-6 hours post ingestion of the contaminated food), and vomiting and diarrhoea are major features. Illness usually lasts 1-3 days 32%

A 22-year-old lady is breastfeeding her first child. One week post partum she presents with a tender indurated mass in the right breast.

Staphylococcus aureus is the commonest cause of lactational mastitis.

A 39-year-old endurance athlete presents to her GP with a 2-month history of macroscopic haematuria and mild dysuria. She has also developed dyspareunia and, on occasion, has had some post-coital bleeding.She recalls developing a pruritic maculopapular rash on her left foot 4 months ago during a kayaking expedition in Sudan however, this quickly healed.Blood tests show eosinophilia and moderate anaemia. Her GP suspects she may have a parasitic infection of her urogenital system.What would be the gold standard investigation for diagnosing the cause of her haematuria?

Stool and urine microscopy is the correct answer here. This patient has urogenital schistosomiasis caused by an infection by the Schistosoma haematobium parasite (a species of digenetic trematode). The parasite enters the body by penetrating through the skin, often causing a local skin hypersensitivity reaction similar to the small, itchy maculopapular lesions described in this patient. Unlike other forms of Schistosoma spp. which release eggs in the intestine, Schistosoma haematobium flukes migrate to the venous plexus of the bladder and eggs are secreted in the urine. This results in the classic urogenital symptoms. The diagnosis of schistosomiasis is established by the presence of terminal spined eggs in the urine or stool, making this the gold standard for investigation.

A 42 year old male is brought by ambulance to A+E with sudden-onset diarrhoea and visual changes, followed by progressive dysphagia and generalized weakness. He has no past medical history, including no recent history of gastroenteritis or other infections. He takes no regular prescribed medications, though he has been taking intravenous heroin regularly for the past two years. On examination, he is alert but his eyelids are drooping and his speech is breathy and nasal in quality. Neurological examination reveals hypotonia, reduced reflexes and reduced power in all four limbs; but the upper limbs are more severely affected than the lower limbs. Respiratory examination is unremarkable. The patient is escalated immediately to ITU and supportive management started. Which diagnostic investigation should be ordered to confirm the diagnosis?

Stool toxin ELISA Recreational intravenous drug use is the commonest cause of wound-associated botulism, and so this is a diagnosis that cannot be missed. The toxin can be identified by ELISA using stool, vomit, serum or urine samples. Once the diagnosis is confirmed, antitoxin should be administered as soon as possible but can still be beneficial even if provided weeks after inoculation or ingestion of the toxin. It can slow progression but has no effect on toxin already bound to the neuromuscular junction 25%

Organisms causing post splenectomy sepsis:

Streptococcus pneumoniaeHaemophilus influenzaeMeningococci

hep a mx

Supportive with no increased risk of hepatocellular carcinoma in the future

mx of cellulitis

Supportive, symptomatic management - limb elevation, analgesia, antipyretics Will need hospital admission if there is significant systemic upset and/or co-morbidities. Antibiotics:Cellulitis: flucloxacillin + benzylpenicillin if severe, flucloxacillin alone if mild-moderate. Clarithromycin or clindamycin substituted if penicillin allergic.Erysipelas: can use the same treatment, though erythromycin preferred if penicillin allergic. Patients may need to take antibiotics for 1-2 months if they have lymphoedema (need to continue for 14 days from time of definite clinical response) Emollients are needed to keep skin hydrated If there is crepitus or necrosis, surgery is needed to exclude necrotizing fasciitis and to debride/amputate if necessary.

A 34-year-old man presents to his GP complaining of a 4-day history of a rash on his chest and palms. He reports feeling out of sorts recently, with a fever, arthralgia and headache over the last week, but denies any night sweats or weight loss. He reports that he is usually well and takes no regular medication, although he tentatively admits that he developed a red, solitary ulcer on his genitals 6 weeks ago, which he found embarrassing, so did not seek medical attention. He reports the ulcer was painless and resolved after 3 weeks with no medical intervention. He works as a banker and does not smoke or drink alcohol. He recently broke up with his long-term partner of 5 years and describes a few months of sex with multiple casual partners. On examination, he has a widespread symmetrical red papular rash on his trunk, palms and soles of his feet. The lesions feel coarse to touch. He denies any pain or itching. Examination of the mouth reveals discrete ulceration of the oral mucosa. What is the most likely diagnosis?

Syphilis This man has presented with a maculopapular rash on the trunk, palms and soles, along with ulceration of the oral mucosa and a recent history of untreated chancre. This is highly suggestive of secondary syphilis. 88% could be HIV but painless ulcer points to syphiliss

Syphilis

Syphilis is a sexually transmitted infection caused by the spirochaete Treponema pallidum. Infection is characterised by primary, secondary and tertiary stages. The incubation period is between 9-90 daysPrimary features chancre - painless ulcer at the site of sexual contact local non-tender lymphadenopathy often not seen in women (the lesion may be on the cervix) Secondary features - occurs 6-10 weeks after primary infection systemic symptoms: fevers, lymphadenopathy rash on trunk, palms and soles buccal 'snail track' ulcers (30%) condylomata lata (painless, warty lesions on the genitalia ) Tertiary features gummas (granulomatous lesions of the skin and bones) ascending aortic aneurysms general paralysis of the insane tabes dorsalis Argyll-Robertson pupil Features of congenital syphilis blunted upper incisor teeth (Hutchinson's teeth), 'mulberry' molars rhagades (linear scars at the angle of the mouth) keratitis saber shins saddle nose deafness

what counts as a reinfection of syphilis

Syphilis test results can often be difficult to interpret! The correct answer here is to treat again with benzathine penicillin, as this patient is likely to be reinfected with syphilis from the sexual encounter(s) he has had in the past three months. His RPR has risen by 4-fold. Treatment failure, although an option, is considered only once the possibility of re-infection has been excluded. e.g. it was 1/2 now it is 1/8 - this means there has been a reinfection

CSF: - cloudy - low glucose - high protein - high lymphocytes

TB

Cause of false negative Mantoux tests include and are not limited to:

TB AIDS Long-term steroid use Lymphoma Sarcoidosis Extremes of age Fever Hypoalbuminaemia Anaemia

A 45 year old male presents to A+E with a 1-week history of worsening back pain. It is now affecting the left buttock, hip and thigh, and has been accompanied by systemic fever, malaise and fatigue. He has a past medical history of HIV infection and excess alcohol use, and he has been homeless for the last 3 years. On examination, he is hemodynamically stable but pale, pyrexic and mildly disorientated. Respiratory, cardiovascular and abdominal examinations are unremarkable, but it is noted that he experiences severe pain whenever the left hip is hyperextended. What is the most likely diagnosis?

TB psoas abscess The patient has demonstrated a positive 'psoas' sign, with other symptoms consistent with a psoas abscess. Given his risk factors for TB, this is the most likely underlying diagnosis 73%

An 88-year-old woman was admitted to the ward following a fall. She was swabbed on admission for MRSA and the swab has come back positive. The swab has been cultured and the results show it is sensitive to Glycopeptides. What is the best treatment for this patient?

Teicoplanin This is a good example of a glycopeptide and is a synthetic version. A more classical Glycopeptide would be Vancomycin. Both drugs would be good options to treat MRSA (Methicillin-resistant Staphylococcus aureus). Some MRSA strains have resistance to Vancomycin as well and are known as VRSA 61%

influenza vaccine

The Department of Health recommends annual influenza vaccination for people older than 65 years and those with: chronic respiratory disease (including asthmatics who use inhaled steroids) chronic heart disease (heart failure, ischaemic heart disease, including hypertension if associated with cardiac complications) chronic kidney disease (at stages 3, 4 or 5, chronic kidney failure, nephrotic syndrome, kidney transplantation) chronic liver disease: cirrhosis, biliary atresia, chronic hepatitis chronic neurological disease: (e.g. Stroke/TIAs) diabetes mellitus (including diet controlled) immunosuppression due to disease or treatment (e.g. HIV) asplenia or splenic dysfunction pregnant women Other at risk individuals include: health and social care staff directly involved in patient care (e.g. NHS staff) those living in long-stay residential care homes carers of the elderly or disabled person whose welfare may be at risk if the carer becomes ill (at the GP's discretion)

A 32-year-old man presents to his general practitioner (GP) with a 3-month history of difficulty concentrating, generalised muscle aches and fatigue. The man works as a deer-stalker and spends most of his day walking through long grass in rural Scotland. He reports removing ticks from his legs most days.On examination, there is a circular 16cm rash on the lower leg consistent with erythema migrans.How should this patient be managed?

The NICE Clinical Knowledge Summary states that erythema migrans only occurs in Lyme disease and may be used to diagnose Lyme disease, so laboratory testing is unnecessary, and prompt treatment will prevent development of further symptoms. Therefore, no further tests are required in this case and treatment with doxycycline should be started immediately.Patients with focal symptoms - for example uveitis, facial nerve palsy or arthritis - should be referred to the relevant specialist or a discussion with a specialist should be sought. However, this should not delay antibiotic treatment.

enteric fever (typhoid/paratyphoid)

The Salmonella group contains many members, most of which cause diarrhoeal diseases. They are aerobic, Gram-negative rods which are not normally present as commensals in the gut. Typhoid and paratyphoid are caused by Salmonella typhi and Salmonella paratyphi (types A, B & C) respectively. They are often termed enteric fevers, producing systemic symptoms such as headache, fever, arthralgia. pathophysiology: - typhoid is transmitted via the faecal-oral route (also in contaminated food and water). features: initially systemic upset as above relative bradycardia abdominal pain, distension constipation: although Salmonella is a recognised cause of diarrhoea, constipation is more common in typhoid rose spots: present on the trunk in 40% of patients, and are more common in paratyphoid. possible complications include: osteomyelitis (especially in sickle cell disease where Salmonella is one of the most common pathogens) GI bleed/perforation meningitis cholecystitis chronic carriage (1%, more likely if adult females)

'...there is a small degree of lymphocytic portal and lobular inflammation. There is a ground-glass appearance to the cytoplasm of the hepatocytes...'Which of the following diagnoses best explains this patient's histology findings?

The appearance of ground-glass hepatocytes on light microscopy can point towards a diagnosis of chronic hepatitis B infection

how to remove a tick

The best way to remove a tick is using fine-tipped tweezers, grasping the tick as close to the skin as possible and pulling upwards firmly

A 27-year-old woman presents with painful genital and oral ulceration. Her past medical history includes treatment for a deep vein thrombosis three years ago.

The classic triad in Behcet's is oral ulcers, genital ulcers and uveitis. Venous thromboembolism is also seen.

A 61 year old homosexual man presents to the genitourinary medicine clinic with an ulcer on the tip of his penis. He is not sure how long it has been there as it has not caused him any pain. He is unsure if there has been any discharge. There is some inguinal lymphadenopathy on examination but this is not painful. What is the most likely cause?

The clue here is in the history. The lesion is painless. Haemophilus ducreyi presents with an ulcer which is painful. Similarly Lymphogranuloma venerum presents with tender swollen inguinal nodes (buboes) but in this case the lymphadenopathy is painless. HIV itself does not cause ulceration, though it may be possible that the ulcer itself is a result of an opportunistic infection in the context of HIV infection.Penile malignancy is not an unreasonable guess given this patients age and the painless nature of the lesion, however it is extremely rare. In this case the painless ulcer is most likey that of primary syphilis. Discuss (3)Improve

A 16-year-old boy comes to see his GP with his mother after complaining of a rash and tiredness. He has felt generally unwell for about 1 week now since returning from an adventure holiday in the USA. On examination he has a circular rash which is worse in the centre and the edges. This rash is warm and red but painless.Which of the following illness should be considered in this patient?

The clue in this question is the adventure holiday that patient has been on. When out in the countryside or forests, insect bite can be common and in the USA, tick bites are a concern. The characteristic bulls-eye rash seen in Lyme disease is known as erythema migrans and occurs in 50-75% of patients bitten by a tick and subsequently infected by the bacteria Borrelia. Other symptoms of Lyme disease are quite non-specific and tend to include fever, headache, and tiredness and can even include loss of ability to move one or both sides of the body.

A 22-year-old woman presents to her general practitioner with some green, foul-smelling vaginal discharge. It has been present for the last month and it is associated with pain. She is sexually active and had non-protected intercourse two months ago. She is not on any contraception. After a negative pregnancy test, the doctor performs a vaginal examination and measures the vaginal pH that is equal to 5.2. Which one of the following is the most likely organism causing her symptoms?

The correct answer is Trichomonas vaginalis. Trichomonas vaginalis and bacterial vaginosis have similar symptoms and are associated with a vaginal pH > 4.5. It usually presents with offensive, yellow/green, and frothy discharge, accompanied by vulvovaginitis and strawberry cervix.

A 58-year-old woman with a previous history of tuberculosis in her youth, presents with small volume haemoptysis. She has no other symptoms currently. Her rheumatoid arthritis is well controlled on methotrexate. She is a non-smoker. Her father died of mesothelioma. Examination identifies dullness to percussion at the right upper zone. Observations are within normal limits. Chest X-ray shows a partially-filled cavity with a crescent of air.What is the most likely diagnosis?

The correct answer is aspergilloma which classically forms in a cavitating lung lesion from previous tuberculosis.The other answers are all causes of haemoptysis.Reactivation of tuberculosis is offered as an option. We are not given any systemic symptoms such as weight loss, anorexia or night sweats, and the X-ray points towards aspergilloma. It will be important to culture and analyse the sputum.

A 35-year-old man requests testing for HIV (human immunodeficiency virus). He has recently discovered that his former partner has been diagnosed with AIDS (acquired immune deficiency syndrome). The last time he had sexual intercourse with his former partner was two years ago. He is currently fit and well and is asymptomatic.What is the most appropriate management option?

The correct answer is offer him testing for HIV p24 antigen and HIV antibody. Combination tests (HIV p24 antigen and HIV antibody) are now standard for diagnosing and screening HIV. CD4 lymphocyte cell count and viral load are measured as part of monitoring for those with confirmed HIV infection. This patient does not have a confirmed diagnosis, therefore the option offer him testing for CD4 lymphocyte cell count and viral load is incorrect. A full blood count may show features suggesting HIV such as lymphocytopenia. However, it is not a diagnostic test for HIV. NICE recommends offering an HIV test in primary care to those who: Request testing Have risk factors for HIV Have another sexually transmitted infection Have an AIDS-defining condition, an indicator condition, or clinical features of HIV infection.

A 24-year-old medical student is diagnosed with schistosomiasis after coming back from their elective in Malawi, where she had gone swimming in a lake.What is the best option for this patient's treatment?

The correct answer is praziquantel . This is an antihelminthic drug used to treat schistosomiasis, an infection caused by parasitic freshwater worms most commonly found in Africa. The worms burrow through the skin and enter the bloodstream, circulating around the body. They can enter the liver and are a major cause of cirrhosis in the developing world. The most appropriate treatment for them is praziquantel as mentioned previously. This often needs to be repeated after a few weeks as it is more effective when the worms have grown. Steroids may be used for symptomatic relief.

A 34-year-old man presents to the sexual health clinic after unprotected intercourse with a female partner. The encounter happened four weeks ago and today he is complaining of coryzal symptoms accompanied by myalgia. He has a past medical history of asthma, controlled with salbutamol. The doctor counsels him on HIV testing, to which he agrees.What is the most appropriate option regarding his management?

The correct answer is to order HIV p24 antigen and HIV antibody tests. This patient is presenting with the classical symptoms of HIV seroconversion (fever, malaise, myalgia). He had unprotected intercourse 4 weeks ago. Combination tests (HIV p24 antigen and HIV antibody) are now standard for the diagnosis and screening of HIV. HIV p24 antigen tests tend to turn out positive between 1 and 4 weeks post-exposure, whilst HIV antibody tests turn out positive between 4 weeks and 3 months post-exposure. When a patient at risk tests positive, the guidelines suggest repeating the test to confirm the diagnosis before starting any treatment.

A 24-year-old man presents to the sexual health clinic after unprotected intercourse with a female partner. The encounter happened two weeks ago and today he is complaining of coryzal symptoms accompanied by myalgia. He has a past medical history of asthma, controlled with salbutamol. The doctor counsels him on HIV testing, to which he agrees. A later repeated combination HIV test is positive.What is the most appropriate management for this patient?

The correct option is to prescribe two nucleoside reverse transcriptase inhibitors (NRTI) and a protease inhibitor (PI). This patient presented with seroconversion symptoms such as coryza and myalgia, and he had two positive combinations of HIV tests, which are diagnostic for the condition.

For a patient undergoing an elective splenectomy, when is the optimal time to give the pneumococcal vaccine?

The current British National Formulary recommends giving the vaccine at least 2 weeks before elective splenectomy

A 28-year-old woman who is 10 weeks pregnant comes to see you for her booking appointment. She has heard there are some vaccinations offered in pregnancy and wants more information.Which of the following combinations of vaccinations are routinely offered to pregnant women in the UK?

The influenza vaccine is offered to all pregnant women who are pregnant during the flu season, regardless of trimester.The pertussis (whooping cough) vaccine has been offered to all pregnant women since October 2012. It is ideally given at 28-32 weeks but can be given up to 38 weeks and is repeated in every pregnancy. There is no individual pertussis vaccine therefore it is given in a vaccine alongside polio, diphtheria and tetanus. It was introduced due to an increasing incidence of whooping cough in babies less than 2 months old who were too young to have their pertussis vaccination.

A 7-year-old child comes to the GP for the annual influenza vaccine. The mother states that he has been ill for the last couple of days with a productive cough and fever. The child has not been given any medications and is not known to have allergies.Regarding the flu vaccine, what should you do?

The intranasal vaccine is live and the intramuscular is inactivated however NICE guidance states that the influenza vaccine (whether intramuscular or intranasal) should not be given to people who are acutely unwell. The vaccine should be postponed until the patient is better. Discuss (1)Improve

A 28-year-old female sex worker presents to her general practitioner with a sore throat, fever and fatigue which has been going on for two weeks. When examining the throat, the GP notices several white patches on the lateral borders of the tongue as well as the buccal mucosa. These lesions are not painful and had not been noticed by the patient until now. The patches can't be removed by the application of light pressure.Given the patient's history, what is the most likely cause of the lesions on the tongue?

The likely diagnosis, in this case, is oral hairy leukoplakia, which is associated with Epstein-Barr virus (EBV) infections. EBV can cause infectious mononucleosis, which classically presents with a sore throat, fever and fatigue. EBV infections in HIV patients can cause oral hairy leukoplakia, which presents with white patches on the tongue, usually on the lateral borders. The patient's social circumstances make her high risk for HIV exposure. These patches are painless or give only mild discomfort, and often go unnoticed. They cannot be removed from the tongue with light pressure. Treatment is non-specific, and the condition usually improves if antiretroviral therapy is started early.

Red man syndrome is associated with rapid intravenous infusion vancomycin. It is a common adverse reaction of intravenous vancomycin use and is a distinct entity from anaphylaxis due to vancomycin use. Typical symptoms include redness, pruritus and a burning sensation, predominantly in the upper body (face, neck and upper chest). Severe cases can be associated with hypotension and chest pain.The pathophysiology of red man syndrome is attributed to vancomycin-related activation of mast cells with release of histamine. how is ut mx

The management of red man syndrome involves cessation of the infusion, and when symptoms have resolved, recommencement at a slower rate. In patients who are more symptomatic antihistamines can be administered, and may require intravenous fluids if the syndrome is associated with hypotension.

what ix are done before starting someone on TB medication

The medication regime is Rifampicin, Isoniazid, Pyrazinamide, and Ethambutol. We need to have baseline LFTs as all drugs in the regimen are hepatotoxic. We test Us and Es to monitor electrolyte disturbances and any elevation in Creatinine as a result of treatment. We need a baseline visual assessment as Ethambutol can cause loss of vision. We need an FBC as a baseline, and to assess for platelet count which can be important in the context of hepatotoxicity.

Non-falciparum malaria

The most common cause of non-falciparum malaria is Plasmodium vivax, with Plasmodium ovale and Plasmodium malariae accounting for the other cases. Plasmodium vivax is often found in Central America and the Indian Subcontinent whilst Plasmodium ovale typically comes from Africa. plasmodium knowlesi is another non-falciparum species which causes clinical pathology, found predominantly in south east asia. Features: general features of malaria: fever, headache, splenomegaly Plasmodium vivax/ovale: cyclical fever every 48 hours. Plasmodium malariae: cyclical fever every 72 hours Plasmodium malariae: is associated with nephrotic syndrome. ovale and vivax malaria have a hypnozoite stage and may therefore relapse following tx. tx: in areas which are known to be chloroquine-sensitive then WHO recommend either an artemisinin-based combination therapy (ACT) or chloroquine in areas which are known to be chloroquine-resistant an ACT should be used ACTs should be avoided in pregnant women patients with ovale or vivax malaria should be given primaquine following acute treatment with chloroquine to destroy liver hypnozoites and prevent relapse

A 23-year-old man presents to his general practitioner with a one week history of pain on urination and a pus-like discharge from the end of his penis. There is no history of haematuria, fever, abdominal pain or joint pain and the patient is otherwise well. He is sexually active and has had penetrative sex with three women in the past two months.Examination of the genitalia is unremarkable.Given the most likely diagnosis, which investigation is most appropriate?

The most likely diagnosis here is chlamydia, although there may also be concomitant gonorrhoea infection. Chlamydia is diagnosed using nucleic acid amplification tests (NAAT). Both urethral swab and first-catch urine samples can be used for NAAT. As first catch urine is as sensitive and less invasive than a urethral swab, this is the investigation of choice.

A 22-year-old female attends your practice complaining of feeling 'sore' in the genital area. She has had multiple sexual partners recently and has not always used barrier contraception. You find small red blisters on the vulva and vagina.How is the cause of her symptoms usually diagnosed?

The most likely diagnosis in this case is genital herpes- a sexually transmitted infection caused but the herpes simplex virus. Ideally, this should be diagnosed and managed in a GUM clinic and choice of treatment is aciclovir 200mg five times a day as well analgesia and possibly local anaesthetic cream. Genital herpes is diagnosed via NAAT testing on swab.

A 56-year-old female is being investigated for a cough. She has recently returned from a trip to India 7 weeks ago and has been experiencing a persistent cough since returning. There has been a loss of appetite, weight loss, and fatigue. She mentions that she often wakes up in the morning to find that the bedsheets are damp. You also find that she has a raised temperature on examination, and there is evidence of a raised, red rash on her legs. The examination of the chest is unremarkable.Given the likely diagnosis, which of the following is the gold-standard investigation for assessing drug sensitivities in this condition?

The most likely diagnosis in this question is tuberculosis (TB). This is evidenced by a chronic cough associated with night sweats, lethargy, loss of appetite, and weight loss. A fever and erythema nodosum are also evident in the question stem.Sputum culture is the gold standard investigation for assessing drug sensitivities in TB. It is more sensitive than sputum smear and nucleic acid amplification tests.The Heaf test and the Mantoux test have been used to screen for latent TB, and are inappropriate for assessing drug sensitivities in active TB.

A patient presents to their GP 2-days following a 1-month volunteering trip to sub-Saharan Africa. They have been feeling very feverish recently and have recorded their temperature to be above 38.0ºC on several occasions. They report feeling very tried recently. They appear jaundiced. On examination they have a tender abdomen.On performing a blood test you discover the patient is hypoglycaemic and has a low platelet count. The diagnosis is confirmed on performing a blood-film.What is the most likely cause of the underlying diagnosis?

The most likely underlying diagnosis is malaria. This is most commonly associated with Sub-Saharan Africa (about 85% of all cases). It cause symptoms such as fever, hepatosplenomegaly, diarrhoea and jaundice. It is also associated with anaemia, thrombocytopaenia rosetting of red blood cells and auto-agglutination of RBCs. Severe signs of malaria include acidosis and a parasitaemia >2%.The commonest cause of malaria is Plasmodium falciparum.

A 54-year-old man presents to the emergency department with a fever, confusion, and a dry cough. He has a past medical history of HIV diagnosed 21 years ago for which he is taking lifelong antiretroviral therapy and metastatic lung cancer.A junior doctor attempts to take blood from him and sustains a needlestick injury.What factor determines the risk of HIV transmission the most in this scenario?

The number of viral copies in the patient's blood is correct. Of the factors listed, the viral load (number of viral copies) in a patient's blood determines the risk of HIV transmission the most. A higher viral load is associated with an increased risk.

A 55-year-old man is admitted to the hospital with suspected sepsis following a percutaneous transhepatic cholangiogram used as part of treatment for cholangiocarcinoma.A doctor is asked to take a venous blood gas sample from the patient and sustains a needlestick injury. They are concerned about the risk of HIV transmission.What factor determines the risk of this complication the most?

The number of viral copies in the patient's blood is correct. This is the single most important factor to determine the risk of HIV transmission following a needlestick injury and is measured using a blood test. A higher number of copies (viral load) is associated with an increased risk of transmission.

A 20-year-old female who recently visited the jungles of Peru for 7 days presents to your clinic. She became ill on the 5th day of her trip with fever, diffuse pain in her legs and lethargy. A few days later she felt much better, however, as of today she deteriorated with visible jaundice, high fever and multiple episodes of vomiting. On examination, there are no obvious skin changes other than jaundice.What is the most likely diagnosis?

The pattern of disease is most consistent with yellow fever. Classically it will present in two phases where the patient experiences a brief remission in between. Yellow fever is mostly concentrated in Africa but it still persists in some rural areas of South America. It can present very quickly with non-specific symptoms and it has an incubation period of 2-14 days which is fitting with this patients history. Malaria tends to present with a cyclical fever with an incubation period of over 7 days. Dengue has an incubation period of 4 to 10 days. Hepatitis B 40 to 160 days. Leptospirosis can present similarly, however, it has an incubation period of 7 to 21 days.

Pneumococcal vaccine

The pneumococcal polysaccharide vaccine is offered to all adults over the age of 65 years and those with: asplenia or splenic dysfunction chronic respiratory disease: COPD, bronchiectasis, cystic fibrosis, interstitial lung disease. Asthma is only included if 'it requires the use of oral steroids at a dose sufficient to act as a significant immunosuppressant' chronic heart disease: ischaemic heart disease if requiring medication or follow-up, heart failure, congenital heart disease. Controlled hypertension is not an indication for vaccination chronic kidney disease (at stages 4 and 5, nephrotic syndrome, kidney transplantation) chronic liver disease: including cirrhosis and chronic hepatitis diabetes mellitus if requiring medication immunosuppression (either due to disease or treatment). This includes patients with any stage of HIV infection cochlear implants patients with cerebrospinal fluid leaks

A 28-year-old intravenous drug user presents to your GP clinic: he is a frequent non-attender and you note he has been recently been discharged from hospital following a diagnosis of chronic hepatitis C.Given his new diagnosis which vaccination does he now qualify for?

The pneumococcal vaccine should be offered to patients with chronic hepatitis Those patients diagnosed with chronic hepatitis should be offered both the annual influenza vaccine and the pneumococcal vaccine, which is given one a one-off basis. The pneumococcal vaccine is only given on a one-off basis, as it is not a rapidly mutating condition, unlike influenza.

A 29-year-old woman develops severe vomiting four hours after having lunch at a local restaurant. What is the most likely causative organism?

The short incubation period and severe vomiting point to a diagnosis of Staphylococcus aureus food poisoning.

TB - mx

The standard therapy for treating active tuberculosis is:Initial phase - first 2 months (RIPE) Rifampicin Isoniazid Pyrazinamide Ethambutol (the 2006 NICE guidelines now recommend giving a 'fourth drug' such as ethambutol routinely - previously this was only added if drug-resistant tuberculosis was suspected) Continuation phase - next 4 months Rifampicin Isoniazid The treatment for latent tuberculosis is 3 months of isoniazid (with pyridoxine) and rifampicin OR 6 months of isoniazid (with pyridoxine)Patients with meningeal tuberculosis are treated for a prolonged period (at least 12 months) with the addition of steroidsDirectly observed therapy with a three times a week dosing regimen may beindicated in certain groups, including: homeless people with active tuberculosis patients who are likely to have poor concordance all prisoners with active or latent tuberculosis

A 19-year-old woman presents to the emergency department with a 48-hour history of fever, shortness of breath, myalgia, diarrhoea, and abdominal pain. She has just got off a 9-hour flight from America where she has been for 3 months. She denies any headache or neck stiffness. Her only past medical history is polycystic ovarian syndrome for which she is on the combined oral contraceptive pill, and her last menstrual period began 5 days ago.On examination, she has a diffuse erythematous rash on the upper and lower extremities, with peeling of the skin on her palms and soles. She has a temperature of 39.5°C, a pulse rate of 132 bpm, a blood pressure of 90/62 mmHg and a respiratory rate of 32 breaths/minute.What is the most likely diagnosis?

The symptoms of fever, hypotension and rash with desquamation (peeling) of the palms and soles of feet are characteristic of staphylococcal toxic shock syndrome. Her last menstrual period starting 5 days ago is also a clue - in this case, it is likely that prolonged placement of a tampon has caused staphylococcal toxic shock syndrome (the cause of ~50% of cases). Her recent 9-hour flight may have contributed to her leaving her tampon in for too long.

Tetanus vaccination

The tetanus vaccine is a cell-free purified toxin that is normally given as part of a combined vaccine.Tetanus vaccine is currently given in the UK as part of the routine immunisation schedule at: 2 months 3 months 4 months 3-5 years 13-18 years This, therefore, provides 5 doses of tetanus-containing vaccine. Five doses is now considered to provide adequate long-term protection against tetanus.

lye disease presentation

There are three clinical stages of disease: Stage 1: Localized disease, lasts several weeks.Tick bite (remembered in 75%)Flu-like symptomsRegional lymphadenopathyErythema Chronicum Migrans (circular target shapred lesion seen in 80% within 30 days)Borrelia lymphocytoma - blue patch on the earlobe, nipple or scrotum (common in children) Stage 2: Early disseminated diseases, lasts days to months. Continued flu-like symptoms Neuroborreliosis: facial nerve (single or bilateral) and other cranial nerve palsies, aseptic meningitis, encephalitis polyradiculitis and Bannwarth's Syndrome, peripheral mononeuritis Cardiovascular: myocarditis, heart block and other arrhythmias, pericarditis. Early painful arthritis Stage 3: Late disseminated disease, lasts months to years Arthritis: recurrent attacks, usually affecting large joints such as the knee. Usually non-destructive Late neurological disorders: polyneuropathy, chronic encephalomyelitis, dementia, psychosis Acrodermatitis chronica atrophicans: blue- red discoloration and swelling at extensor surfaces, may be associated with peripheral neuropathy. Controversial cause of fibromyalgia and chronic fatigue syndromes.

A 28-year-old woman presents to her GP, concerned because she was bitten by a tick 2 days ago whilst hiking in the Lake District. She managed to remove the tick herself and says she has not noticed a rash. She feels generally well in herself and has no past medical history other than migraines. On examination, no rash is visible. What should the GP offer?

There is no need for prophylactic antibiotics for Lyme disease in asymptomatic patients bitten by a tick

The question here is asking for you to first make a diagnosis and then decide which investigation is going to give a definitive diagnosis. The scenario describes someone with systemic symptoms suggestive of malignancy (pyrexia of unknown origin and weight loss) as well as features in keeping with a haematological disorder (lymphadenopathy and lymphocytosis). The two differentials here would be chronic lymphocytic leukaemia and lymphoma, the second one being the more likely diagnosis due to the presence of lymphadenopathy.

Therefore the correct answer is a lymph node biopsy as this would provide a diagnosis of either of these conditions while a bone marrow aspirate would be more likely to pick up a leukaemia but not be as helpful in making a formal diagnosis of lymphoma.

A 30-year-old woman presents with a headache, myalgia, fever and a cough. On auscultation her chest is clear but she has a temperature of 38.6ºC.

These are typical flu symptoms. Influenza is much more common than Mycoplasma pneumonia.

A 34 year old male presents to the Emergency Department with a week history of fevers and non-bloody diarrhoea. Temperatures of 39 degrees were recorded at home which develop predictably every two days, and are accompanied with rigors and night-sweats. He has been working for a charity in rural Kenya for the past year. Which of the following investigations is most likely to be diagnostic?

Thick and thin blood films for Malaria The most important diagnosis that needs to be excluded in a returning traveler from the tropics with pyrexia is Malaria, as this is imminently life-threatening. Although the history of diarrhoeal illness could point towards a gastroenteric infection, diarrhoea alone can also be a symptom of systemic illness generally so this should not be overlooked. The history of fevers every two days (known as a tertian pattern) is characteristic of Plasmodium vivax or ovale species 70%

An 18 year old British woman presents to the Emergency Department one week after coming back from a trekking holiday in the Amazon rainforest in Brazil. She feels unwell and has diarrhoea as well as a fever and muscle pain. On examination, she has no stigmata of liver disease. Which investigation will make a definitive diagnosis in this case?

Thick and thin blood smears This case is highly suspicious for Malaria, which is an endemic disease in the Amazon. Therefore, thick and thin blood smears are the single best investigation in making a definitive diagnosis. You will see the parasite on the film in varying stages of its life cycle. You may also see lysed red blood cells cells as malaria causes a haemolytic anaemia as it reproduces inside the red blood cell. An important differential to consider in a patient returning from the Amazon is Yellow Fever, however that is less likely compared to Malaria and more so given there are no features of liver involvement 49%

An incidental finding is reported on the otherwise normal chest X-ray of a 65-year-old priest. A small (1-2cm) calcified nodule is visible in the lateral area of the right mid zone. The patient has never smoked and has no family history of respiratory disease, but spent many years working in a rural hospital in South Africa.What is this finding likely to represent?

This gentleman has potential exposure to tuberculosis through his work in rural South African hospitals, where there is a high prevalence of TB. The nodule described is a calcified Ghon focus. This indicates that he has had a primary TB infection in the past, which became contained in a granuloma (the Ghon focus) and over time has calcified. TB bacteria may still be present in the lesion and he could develop active disease, particularly if he becomes immunocompromised (see below).

A patient has been diagnosed with HIV. He remembers a period of illness 3 years before his diagnosis when he had a rash and sore throat for several weeks.How long after HIV infection were these symptoms likely to have occurred?

This individual is describing seroconversion illness, part of the process of HIV infection. It is important to know when seroconversion occurs, as before this time an infected patient could have a falsely reassuring negative result on blood testing. This illness can be nonspecific, but can often present with the triad described above (rash, pharyngitis, and fever). It commonly occurs between 3-12 weeks post-infection and symptoms are caused by the body's production of HIV antibodies.

A 42-year-old male attends your practice complaining of a two-day history of pain 'down below'. He has been suffering from urinary frequency and nocturia. Digital rectal examination reveals a tender, boggy prostate.How would you manage him in the first instance alongside painkillers?

This is a case of acute prostatitis, which can be quite distressing to patients. This should be treated with a 14-day course of ciprofloxacin 500mg BD and reviewed thereafter as a further two-week course may be required.

A 42-year-old male attends your practice complaining of a two-day history of pain 'down below'. He has been suffering from urinary frequency and nocturia. Digital rectal examination reveals a tender, boggy prostate.How would you manage him in the first instance alongside painkillers?

This is a case of acute prostatitis, which can be quite distressing to patients. This should be treated with a 14-day course of ciprofloxacin 500mg BD and reviewed thereafter as a further two-week course may be required.Nitrofurantoin would be an appropriate choice if it was simply a suspected urinary tract infection, however not in prostatitis.

A 32-year-old female patient attends your practice complaining of severe frontal facial pain, fever, and rhinorrhoea. This has been ongoing for nearly two weeks and she is now feeling more unwell. She has tried over the counter nasal sprays but with no effect.Her observations are: Temperature: 38.1ºC Pulse: 96 bpm Blood pressure:118/80 mm/Hg She asks you to prescribe some antibiotics to help with her symptoms, she has no known allergies.Which of the following would be your first-line choice?

This is a case of sinusitis. Sinusitis is usually caused by viral infections and therefore treatment is not usually indicated if symptoms are present for less than 10 days. Beyond this, steroid nasal sprays are usually the first-line treatment, unless antibiotics are indicated.In this case, her symptoms have been present for nearly two weeks. Her fever and slightly raised heart rate are consistent with her being unwell and would therefore be indications for antibiotics in this case.In acute uncomplicated sinusitis, first-line therapy as per NICE guidelines would be phenoxymethylpenicillin. In those who are penicillin-allergic, doxycycline or clarithromycin can be used.The second-line choice would normally be co-amoxiclav.

A 32-year-old female patient attends your practice complaining of severe frontal facial pain, fever, and rhinorrhoea. This has been ongoing for nearly two weeks and she is now feeling more unwell. She has tried over the counter nasal sprays but with no effect.Her observations are:Temperature: 38.1ºCPulse: 96 bpmBlood pressure:118/80 mm/HgShe asks you to prescribe some antibiotics to help with her symptoms, she has no known allergies.Which of the following would be your first-line choice?

This is a case of sinusitis. Sinusitis is usually caused by viral infections and therefore treatment is not usually indicated if symptoms are present for less than 10 days. Beyond this, steroid nasal sprays are usually the first-line treatment, unless antibiotics are indicated.In this case, her symptoms have been present for nearly two weeks. Her fever and slightly raised heart rate are consistent with her being unwell and would therefore be indications for antibiotics in this case.In acute uncomplicated sinusitis, first-line therapy as per NICE guidelines would be phenoxymethylpenicillin. In those who are penicillin-allergic, doxycycline or clarithromycin can be used.The second-line choice would normally be co-amoxiclav.

A mother brings in her 4-year-old girl, (Lily), who has been drowsy and irritable all day. Lily has a raised temperature and has recently developed a reddish-purple rash on her legs and bum, which mum says doesn't go away under a glass. On examination, you find her to be withdrawn and persistently crying despite Mum's attempts to soothe, she is hypertensive with a low-normal heart rate of 70 bpm. You suspect a diagnosis of meningitisWhat is the next step to confirm the diagnosis in this case?

This is a difficult question requiring the application of knowledge. Lily has typical signs of meningococcal septicaemia, she has developed a petechial rash. The 'glass test' is a simple way to check to see if the rash is blanching, if it doesn't disappear under glass it is likely non-blanching. Lily is also showing signs of Cushing's reflex, a sign of raised intracranial pressure (ICP). Cushing's reflex is bradycardia and hypertension in response to compression of the cerebrum. The fact that this has also developed to be meningococcal sepsis as indicated by the purpuric rash is a second contraindication to LP. While an LP is the gold standard for diagnosis of meningitis, raised ICP is an absolute contraindication to this. Therefore to diagnose in this instance whole blood PCR and blood cultures is the best answer. Whole blood PCR will show if N.meningitides is present and blood cultures will likely also show this but may take a few days to get a result.

A 65-year-old woman with COPD attends surgery with a 2-day history of worsening dyspnoea, productive cough, and fever. On examination, she has basal crackles on the right side.

This is an infective exacerbation of COPD, with the fever and basal crackles indicating a bacterial cause. The most common cause in COPD is Haemophilus influenzae.

A 23-year-old man presents with a three day history of general malaise and low-grade temperature. Yesterday he developed extensive painful ulceration of his mouth and gums. On examination his temperature is 37.4ºC, pulse 84 / min and there is submandibular lymphadenopathy. What is the most likely diagnosis?

This man has gingivostomatitis, a characteristic feature of primary herpes simplex virus infection Discuss (1)Improve

A 73-year-old man presents to the emergency department with a two-week history of shortness of breath and dry cough. He has no relevant past medical history.On examination, his saturations are 94% on 4 litres of oxygen. His heart rate is 91/min and his blood pressure is 96/63mmHg. His temperature is recorded at 38.4ºC. On exposure, several patches with a 'target' appearance are noted. what further ix will confirm the responsible pathogen?

This man has presented to the emergency department with pneumonia. The findings of erythema multiforme as well as bilateral consolidation on chest x-ray are classical findings for pneumonia caused by Mycoplasma pneumoniae, for which serology is diagnostic.

A 23-year-old female calls 111 due to worsening diarrhoea. She has had 5 episodes of watery diarrhoea in the past 24 hours since flying back from Colombia. Her diarrhoea is associated with severe abdominal cramps, nausea and retching, and feeling faint. She denies any blood in her stool.What is the most likely causative organism for her symptoms?

This patient has a history which correlates strongly with infection with E. coli. She has non-bloody, watery diarrhoea alongside abdominal cramping and nausea. Her history of returning from Columbia correlates with this diagnosis as this is deemed amongst the 'high risk' areas for contracting travellers' diarrhoea (other locations include Latin America, Middle East, and most of Africa).

A 14-year-old boy attends the emergency department with a 1-day history of a maculopapular rash. He presented to his general practitioner 2 days earlier with coryzal symptoms and a sore throat and he was prescribed a course of oral antibiotics.On examination, he has a low-grade fever of 37.8ºC. He has a widespread maculopapular rash over his face and torso. His throat is erythematous with enlarged tonsils and there is palpable tender posterior cervical lymphadenopathy.What is the most appropriate investigation to diagnose the underlying condition?

This patient has a likely diagnosis of glandular fever, a viral illness usually caused by the Ebstein-Barr virus. Symptoms include a sore throat, fever, and general malaise. On examination, it is common to see palpable cervical lymphadenopathy and an erythematous throat. In the later stages of the illness, splenomegaly may appear. It is more common in teenagers and is commonly spread via respiratory droplets and has been nicknamed the "kissing disease". This child was given a course of antibiotics by his general practitioner. In patients with glandular fever, if amoxicillin is given, a non-specific maculopapular rash appears over the skin. Glandular fever is diagnosed by the Monospot test, a blood test that detects the presence of heterophil antibodies made following infection with Ebstein-Barr virus.

A 32-year-old man presents with a history of fevers along with dysentery varying in severity but being profuse at times. The patient is unsure exactly but believes he may have had symptoms for nearly 6 months.The patient has no known medical conditions and is not on any medications. On further questioning, he reports the symptoms started approximately 3 months after returning from a rural charity trip in Central Africa.He has right upper quadrant discomfort on palpation, but examination is otherwise normal.What is the most likely diagnosis?

This patient has presented with infective dysentery (bloody diarrhoea) following travel to a tropical region, with potential poor sanitation. The patient's symptoms both started following an extended incubation period and have continued for an extended period. These timeframes, along with the common features of profuse, bloody diarrhoea, right upper quadrant pain (likely due to liver involvement) and fever are most in keeping with amoebiasis caused by Entamoeba histolytica. With its ability to form liver abscesses, amoebiasis infections can persist for months, even years and it should be considered in any dysentery case with an extended incubation period.

Mrs Fraser, a 40-year-old woman is referred to the emergency department by her GP who tells you that she has developed new-onset balance issues. Her GP says that 1 week ago Mrs Fraser developed coryzal-type symptoms and 3 days ago she had developed a lower respiratory tract infection (LRTI) at which point she was started on oral co-amoxiclav.Mrs Fraser tells you that yesterday morning she woke up and noticed paraesthesia in her hands and feet. She then noticed bilateral weakness in her legs which has since become worse. She now also has severe shooting pains in the anterior aspect of her lower limbs.On examination, Mrs Fraser has normal tone, 5/5 power, normal coordination and sensation in all 4 limbs. She is unable to stand without support however and is unable to walk. She tells you this is due to weakness in her legs which makes her feel as though she will fall. You are unable to elicit any reflexes on examination. You also note that she has bilateral crackles on chest auscultation.Her chest X-ray shows bilateral peribronchial and lower-zone patchy opacification.Which of the following tests would you be most likely to identify the causative organism of this presentation?

This patient has presented with signs of a peripheral demyelinating condition similar to Guillain-Barre which we must consider could be secondary to her concurrent illness. The worrying signs above are her loss of reflexes.Her chest X-ray shows an atypical sounding pneumonia and her symptoms fit with this given her dry cough and flu-type symptoms during the early stage of the illness.Mycoplasma is an atypical organism which is known to cause immune-mediated neurological diseases, some of which can present as an ascending weakness. It can also cause other, less usual sensory signs such as paraesthesia or neuropathic pain as above. There would be a reasonable chance of mycoplasma serology coming back positive in this case. You would need to consider adding treatment to cover atypical LRTI.

A 57-year-old man presents with worsening breathlessness, haemoptysis and fatigue. He does not have any known past medical history. He is pyrexic on admission. On examination, there is bronchial breathing heard over the right upper lobe and the right lower lobe is dull to percussion. There is no evidence of finger clubbing, rash or peripheral oedema. Heart sounds are normal. Chest X-ray shows patchy areas of consolidation and right-sided pleural effusion. Sputum microscopy reveals acid-fast bacilli.Which is the following should be part of your next steps in management?

This patient has pulmonary tuberculosis (TB). Next steps include at least 3 sputum cultures, screening for immunosuppression including an HIV test - and so this is the correct answer from the options above - and antibiotic therapy. HIV infection may alter the treatment of TB and treatment of HIV infection may lead to a more rapid resolution of TB.

A 63-year-old farmer presents with a severely painful right leg. She accidentally impaled herself with a pitchfork whilst moving hay 12 hours ago. The wound was approximately 1cm deep, stopped bleeding within 20 minutes, and she covered it with a plaster. Over the last 12 hours, the pain has been increasing steadily and is now very severe. She has no other symptoms. She takes metformin for type 2 diabetes.On examination, her observations are heart rate 85/minute, respiratory rate 18/min, blood pressure 124/82mmHg, temperature 38.5ºC. There is a 1cm wound on her lower right calf. It is surrounded by an area of prominent erythema and swelling that is approximately 15cm at its widest diameter. There is also a very dark purple area around the wound. There is one blister with foul-smelling discharge. The wound is very tender.An emergency x-ray demonstrates air bubbles in the tissues.Swabs are taken and broad-spectrum antibiotics are started. What is the most likely organism to be isolated?

This patient has radiologically-demonstrated gas gangrene, also known as myonecrosis. The causative organism is almost always clostridial and is typically Clostridium perfringens. It is important to note that having a peripheral vascular disease increases the risk of developing gangrene of all causes. C perfringens is a Gram-positive spore-forming bacillus.

A 67 year patient with known emphysema presents to the Emergency Department with a two week history of cough productive of blood stained sputum. Chest X-Ray shows a circular area of dense right upper lobe consolidation. Despite seven days of intravenous antibiotics (piperacillin and tazobactam) his condition has not improved. An urgent inpatient bronchoscopy reveals no endobronchial lesion but broncho-alveolar lavage reveals an underlying pathogenic organism. Ziehl-Nielson staining is negative. What organism would you suspect?

This patient is likely to have developed an aspergilloma in an emphysematous cavity, which explains the lack of improvement with broad spectrum intravenous antibiotics, haemoptysis and chest X-Ray findings. Moraxella and pseudomonas are usually sensitive to piperacillin + tazobactam and do not classically cause clinical haemoptysis. M. tuberculosis is unlikely given the negative Ziehl-Nielson staining. Burkholderia is typically an infective organism in cystic fibrosis patients, not those with emphysema. Discuss (3)Improve

A 44-year-old presents with a 3-hour history of perineal and testicular pain, redness, and rash. He reports sudden onset, severe pain that is worst over the rash site with reduced sensation to the surrounding skin. His observations are heart rate 97 beats/min, respiratory rate 18 with 98% oxygen saturation in room air, temperature 36.9ºC, blood pressure 122/93mmHg. Examination shows an erythematous rash over the perineum and testicles which the patient believes to have spread since he last checked 30 minutes ago, the cremasteric reflex is present, and both testicles are of equal height. His only past medical history is type 2 diabetes for which he takes dapagliflozin.What is the most likely diagnosis?

This patient is presenting with an acute rash alongside testicular and perineal pain. All the options listed can cause testicular/perineal pain, however, there are features in the vignette that indicate that the most likely answer is necrotising fasciitis. These include: Diabetes mellitus comorbidity. Use of an SGLT-2 inhibitor. Rapidly spreading rash and severe pain. Reduced sensation (hypoaesthesia) to light touch in the area.

A 27-year-old woman presents to the emergency department with a 2-day history of crampy abdominal pain, bloody diarrhoea, and a fever. She denies any previous episodes. She has returned from Ghana two months ago. In Ghana, she volunteered as a teaching assistant for three months. She tells you that she has also attended a barbecue four weeks ago.What is most likely to be underlying her symptoms?

This patient is presenting with bloody diarrhoea two months following a trip to Ghana. Amoebic dysentery is the most likely cause in this patient given this history.

A 21-year-old woman presents to the emergency department after falling unwell soon after a trip to India. She has had a fever for nine days, associated with malaise and a headache. She is also experiencing constipation with generalised abdominal pain.On examination, her abdomen is soft and non-tender. A blanching rash of erythematous maculopapular lesions is noted on her torso. Her temperature is 39ºC and her heart rate is 55 beats per minute.Whilst she has had her childhood vaccinations, she admits that she did not attend any travel vaccine clinic before her trip.What is the likely diagnosis?

This patient presents with typical features of typhoid fever which is caused by infection with Salmonella typhus. Typical features, as seen in this patient, include fever (which is prolonged), influenza-like symptoms, abdominal pain, constipation, 'rose' spots and relative bradycardia. The suspicion is heightened as the patient has been to the Indian subcontinent, where typhoid fever is endemic and has not had any travel vaccinations. Typhoid fever should be suspected in any returning traveller with prolonged fever and is typically included in travel vaccinations.

A 50 year old male is brought by ambulance to A+E with a 4-hour history of high fever, development of a widespread rash, and collapse. He has a background of Type II Diabetes Mellitus, and has no allergies to medications. He has had no recent travel abroad. On examination, he is systemically unwell with severe hypotension, tachycardia and a temperature of 40 degrees. There is a diffuse macular rash covering 90% of his body, which blanches on the application of pressure. A wound is noted on his right shin, which appears mildly erythematous and his wife recalls he sustained the injury yesterday while gardening. What is the most likely underlying diagnosis?

Toxic Shock Syndrome The patient has presented with shock, fever, and erythroderma. Considering the wound on his shin, the diagnosis of Toxic Shock Syndrome has to be considered. Diagnosis will rest on cultures and swabs from the wound itself, and ruling out other differentials 51%

Toxoplasmosis gondii pregnancy

Toxoplasma gondii is incorrect. It is not linked to causing foetal hydrops, however, it can result in premature birth if acquired during pregnancy, leading to congenital toxoplasmosis. This can present as hydrocephalus, microcephaly, intracranial calcifications, eye conditions, epilepsy, psychomotor and mental retardation, low platelets, and anaemia.

A 25 year old man returns from a trip to Malawi and presents to his GP. He is worried as he swam in snail-infested fresh water and has now developed symptoms of abdominal pain, diarrhoea and haematuria. What is the best treatment for this patient?

Two doses of Praziquantel This is a classical description of Schistosomiasis exposure. It is important to note that Schistosomiasis requires treatment even if the patient is asymptomatic, as the infection can have a long lag time. The complications of not being treated early are severe, so all patients who have been exposed should be treated Praziquantel is the drug of choice. Kills adults but not the eggs or migrating schistosomula Hence needs to be given again 2-3 months after exposure to allow for the development of the worms. Steroids are needed in acute Katayama Fever to suppress hypersensitivity reaction.

A 30-year-old immunosuppressed cardiac transplant patient on tacrolimus attends the emergency department feeling tired, feverish and reports to have neck and groin swellings for 1 week.On examination, she has generalised lymphadenopathy with prominent cervical lymph nodes. Her examination is otherwise unremarkable. Her observations are unremarkable other than a temperature of 37.6ºC.In her history, you note that she works in a veterinary surgery.She is investigated and is diagnosed with Toxoplasmosis gondii .Which of the options is the most appropriate for this patient's management?

Toxoplasmosis is a parasitic disease whose main reservoir is cats. It is caused by Toxoplasmosis gondii protozoa. It can cause a flu-like illness in humas with symptoms of malaise, lymphadenopathy and myalgia or may be asymptomatic. Most healthy individuals will clear the infection but it can cause complications such as anaemia, seizure or chorioretinitis. These complications mainly occur in neonates (born to mothers with acute infection) or immunocompromised patients.Immunocompetent patients will often not require treatment.It is important to note that immunocompromised patients are treated with pyrimethamine and sulphadiazine.

A 57 year old man is seen in A+E with a 2-week history of progressively worsening headache, fever, and general malaise. He has a background of chronic kidney disease and end-stage renal failure, for which he received a transplant 1 month ago. He is currently taking Tacrolimus, MMF and prednisolone in addition to medications for type 2 diabetes, hypertension, and angina. He has no known drug allergies. On examination, he is hemodynamically stable with GCS 15/15. There are no focal neurological signs or papilloedema present, but he demonstrates moderate neck stiffness and photophobia. Blood tests and cultures are taken, and a CT head is carried out and found to be clear. A lumbar puncture is performed, and the following CSF results are found: Appearance: clear WCC: 14x10^9 lymphocyte predominant (normal - 0.5-10x10^9 lymphocytes) Glucose serum: 8.9 Glucose CSF: 2 Protein: 0.7 Opening pressure: 35cm H20 (5-25cm H2O) What is the treatment for the most likely cause of disease in this patient?

Treat with 2 weeks IV amphotericin B and oral flucytosine, followed by 8 weeks of fluconazole The CSF results are vague but the high opening pressures, subacute nature of symptoms, and underlying immunocompromise (particularly T cell function), all point to Cryptococcal meningitis being the most likely etiology. Testing for cryptococcal antigen and India Ink staining will be needed to confirm this. Note that treatment consists of initiation therapy with IV Amphotericin B and oral flucytosine for 2 weeks, followed by maintenance therapy with oral fluconazole for 8 weeks, followed by long-term low dose oral fluconazole as maintenance therapy 30%

Tx of leprosy

Treatment of multibacillary leprosy involves the use of dapsone, rifampicin and clofazimine (an immunosuppressive agent, not needed in paucibacillary disease) for 12-24 months. Thalidomide is another treatment option for non-pregnant individuals. Patients commenced on medications need to be monitored closely throughout the course of treatment for immunological complications known as type I and II (erythema nodosum lepromum) reactions, which require hospital in-patient treatment. Side-effects of dapsone include methaemoglobinaemia, agranulocytosis, Stevens-Johnson Syndrome and the DRESS syndrome, and it can also trigger a haemolytic crisis is G6PD deficiency. Clofozamine can cause abnormal skin pigmentation.

what can trimethoprim cause on u+e's

Trimethoprim can cause tubular dysfunction, leading to hyperkalaemia and increased serum creatinine

You are asked to see a 31-year-old woman on the labour ward who has developed a high fever (39.1ºC) 2 days post-partum. She complains of dysuria, and frequency, you suspect a urinary tract infection. She is choosing to breast feed, has severe anaphylactic reactions to penicillins. Which antibiotic would be appropriate?

Trimethoprim is present in milk but is not known to be harmful for short term use. This would be an appropriate choice. Nitrofurantoin should be avoided when breastfeeding - small amounts in milk but can cause haemolysis in G6PD infants.

A 21 year old woman returns from a trip to Malawi and presents to her GP. She is worried as she swam in snail-infested fresh water and later found out a local infection was present. What is the next best step in the management of this patient?

Urine Microscopy This patient has likely been exposed to Schistosomiasis, which is endemic in freshwater lakes in Africa. It is caused by blood-flukes which have part of their life cycle in snails. The most specific way to investigate the infection is with Ova in the urine, however the sensitivity can be less than 50% if there is a light infection 74%

A 50-year-old man has requested a sexual health check-up after recently entering into a relationship with another man. He does not have any symptoms of a sexually transmitted infection (STI) and is otherwise well with no long-term health conditions. His partner has already undergone STI testing which was negative.Besides performing a sexual health screen, what other intervention should be opportunistically offered to this patient?

Vaccination against hepatitis A is recommended for certain people at increased risk of infection, including men who have sex with men (MSM). Other at-risk groups include: Close contacts (household or sexual) of infected persons Travellers to countries where hepatitis A is common People with chronic liver disease Injecting drug users People with haemophilia Those with high-risk of occupational hepatitis A exposure e.g. sewage workers, people who work for organisations where personal hygiene may be poor (e.g. homeless shelters), and people working with primates

This patient most likely has genital ulceration and systemic symptoms from a primary herpes simplex genital infection. This is a common cause of painful genital ulcers and while awaiting swabs, treatment should be commenced.

Valaciclovir twice daily for 10 days

varicella zoster pregnancy

Varicella zoster virus is incorrect. This virus causes chickenpox. If the mother is infected with chickenpox in the first 20 weeks of pregnancy, the baby is at an increased risk of congenital varicella syndrome. This can cause the baby to have multiple conditions such as muscle hypoplasia, developmental delay and gastrointestinal abnormalities. It is not known to be associated with foetal hydrops.

A 33-year-old gentleman attends a routine sexual health clinic screen and is found to be HIV positive, with a CD4 count of 900 cells/mm3. He remains asymptomatic. What is the recommended next best step in terms of treatment?

Viral load is the single greatest determinant of the risk of HIV transmission. When someone is virally suppressed (viral load is undetectable), the risk of HIV transmission is significantly reduced. Antiretrovirals (ART) reduce HIV transmission by lowering viral load and the evidence now supports early initiation of ART irrespective of CD4 count (WHO 2015 guidelines).

HIV: pneumocystis jiroveci pneumonia

Whilst the organism Pneumocystis carinii is now referred to as Pneumocystis jiroveci, the term Pneumocystis carinii pneumonia (PCP) is still in common use Pneumocystis jiroveci is an unicellular eukaryote, generally classified as a fungus but some authorities consider it a protozoa PCP is the most common opportunistic infection in AIDS all patients with a CD4 count < 200/mm³ should receive PCP prophylaxis Features dyspnoea dry cough fever very few chest signs Pneumothorax is a common complication of PCP.Extrapulmonary manifestations are rare (1-2% of cases), may cause hepatosplenomegaly lymphadenopathy choroid lesions Investigation CXR: typically shows bilateral interstitial pulmonary infiltrates but can present with other x-ray findings e.g. lobar consolidation. May be normal exercise-induced desaturation sputum often fails to show PCP, bronchoalveolar lavage (BAL) often needed to demonstrate PCP (silver stain shows characteristic cysts) Management co-trimoxazole IV pentamidine in severe cases aerosolized pentamidine is an alternative treatment for Pneumocystis jiroveci pneumonia but is less effective with a risk of pneumothorax steroids if hypoxic (if pO2 < 9.3kPa then steroids reduce risk of respiratory failure by 50% and death by a third)

A 20-year-old woman comes in for review. 4 weeks ago, she had acute cystitis and was treated with no complications. Urine culture showed no resistant or atypical organisms. She has a history of recurrent lower UTIs and is frustrated as she had 6 in the last year, which has strained her new relationship. She tried cranberry juice and probiotics without benefit.An ultrasound you arranged of her abdomen revealed no abnormality. Her post-void volume was 25 ml.You review her behavioural and self-hygiene measures and find the only identifiable trigger is sexual intercourse.What is the next best action?

Women who suffer regular urinary tract infection following sexual intercourse can be offered post-coital antibiotic prophylaxisImportant for meLess important The correct answer isprescribe oral antibiotic prophylaxis for single-dose use with sexual intercourse.NICE guidance recommends a trial of single-dose oral antibiotics for premenopausal, non-pregnant women with an identifiable trigger such as sexual intercourse, in whom behavioural and self-hygiene measures are ineffective.NICE CKS recommends the following behavioural and self-care advice as 1st line advice for the management of recurrent UTIs: Avoiding douching and occlusive underwear. Wiping from front to back after defecation. Avoiding delay of habitual and post-coital urination. Maintaining adequate hydration. Other behavioural patterns include constipation, washing the genital area with soap (as opposed to water only) and frequent and excessive washing of the genital area. The frequency of sexual intercourse and use of contraceptive diaphragms and/or spermicide gels or condoms has been found to highly correlate with recurrent lower UTIs in young women.

A 9-year-old boy who has recently arrived from India presents with fever. On examination a grey coating is seen surrounding the tonsils and there is extensive cervical lymphadenopathy. What is the most likely diagnosis?

diptheria

A 17-year-old female presents to the GP with a 3-day history of fever, low-grade back pain and rigors. Additionally, she has noticed that she has to go to the toilet more frequently. She has a past medical history of asthma and type 1 diabetes mellitus (T1DM) which are both well controlled.With her likely diagnosis, what is the most common causative agent of this condition?

Young females have the highest incidence of pyelonephritis. As well as this her symptoms and previous diagnosis of T1DM point towards this diagnosis. E coli is by far the most common organism that causes pyelonephritis. Any organism that can ascend up the genitourinary tract can cause pyelonephritis, the other answers are all causes but not the most frequent.

staina

Ziehl-Neelsen stain is typically used to identify mycobacteria. They are not stained in the Gram staining process. Van Gieson and Masson trichrome are histological staining methods for identification of connective tissues. The Von Kossa technique is useful for identifying tissue mineralisation. Discuss (1)Improve

zika virus pregnancy

Zika virus is incorrect. This can result in microcephaly (small head) and affects the baby's brain leading to decreased brain tissue and retinal damage. It does not result in fetal hydrops. DiscussImprove

AFB '(acid fast bacilli) smear is not specific for TB -

all mycobacteria will stain positive including leprosy

A 72-year-old male, who migrated to the UK from India 10 years ago, attends the respiratory clinic with a longstanding cough, and more recently haemoptysis. The patient has never smoked. Physical examination is unremarkable. A chest X-ray was done which shows a crescent of air partially outlining a cavitating mass in the right upper lobe. A CT chest is done in the supine and prone positions and shows movement of the mass within the cavity. The patient has not been previously tested for tuberculosis.What is the most likely cause of the mass?

an aspergilloma may arise in a lung cavity that developed secondary to previous TB. An aspergilloma is a mass caused by the fungus aspergillus, which mainly affects pre-existing cavities, such as those caused by TB, sarcoidosis and lung cancer. signs and symptoms of aspergilloma include haemoptysis, which may be massive and life-threatening, or less commonly haemoptysis or fever. Sarcoidosis is a cause of cavitating lung lesions however this would not explain the mass. There may also be additional radiographic signs such as bilateral hilar/ mediastinal lymphadenopathy, pulmonary fibrosis or pleural effusion. Additional symptoms may include dyspnoea and skin changes such as erythema nodosum. However, sarcoidosis is asymptomatic in up to 50% of patients. small cell lung cancers are another cause of cavitating lung lesions, however due to the lack of a smoking history, this is unlikely. Small cell lung cancers are more likely to affect the hilar or peri-hilar areas as opposed to the upper lobe. A squamous cell lung cancer is unlikely due to the lack of smoking history. these may present with pulmonary symptoms such as dyspnoea, haemoptysis, and a chronic cough, or with a paraneoplastic syndrome such as syndrome of inappropriate ADH (SIADH) or cushings syndrome. TB - is the likely cause of the cavitating lesion in this case due to the history of migration from an endemic area, and the absence of the previous testing. However, tuberculosis would not explain the mass, which is what the question is specifically asking about.

Mycoplasma pneumoniaepatient with anaemia, raised LDH, raised unconjugated bilirubin →

autoimmune haemolytic anaemia

A 34-year-old woman presents to the emergency department complaining of a dry cough, headache and malaise for the past three days. She has also noticed that she is passing loose green stool. She returned from a week-long trip to Pakistan about two weeks ago to see her family. She has no past medical history of note other than asthma for which she takes salbutamol as required and once-daily steroid inhalers. She has also completed a full course of anti-malarial prophylaxis with full concordance and has also received a BCG vaccination when she was younger.Her observations are:Heart rate: 59 beats/minute;Respiratory rate: 14 breaths/minute;Blood pressure: 115/75 mmHg;Oxygen saturation: 97% on room air;Temperature 40.4ºC.On examination, she feels hot to the touch and she looks fatigued. Her heart sounds are normal with no murmurs or added sounds. Her chest is clear. She has mild epigastric abdominal pain, and bowel sounds are normal.Bloods are sent and results are pending. Her chest X-ray and ECG are both normal.Which single additional test will be most useful in establishing the diagnosis?

blood cultures. A relative bradycardia, also known as Fagets sign, is a recognised sign of typhoid fever. It is defined as a heart rate that is slower than expected for the degree of fever. this patients fever is reaching 40.4 degrees so one would expect her heart rate to be faster than 59 beats per minute. Typhoid fever normally presents within 21 days of a return from travel, especially to countries around the Indian subcontinent. It has four stages: the first stage lasts for approximately the first week and often precedes the classical 'pea green diarrhoea' of typhoid. It can present with abdominal pain, malaise, headache, fever, a dry cough and epistaxis. Blood cultures are most appropriate to diagnose the disease here (bone marrow aspiration yields the most sensitive result).

You are working in the urgent care centre, where Sarah, a 46-year-old woman comes to see you with a laceration to her left lower leg. She explains that 3 hours ago she was using a sharp gardening tool to remove weeds in her garden when it slipped and hit deep into her foot.On examination, there is a 5.5cm laceration on the medial aspect of Sarah's left lower leg which is 3cm deep. There is a lot of soil visible inside the wound.Sarah is unsure about her tetanus immunisation history and there is no further information about this in her records.With regards to tetanus prophylaxis, which of the following option is most appropriate if the patient is unaware of their tetanus vaccination history?

booster vaccine and tetanus immunoglobin should be given unless the wound is minor and less than 6 hours old. NICE guidelines state if the person is not immunised or the immunisation status is unknown or uncertain, give an immediate dose of vaccine followed, if records confirm the need, by completion of a full five-dose course to ensure future immunity.

acute pyelonephritis mx

broad spectrum cephalosporin or quinolone

Kaposi's sarcoma

caused by HHV-8 (human herpes virus 8) presents as purple papules or plaques on the skin or mucosa (e.g. gastrointestinal and respiratory tract) skin lesions may later ulcerate respiratory involvement may cause massive haemoptysis and pleural effusion radiotherapy + resection

most common organism causing cellulitis or erysipelas

cellulitis = staphylococcus aureus erysipelas = group A strep

You are reviewing an 82-year-old patient who is an inpatient on the medical ward, who was admitted after falling at home 3-weeks ago. He suffers from urinary incontinence and as such has a permanent indwelling catheter. Whilst reviewing the patient's notes, you see they had a urine culture performed 3-days ago which showed the presence of bacteria in his urine. He denies any current urinary symptoms.What is the most appropriate management of this patient's urinary tract infection (UTI)?

do not need to treat asymptomatic bacteria in catheterised patients.

A 33-year-old man who is HIV positive is admitted to the Emergency Department with confusion and drowsiness. He has been complaining of headaches for a number of days. On examination heart rate is 90/min, blood pressure 104/78 mmHg and temperature is 37.2ºC. He is confused giving a Glasgow Coma Scale (GCS) score of 14. There is no photophobia or neck stiffness.His infectious diseases consultant reports that he is prescribed highly active antiretroviral treatment (HAART) but his compliance is poor and he often misses clinic appointments.A CT brain is requested: CT brain (with contrast): Multiple hypodense regions predominantly in the basal ganglia which show ring enhancement. Minimal surrounding oedema. No mass effect. What is the most likely diagnosis?

cerebral toxoplasmosis. HIV, neuro symptoms, multiple brain lesions with ring enhancement - toxoplasmosis. this is the most common neurological infection seen in HIV, occuring in up to 10% of patients.

someone with a catheter gets a UTI, as well as giving abx what other advcie should you give

change catheter

diarrhoea and hypoglycaemia

cholera

salmonella mx

ciprofloxacin

Campylobacter infection is often self-limiting but if severe then treatment with

clarithromycin

mx of wounds

classify the wound: clean wound - wounds less than 6 hours old, non-penetrating with negligible tissue damage. tetanus prone wound - puncture-type injuries acquired in a contaminated environment e.g. gardening injuries wounds containing foreign bodies compound fractures wounds or burns with systemic sepsis certain animal bites and scratches high-risk tetanus prone wound - heavy contamination with material likely to contain tetanus spores e.g. soil, manure wounds or burns that show extensive devitalised tissue wounds or burns that require surgical intervention patient has had a full course of tetanus vaccines, with the last dose < 10 years ago = no vaccine nor tetanus Ig is required, regardless of the wound severity. Patient has had a full course of tetanus vaccines, with the last dose > 10 years ago if tetanus prone wound: reinforcing dose of vaccine high-risk wounds (e.g. compound fractures, delayed surgical intervention, significant degree of devitalised tissue): reinforcing dose of vaccine + tetanus immunoglobulin If vaccination history is incomplete or unknown reinforcing dose of vaccine, regardless of the wound severity for tetanus prone and high-risk wounds: reinforcing dose of vaccine + tetanus immunoglobulin

A history of Intravenous drug use coupled with a descending paralysis, diplopia and bulbar palsy is characteristic of infection with

clostridium botulinism

Fever, facial spasms, dysphagia in an intravenous drug user

clostridium tetani clostridium botulinism causes flaccid paralysis

how are human bites managed

co-amoxiclav

mx of cellulitis near the eyes

co-amoxiclav = Amoxicillin + clavulanic acid

A 32-year-old man is brought to the emergency department complaining of severe shortness of breath, fever, and dry cough. He gets a chest x-ray and bronchoalveolar lavage, and is found to have Pneumocystis jiroveci pneumonia (PJP). He is not allergic to any medications.What is the best treatment for this condition?

co-trimoxazole

You are called to see a 67 year old male on the renal ward, who has been complaining of fever and malaise. He has been receiving dialysis through a Hickman line, due to end-stage renal failure related to diabetes. On examination, he is pyrexial but hemodynamically stable. Cardiovascular examination reveals a pansystolic murmur heard best in the tricuspid area, and TTE shows a vegetation on the tricuspid valve. Blood cultures show evidence of coagulase-negative staphylococci. Which antibiotic should be started?

coagulase negative = staph epidermis = vancomycin staph epidermis is also the most common cause of infective endocarditis in indwelling catheters

what test is used to diagnose and screen for HIV

combination tests - HIV P24 antigen and HIV antibody. There are multiple different tests used in the screening of HIV. Antibody testing may be positive at 4 weeks post-insult and will be positive by 3 months if there is an infection. p24 (antigen) testing may be positive at 1-week post-insult and will be positive by 4 weeks if there is an infection. Viral load (RNA levels) is used to track the progression of the disease and response to treatment - it is not normally used for screening unless there is a high suspicion of being positive.The gold standard for screening is to do a combined HIV test (p24 antigen and antibody), both at 4 weeks (the earliest both tests may show a positive result) and at 3 months (the earliest both tests will definitely show a positive result). Therefore, p24 antigen and antibody testing in 4 weeks time and in 3 months time is the correct answer.

You review a 45-year-old woman who has been admitted feeling generally unwell. Four months ago she had a renal transplant and has since been taking a combination of ciclosporin and mycophenolate for immunosuppression. For the past three days she has had fever, dyspnoea and a dry cough. A chest x-ray shows bilateral interstitial infiltrates. What is the most likely diagnosis?

cytomegalovirus pneumonitis renal transplant + infection

Glucose in bacterial meningitis

decreased - less than 40% of serum glucose

how to know if something is an osteomyelitis vs cellulitis

deep penetration and increased length of time from injury = osteomyelitis. cellulitis onset would be shorter

Retro-orbital headache, fever, facial flushing, rash, thrombocytopenia in returning traveller

dengue fever

A 29-year-old woman is admitted to the Medical Admissions Unit.She presents with neck stiffness, photophobia and a fever. There is no evidence of a rash. Her GCS is 15.Her lumbar puncture shows an increased opening pressure and looks turbid. There is a raised white cell count and low glucose in the CSF. She is started on ceftriaxone.What other treatment should be given to improve outcomes?

dexamethasone - this improves outcomes by reducing neurological sequelae in the treatment of bacterial meningitis. such as deafness.

A 9-year-old boy who has recently arrived from India presents with fever. On examination a grey coating is seen surrounding the tonsils and there is extensive cervical lymphadenopathy. What is the most likely diagnosis?

diphtheria

A 3-year-old girl presents with a 3 day history of fever and bloody diarrhoea. Over the past 24 hours she has had 5 episodes of loose bloody stools. On examination she has a temperature of 39.6ºC, a heart rate of 175 bpm and her abdomen is soft with generalised tenderness. It is also noted that she has a reduced urinary output. Blood tests show a haemolytic anaemia and raised urea.What is the most likely diagnosis?

e.coli gastroenteritis A short history of bloody diarrhoea is very suggestive of haemorrhagic gastroenteritis which can occur due to a variety of pathogens including Campylobacter, Salmonella and Escherichia coli.In this case, the haemolytic anaemia and raised urea suggest haemolytic uraemic syndrome. Haemolytic anaemia and renal failure form two parts of the classic triad of haemolytic uraemic syndrome. The third part of the triad is thrombocytopenia. It is usually caused by Escherichia coli subtype 0157. Treatment is supportive as antibiotics are contraindicated.

what is the most common cause of viral meningitis in adults?

enterovirus such as coxsackie B virus.

A 30-year-old woman presents to her GP with pain and swelling in her left shin for four days. She is 8 weeks pregnant and has no concerns about her baby. She has suffered from migraines in the past but has no other past medical history. She takes folic acid and has an allergy to penicillin.On examination there is erythema, swelling and tenderness of her left shin. Her temperature is 36.7ºC and other observations are normal. A diagnosis of cellulitis is made.What would be the most suitable treatment for this patient?

erythromycin

patient has cellulitis, is pregnant and has a penicillin allergy. what is the most appropriate treatment option?

erythromycin

antibiotic of choice for cellulitis in pregnancy if the patient is penicillin allergic

erytromycin

e.coli

facultative anaerobic, lactose-fermenting, gram negative rod which is a normal gut commensal. e.coli infections lead to a variety of diseases in humans including: - diarrhoeal illnesses - UTIs - neonatal meningitis serotypes: E.coli may be classified according to the antigens which may trigger an immune respnse: O antigen = originates from lipopolysaccharide layer. K antigen = in capsule = neonatal meningitis secondary to E. coli is usually caused by a serotype that contains the capsular antigen K-1 H antigen = originates in flagellin. E. coli O157:H7 is a particular strain associated with severe, haemorrhagic, watery diarrhoea. It has a high mortality rate and can be complicated by haemolytic uraemic syndrome. It is often spread by contaminated ground beef.

A 34-year-old woman presents to the GP with night sweats and fever. On further questioning, the GP notes that the woman is suffering from both fever and chills and that they have been on and off for alternate days. She also notes muscle aches and a headache.On examination, she has a temperature of 39ºC and mild hepatomegaly. She also mentions that she returned from Haiti 4-weeks ago where she did some voluntary work.Given her presentation what is the most likely diagnosis?

fever on alternating days = malaria

abx for erysipelas

flucloxacillin

extensive otitis externa mx

flucloxacillin

what should be added to the treatment of pneumonia if secondary to influenza

flucloxaicillin

HIV: neurocomplications

focal neurological lesions: toxoplasmosis: - accounts for around 50% of cerebral lesions. - constitutional sx, headache, confusion, drowsiness - CT: usually single or multiple ring enhancing lesions, mass effect may be seen. - mx : sulfadiazine and pyrimethamine. primary CNS lymphoma: - accounts for around 30% of cerebral lesions. - associated with the EBV - CT: single or multiple homogenous enhancing lesions - Tx generally involves steroids (may significantly reduce tumour size), chemotherapy (e.g. methotrexate) + with or without whole brain irradiation. Surgical may be considered for lower grade tumours differentiating between toxoplasmosis and lymphoma is a common clinical scenario in HIV patients. It is clearly important given the vastly different treatment strategies. The table below gives some general differences. Please see the Radiopaedia link for more details. toxoplasmosis: - multiple lesions. - ring or nodular enhancement - thallium SPECT negative Lymphoma: -single lesion - solid (homogenous) enhancement - thallium SPECT positive TB = much less common that toxoplasmosis or primary CNS lymphoma. CT single enhancing lesion. GENERALISED NEUROLOGICAL DISEASE encephalitis: - may be due to CMV or HIV itself. - HSV encephalitis but is relatively rare in the context of HIV, - CT oedematous brain Cryptococcus: - most common fungal infection of CNS. - headache, fever, malaise, Nausea/vomiting, seizures, focal neurological deficit. - CSF: high opening pressure, India ink test positive - CT: meningeal enhancement, cerebral oedema - meningitis is typical presentation but may occasionally cause a space occupying lesion Progressive multifocal leukoencephalopathy (PML) = - widespread demyelination - due to infection of oligodendrocytes by JC virus (a polyoma DNA virus) - symptoms, subacute onset : behavioural changes, speech, motor, visual impairment - CT: single or multiple lesions, no mass effect, don't usually enhance. MRI is better - high-signal demyelinating white matter lesions are seen. AIDS dementia complex: - caused by HIV virus itself - symptoms: behavioural changes, motor impairment - CT: cortical and subcortical atrophy.

when is HPV vaccine offered?

girls and boys aged 12-13 years.

A prison GP is bitten by a patient who is known to have hepatitis B. The GP has a documented full history of hepatitis B vaccination and was known to be a responder. What is the most appropriate action to reduce the chance of contracting hepatitis B?

give hep B booster * Adequate respond : no need immunoglobulin, give booster dose of HBV vaccine. * non-responders : hepB immunoglobulin + full dose vaccine * Still on vaccine course : HepB Immunoglobulin + continue previous vaccine but accelerated

what type of bacteria is e.coli

gram negative bacilli

what type of bacteria is h.infleunza

gram negative coccobacili

Sally is a 29-year-old female who has come to the GP because of a few symptoms she has been experiencing for the past 3 weeks. She first noticed a rash on her thighs which then appeared on her forearms. It is dry, itchy and red. Then she began to have pain in her knees which was worse on movement, the same pain then spread to her left wrist and began to limit her movement. In the last week she has noticed difficulty in moving some of her right fingers from a bent to straight position, it being painful to do so.The doctor takes blood cultures and sends them off.What is most likely to been seen on microscopy?

gram negative diplococci Disseminated gonococcal infection triad = tenosynovitis, migratory polyarthritis, dermatitis

what type of bacteria is e.coli

gram negative rod

what type of bacteria is strep pneumonia

gram positive diplocci

what type of bacteria is L. monocytogenes

gram positive rod

someone has had three doses of their hep B vaccine but their anti-Hbs is not above 100 so what happens

he needs another vaccine

A 36-year-old male presents to the Emergency Department with a 24 hour history shortness of breath and cough. His heart rate is 126 beats per minute with a blood pressure of 103/57 mmHg. His respiratory rate is 28 breaths per minute with oxygen saturations at 93% on air and his temperature is 39.6ºC. 8 months previously he underwent an emergency laparotomy and splenectomy following trauma.What is most likely diagnosis?

he risk of overwhelming post splenectomy infection (OPSI) is greatest in the first two years following splenectomy The combination of pyrexia, hypotension, tachycardia and a recent splenectomy make sepsis the most likely diagnosis. From the organisms given, Pneumococcus is encapsulated and therefore the most likely pathogen. The risk of OPSI is life long, but is greatest in the first two years following the operation. It is a serious life-threatening condition that can proceed rapidly to multi organ failure and death.

pts eat undercooked pork/hog - which hepatitis are they at risk of

hep E

severe hepatitis in a pregnant woman

hep E

A 24-year-old patient has had a sexual encounter and is concerned about his risk of acquiring a sexually transmitted Infection (STI). He has had one incidence of unprotected sex in the last 3 months - a homosexual encounter with a new, asymptomatic male partner, during which they both engaged in anilingus (oro-anal sex). They did not have penetrative sex.Which STI is this patient most at risk of acquiring?

hep a Although Hepatitis B is associated with sexual transmission, anal-oral sex is responsible for the transmission of Hepatitis A

what other medication is given in meningitis alongside the abx

however, the BNF recommend to 'consider adjunctive treatment with dexamethasone (particularly if pneumococcal meningitis suspected in adults), preferably starting before or with first dose of antibacterial, but no later than 12 hours after starting antibacterial; avoid dexamethasone in septic shock, meningococcal septicaemia, or if immunocompromised, or in meningitis following surgery'

oral ulcers cause

hsv 1

caused by HHV-8 (human herpes virus 8)

kaposi's sarcoma

flu-like symptoms and a dry cough, relative bradycardia and confusion. Blood tests may show hyponatraemia

legionella

Bilateral conjunctivitis, bilateral calf pains and high fevers in a sewage worker

leptospirosis

UTI management in adults

lower uti: non pregnant women = local antibiotic guidelines should be followed if available NICE Clinical Knowledge Summaries recommend trimethoprim or nitrofurantoin for 3 days send a urine culture if: aged > 65 years visible or non-visible haematuria pregnant women = if the pregnant woman is symptomatic:a urine culture should be sent in all casesshould be treated with an antibiotic forfirst-line: nitrofurantoin (should be avoided near term)second-line: amoxicillin or cefalexintrimethoprim is teratogenic in the first trimester and should be avoided during pregnancy asymptomatic bacteriuria in pregnant women:a urine culture should be performed routinely at the first antenatal visitClinical Knowledge Summaries recommend an immediate antibiotic prescription of either nitrofurantoin (should be avoided near term), amoxicillin or cefalexin. This should be a 7-day coursethe rationale of treating asymptomatic bacteriuria is the significant risk of progression to acute pyelonephritisa further urine culture should be sent following completion of treatment as a test of cure men = an immediate antibiotic prescription should be offered for 7 days as with non-pregnant women, trimethoprim or nitrofurantoin should be offered first-line unless prostatitis is suspected NICE Clinical Knowledge Summaries state: 'Referral to urology is not routinely required for men who have had one uncomplicated lower urinary tract infection (UTI).' catheterised patients = do not treat asymptomatic bacteria in catheterised patients if the patient is symptomatic they should be treated with an antibiotica 7-day, rather than a 3-day course should be given acute pyelonephritis = for patients with sign of acute pyelonephritis hospital admission should be considered: - local abx guidelines should be followed if available - the BNF currently recommends a broad-spectrum cephalosporin or a quinolone (for non-pregnant women) for 10-14 days

Each one of the following is typically seen in Legionella pneumonia, except: dry cough lymphocystosis hyponatraemia flu-like sx deranged LFTs

lymphocystosis

wbc and protein level in viral meningitis

lymphocytes and mildly raised protein

A 30-year-old man comes for review. He lives with a woman who has recently been diagnosed with having tuberculosis. The man was born in the UK, has no past medical history of note and is currently asymptomatic. What is the most appropriate test to check for latent tuberculosis?

mantoux test The two main tests used for screening in the UK are the Mantoux (skin) test and the interferon-gamma (blood) test. Whilst the use of the interferon-gamma test is increasing it is still reserved for specific situations, none of which apply in this case. Please see the NICE guidelines for more details.

who is given iv dex in meningitis

meningitis in a 6 mnth old

A 41-year-old man with HIV attends his GP complaining of shortness of breath. He has a dry cough which has been present for the past 5 months and occasional symptoms of chest tightness and discomfort.On examination his temperature is 36.7ºC, his heart rate is 69 bpm, his blood pressure is 131/86 mmHg and his oxygen saturations on air are 98%. Examination of his chest is normal but when he is asked to stand and walk 10m, upon returning to his chair his oxygen saturations on air are measured as 89%. A chest x-ray is subsequently performed which shows some patchy opacities in the right apex and bilateral hilar enlargement.What is the most likely causative organism of this man's symptoms?

pneumocystis jirovecii pneumonia = causes desaturation on exercise Given this man's HIV infection, he is susceptible to opportunistic infections, of which Mycobacterium tuberculosis and Pneumocystis jirovecii are the most common to affect the respiratory tract. The non-specific x-ray and chest examination findings are both quite common for either tuberculosis or Pneumocystis pneumonia (PCP) and cannot reliably be used to guide diagnosis. The feature of profound desaturation on exercise however is a very common clinical feature in PCP and this is the key feature of the history and examination which gives the answer in this case.

viral meningitis

meningitis is an inflammation of the leptomeninges and the CSF of the subarachnoid space. viral meningitis is inflammation attributed to a viral agent. In comparison with bacterial meningitis, it may be considered to be a more benign condition and is much more common. Approximately 3,000 cases of confirmed viral meningitis are reported yearly, however, the actual number of cases is likely to be much higher, as patients often do not present to medical services. causes: - non-polio enteroviruses e.g. coxsackie virus, echovirus - mumps - HSV, CMV, herpes zoster viruses - HIV - measles Risk factors: - patients at the extremes of age (< 5 years and the elderly). - Immunocompromised, e.g. patients with renal failure, with diabetes. - IV drug users. clinical presentation: common features: -headache -evidence of neck stiffness -photophobia - often milder than the photophobia experienced by a patient with bacterial meningitis - confusion - fevers less common features: -focal neurological deficits on examination. - seizures: suggests a meningoencephalitis. CSF in viral meningitis: - high lymphocytes - normal glucose - high protein viral PCR may demonstrate an underlying organism. Mx: - whilst awaiting the results of the lumbar puncture, treatment should be supportive and if there is any question of bacterial meningitis or of encephalitis, the patient should be commenced on broad spectrum abx with CNS penetration e.g. ceftriaxone and aciclovir IV. This is particularly the case if the patient has risk factors e.g. elderly, immunocompromised. - generally speaking, viral meningitis is self limiting, with symptoms improving over the course fo 7-14 days and complications are rare in immunocompetent patients. - aciclovir may be used if the patient is suspected of having meningitis secondary to HSV.

The first line treatment in amoebiasis is:

metronidazole

A 55-year-old female is on a course of oral antibiotics for mild diverticulitis. She attends a birthday party and soon afterwards complains of flushing, headache, vomiting and palpitations. Her heart rate is 115 beats/min.She denies any significant past medical history and no history of allergies to any food or medication. She denies recreational drug use but admits having had 1 glass of wine at the party.Which of the following antibiotics is she likely on?

metronidazole The combination of metronidazole and ethanol can cause a disulfiram-like reaction. Clinical features of this include head and neck flushing, nausea and vomiting, sweatiness, headache and palpitations.Cefoperazone, a cephalosporin, is also associated with a disulfiram-like reaction to alcohol.

first line ix for osteomyelitis definitive ix for osteomyelitis

mri bone biopsy

mx of mrsa vs esbl

mrsa = vanc esbl = meropenam

does mycoplasma pneumonia have erythema migrans or multiforme

multiforme

genital wart tx

multiple, non-keratinised warts: topical podophyllum solitary, keratinised warts: cryotherapy

rythema multiforme and bilateral consolidation

mycoplasma pneumoniae

You are reviewing test results. The midstream specimen of urine (MSU) from a 24-year-old woman who is 11 weeks pregnant shows a urinary tract infection. On discussing the result with the patient she does describe some dysuria and 'smelly urine'. What is the most appropriate management?

nitrofurantoin for 7 days

do imunocompotenent pts of toxoplasmosis need mx

no

is asx bacteruria treated in catheterised pts

no

is hep A a DNA virus

no - RNA

is campylobacter always treated

no - only if severe - i.e. if self limiting - clarithromycin

resp exam for pneumocystis jirovecci

no findings

A 35-year-old man who is known to have advanced HIV disease presents with dysphagia and odynophagia. What is the most likely cause of his problems?

oesophageal candidiasis

A 24-year-old female presents to her GP complaining of a painful red patch on her left leg that she says has been gradually increasing in size over the past week. She mentions that she had accidentally grazed her left shin last week following a fall. The patient has no other co-morbidities. On examination, the GP notes a warm tender area of erythema over the patient's left shin that is poorly demarcated. There is also mild pitting oedema around the area of erythema. Observations reveal that the patient is currently non-febrile with a blood pressure of 115/75 mmHg and a heart rate of 72 bpm.What should the GP do next?

offer oral antibiotics. the diagnosis of cellulitis is clinical. No further investigations are required in primary care.

someone was recently diagnosed with chronic hep C - this means they can take the pneumococcal vaccine, how often is this taken?

one-off pneumococcal vaccine

A 23-year-old woman presents to the GP practice with a 24-hour history of dysuria, visible haematuria and general malaise. She is pyrexial. She has no past medical history and takes no regular medications. Examination reveals a soft abdomen with mild suprapubic tenderness. There is no renal angle tenderness and bowel sounds are normal.What is the correct management?

oral abx and MSU to anyone that has visible and non-visible haematuria

A 19-years-old man presents to his general practitioner clinic. He complaints about some painful blisters and ulcers that have appeared on his glans yesterday, associated with some pain during urination.He is sexually active and does not use a condom during intercourse. His last unprotected sexual encounter was 7 days ago.Which one of the following medications would help the patient?

oral aciclovir

sickle cell crisis usually presents with what symptom

pain

Genital ulcers

painful: herpes much more common than chancroid. painless: syphilis more common than lymphogranuloma venereum

what is a gumma

part of tertiary syphilis A gumma is a form of granuloma which is very rare and responds rapidly to treatment

A 25-year-old patient known to be a user of intravenous drugs gets into an altercation with a member of the nursing staff. Unfortunately, he bites the nurse while being restrained by security and they suffer a small bite wound with minimal bleeding. They are worried about the risk of infection transmission, in particular, HIV.What is the most suitable advice regarding post-exposure prophylaxis (PEP)?

post-exposure prophylaxis for HIV is not recommended following human bites. They do not need PEP, but should be given other prophylactic medication is correct. The risk of transmitting HIV through bites is thought to be very low, therefore PEP for HIV is not recommended following human bites. A risk assessment may still be performed, however, no HIV PEP is necessary yet. Human and animal bites still require other prophylactic medications, such as co-amoxiclav, to prevent bacterial infection.

patients with non-falciparum malaria have just been treated with chloroquine and recovers over the next week. what addition tx should be given?

primaquine - Primaquine is used in non-falciparum malaria to destroy liver hypnozoites and prevent relapse. This patient has been treated acutely for a type of non-falciparum malaria with an appropriate acute treatment (chloroquine) which seems to have worked. Since Plasmodium vivax and Plasmodium ovale species both have hypnozoite stages in the liver, these need to be eradicated with an additional therapy to prevent relapse of malarial illness. Primaquine is the appropriate treatment for this.

A 34-year-old man was recently diagnosed with HIV. He was started on a drug regime that included ritonavir. He wanted to know how ritonavir can help to prevent progression to AIDS.Which of the following best describes the mode of action of this drug?

protease inhibitor. 'navir tease a pro' = HIV drugs that end with -navir are protease inhibitors. remember 'its grave/great you integrate' = HIV drugs that end in gravit are integrase inhibitors - e.g. raltegravir.

which TB drug is most likely to cause acute hepatotoxicity

pyrazinamide

Given that the patient is a farmer, this exposes him to zoonotic infections. The presence of fevers and associated transaminitis raises the suspicion for

q-fever caused by coxiella burnetti Its natural reservoir exists within cattle and sheep. It typically presents with fevers, headaches, fatigue and muscle aches. Diagnosis of this is often made by serological testing and is treated with doxycycline. Chronic Q-fever is possible and can present with endocarditis.

mx of IM

reassurance

common cold: 45 year old smoker develops pneumonia: A 6-month-old who is 'chesty', has rhinitis and is off her feeds. Auscultation of her chest reveals bibasal crackles and an expiratory wheeze.

rhinovirus strep pneumoniae RSV

TB screening and diagnosis

screening for latent TB: - The Mantoux test is the main technique used to screen for latent tuberculosis. In recent years the interferon-gamma blood test has also been introduced. It is used in a number of specific situations such as: - the Mantoux test is positive or equivocal - people where a tuberculin test may be falsely negative (see below) Mantoux test = 0.1 ml of 1:1,000 purified protein derivative (PPD) injected intradermally result read 2-3 days later Diameter of induration: <6mm = Negative - no significant hypersensitivity to tuberculin protein. this means that previously unvaccinated individuals may be given the BCG. 6-15mm = positive = hypersensitive to tuberculin protein. Should not be given BCG. May be due to previous TB infection or BCG >15mm = strongly positive - strongly hypersensitive to tuberculin protein. suggests tb infection False negative tests may be caused by: miliary TB sarcoidosis HIV lymphoma very young age (e.g. < 6 months) heaf test = The Heaf test was previously used in the UK but has been since been discontinued. It involved injection of PPD equivalent to 100,000 units per ml to the skin over the flexor surface of the left forearm. It was then read 3-10 days later. dx of active TB: Chest x-ray upper lobe cavitation is the classical finding of reactivated TB bilateral hilar lymphadenopathy Sputum smear 3 specimens are needed rapid and inexpensive test stained for the presence of acid-fast bacilli (Ziehl-Neelsen stain)all mycobacteria will stain positive (i.e. nontuberculous mycobacteria) the sensitivity is between 50-80%this is decreased in individuals with HIV to around 20-30% Sputum culture the gold standard investigationmore sensitive than a sputum smear and nucleic acid amplification testscan assess drug sensitivities can take 1-3 weeks (if using liquid media, longer if solid media) Nucleic acid amplification tests (NAAT) allows rapid diagnosis (within 24-48 hours) more sensitive than smear but less sensitive than culture

diagnostic test for mycoplasma

serology

A 63-year-old man is brought into the emergency department after he was found collapsed on the street. He is known to services as being homeless. His current GCS is 10.Initial observations show a heart rate of 110 beats per minute, a blood pressure of 101/57 mmHg, a respiratory rate of 21 breaths per minute, oxygen saturation of 94% on 15 litres/min and a temperature of 39.1 ºC.On examination, he has severe cellulitis of his left leg from his foot to the upper thigh.He is started on IV fluids.What is the most appropriate antibiotic to prescribe?

severe cellulitis should be managed with co-amoxiclav This man is significantly unwell due to his cellulitis infection. He is showing signs of sepsis. A broad spectrum antibiotic should be chosen, and given intravenously for best effect. IV co-amoxiclav would be an appropriate choice, as this offers good coverage of organisms which are commonly indicated in cellulitis infections. These include gram positive cocci (i.e. Streptococcus spp. and Staphylococcus spp.), anaerobes (i.e. Clostridium spp. and Fusobacterium spp.) and gram negative organisms (i.e. Haemophilus spp.).

A 25-year-old woman presents to the Emergency Department with a two-day history of jaundice. She is 12 weeks pregnant and has no other past medical history. She denies diarrhoea or vomiting but did admit to attending a barbecue two weeks before, where she had concerns about how thoroughly the meat had been cooked. her blood results showed low platelets and high liver enzymes. what is her likely diagnosis?

severe hepatitis in a pregnant woman - think hep E. The correct answer is hepatitis E infection. This patient has severe transaminitis in the first trimester of pregnancy. This should make candidates consider hepatitis E infection, as pregnant patients tend to experience a more severe course of infection than other patient groups. Additional factors supporting the diagnosis of hepatitis E infection include thrombocytopenia, which is a recognised extra-hepatic manifestation of this infection, and the history of recent attendance at a barbecue (hepatitis E is often contracted from undercooked pork).

A 27-year-old man attends the general practitioner (GP) complaining of 'spots' around the head of his penis. He reports that they have always been there and that they have not changed at all. The patient reports that he is not sexually active and has not had any sexual partners in the past.On examination there are multiple flesh coloured papules on the corona of the penis. The GP explains that these are pearly penile papules.What is the single best advice the GP can give the patient?

they are benign and dont need investigating

A 56-year-old female is being investigated for a cough. She has recently returned from a trip to India 7 weeks ago and has been experiencing a persistent cough since returning. There has been a loss of appetite, weight loss, and fatigue. She mentions that she often wakes up in the morning to find that the bedsheets are damp. You also find that she has a raised temperature on examination, and there is evidence of a raised, red rash on her legs. The examination of the chest is unremarkable.Given the likely diagnosis, which of the following is the gold-standard investigation for assessing drug sensitivities in this condition? heaf test mantoux test NAAT sputum culture sputum smear

sputum culture is used to assess drug sensitivities in TB. Heaf and mantoux test are used to screen for latent TB and are inappropriate for assessing drug sensitivities in active TB.

A 40-year-old man develops pneumonia following an episode of influenza. what resp pathogen causes it?

staph aureus

A 40-year-old man is admitted to the intensive care unit following a severe episode of acute pancreatitis. On the third day of his admission he becomes pyrexial. A septic screen is ordered including cultures taken from both peripheral blood and the internal jugular line. There is no signs of infection on the chest x-ray or urine sample. The microbiology laboratory phone to report signs of bacterial infection in the sample from the central line. What is the most likely organism to be isolated?

staph epidermis is the most common organism found in central line infections.

A 21-year-old man presents to the emergency department with a 5-week history of progressive lethargy and intermittent dizziness and palpitations, particularly on exertion and often related to shortness of breath. Today he has had a syncopal episode and is concerned that there may be something wrong with his heart as he has a strong family history of heart disease.He has no past medical history, is an ex-smoker with a 3 pack year history and takes no recreational drugs. He has recently emigrated from India and is working in an office. His weight has been stable for the last year and he reports no change in bowel habit or appetite. He reports an itchy rash on his feet appearing several months ago, but this self resolved. his blood count showed low Hb and high oesinophils. which of the following confirm the cause of the patients symptoms and blood test results?

stool sample for ova, cysts and parasites. This patient has likely been infected with hookworm, explaining the anaemia, eosinophilia and the rash he described. Hookworms (nematodes) are acquired from skin contact with contaminated soil, commonly from walking barefoot in an affected area and are endemic in much of Asia, Africa and South America. The worm migrates from the skin into the intestine and from there causes chronic blood loss, hence causing anaemia. Diagnosis can be made from stool culture with identification of the ova in the patient's faeces.

6 years - 60 years age group are at risk from meninigitis caused by

strep pneumoniae and neisseria meningiditis

gram positive cocci in strands

streptococci

when to have a high suspicion for sepsis (red flag signs)

systolic blood pressure < 90mmHg or > 40mmHg fall from baseline mean arterial pressure < 65mmHg heart rate > 131 per minute respiratory rate > 25 per minute unresponsive or responsive only to voice or pain.

An 18-year-old girl is brought into the emergency department with pyrexia, headache, vomiting, a widespread erythematous rash with desquamation of the rash, particularly on the palms and soles of her feet and confusion. Her temperature is 39.2ºC, oxygen saturation 93% on air, heart rate 123 beats per minute, respiratory rate of 24 breaths per minute and blood pressure 82/50 mmHg.What confers the highest risk for her presentation?

tampon use = increased risk of SSSS

Disseminated gonococcal infection triad

tenosynovitis, migratory polyarthritis, dermatitis

A 64-year-old man developed a polymicrobial wound infection following an emergency operation for a ruptured abdominal aortic aneurysm. He was placed on antibiotic treatment, and 5 days later, he reported developing bad breath and a metallic taste in his mouth. On examination, the patient had a black discolouration of the tongue and hairy appearance of the tongue.What class of antibiotics can lead to this presentation?

tetracycline Black hairy tongue is a temporary, harmless oral condition relatively common condition which results from defective desquamation of the filiform papillae. Despite the name, the tongue may be brown, green, pink or another colour. Predisposing factors include poor oral hygiene, tobacco use, intravenous drug use, and antibiotics (particularly tetracyclines). Discuss (5)Improve

Jarisch-Herxheimer reaction

the Jarisch-Herxheimer reaction is sometimes seen following treatmentfever, rash, tachycardia after the first dose of antibioticin contrast to anaphylaxis, there is no wheeze or hypotensionit is thought to be due to the release of endotoxins following bacterial death and typically occurs within a few hours of treatmentno treatment is needed other than antipyretics if required mx = oral paracetamol

if a question asks what to do if someone has suspected sepsis/describes someone who is septic which option would you choose

the one that is included in the sepsis 6 protocol

genital herpes

there are two strains of the HSV in humans: HSV 1 and HSV 2. Whilst it was previously thought HSV1 accounted for oral lesions (cold sores) and HSV 2 for genital herpes it is known there is is considerable overlap. Features: - painful genital ulceration = may be associated with dysuria and pruritis - the primary infection is often more severe than recurrent episodes = systemic features such as headache, fever, malaise are more common in primary episodes. - tender inguinal lymphadenopathy - urinary retention may occur Investigations: - NAAT is the investigation of choice in genital herpes and are now considered superior to viral culture. - HSV serology may be useful in certain situations such as recurrent genital ulceration of unknown cause. Mx: general measures include = saline bathing, analgesia, topical anaesthetic agents e.g. lidocaine. oral aciclovir = some patients with frequent exacerbations may benefit from longer term aciclovir. Pregnancy: elective caesarean section at term is advised if a primary attack of herpes occurs during pregnancy at greater than 28 weeks gestation women with recurrent herpes who are pregnant should be treated with suppressive therapy and be advised that the risk of transmission to their baby is low

what is klebsiella granulomatis?

this causes painless ulceration but is rare in developed countries.

Necrotising fasciitis

this is a medical emergency that is difficult to recognise in the early stages. it can be classified according to the causative organism: type 1 is caused by mixed anaerobes and aerobes (often occurs post-surgery in diabetics). This is the most common type type 2 is caused by Streptococcus pyogenes risk factors: skin factors: recent trauma, burns or soft tissue infections diabetes mellitusthe most common preexisting medical conditionparticularly if the patient is treated with SGLT-2 inhibitors intravenous drug use immunosuppression the most commonly affected site is the perineum (Fournier's gangrene) features: acute onset pain, swelling, erythema at the affected siteoften presents as rapidly worsening cellulitis with pain out of keeping with physical featuresextremely tender over infected tissue with hypoaesthesia to light touchskin necrosis and crepitus/gas gangrene are late signs fever and tachycardia may be absent or occur late in the presentation mx: urgent surgical referral debridement intravenous antibiotics Prognosis average mortality of 20%

what is treponema denticola

this is one of the main aetiological agents for periodontitis.

A 25-year-old woman presents with painless, itchy skin lesions around her vulva which she has noticed 2 days ago. She describes them as 'lumpy skin tags' and can count about 4-5 discrete lesions. They are non-tender and there is no exudate. She is worried they are not clearing up despite treatment with topical antifungal creams. On further questioning, she engaged in unprotected sexual intercourse with a casual partner 3 months ago.Given the likely diagnosis, what is the most appropriate treatment for her condition?

topical podophyllum Genital wart treatment multiple, non-keratinised warts: topical podophyllum solitary, keratinised warts: cryotherapy

abx to avoid in pregnancy

trimethoprim sulfonamides quinolones

types of necrotising fasciitis

type 1 is caused by mixed anaerobes and aerobes (often occurs post-surgery in diabetics). This is the most common type. type 2 is caused by Streptococcus pyogenes

A relative bradycardia, also known as Faget's sign, is a recognised sign of

typhoid fever. It is defined as a heart rate that is slower than expected for the degree of fever. This patient's fever is reaching 40.4ºC, so one would expect a heart rate faster than 59 beats per minute.Typhoid fever normally presents within 21 days of a return from travel, especially to countries around the Indian subcontinent. It has four stages: the first stage lasts for approximately the first week and often precedes the classical 'pea green diarrhoea' of typhoid. It can present with abdominal pain, malaise, headache, fever, a dry cough and epistaxis. Blood cultures are most appropriate to diagnose the disease here (bone marrow aspiration yields the most sensitive result).

You are reviewing a 31-year-old man in the liver clinic. He is currently on triple therapy for hepatitis C. What is the best way to assess his response to treatment?

viral load

where are swabs taken from for gonorrhoea and chlamydia

vulvovaginal

shiga toxin from shigella - diarrhoea or vomiting

watery diarrhoea

can you give co-trimoxazole in someone who has a penicillin allergy

yes

is acute infective hepatitis a notifiable disease

yes The Public Health England guidance states that notification should be made 'immediately on diagnosis of a suspected notifiable disease' and advises doctors not to 'wait for laboratory confirmation of a suspected infection or contamination before notification'.


Related study sets

Cellular Respiration/ Fermentation Questions

View Set

What is the fundamental problem in economics?

View Set

NEETS MODULE 22 - DIGITAL COMPUTING

View Set

Chapter 69: Caring for Clients with Mood Disorders

View Set

19D Cavalry Scout General Knowledge

View Set

Chapter 6 Financial Institutions

View Set